Sie sind auf Seite 1von 197

1

A pregnant woman presents with the following condition which is bothersome to her. Which of the
following is the most appropriate treatment option based on FDA classifications of medication in
pregnancy?
1

Topical erythromycin/benzoyl peroxide gel
2

Topical tretinoin
3

Tazarotene 0.1% cream
4

Azelaic acid 20% cream
5

Bactrim DS
Q/Q(M)-482157 Report a Problem


A pregnant woman presents with the following condition which is bothersome to her. Which of the
following is the most appropriate treatment option based on FDA classifications of medication in
pregnancy?
4

Azelaic acid 20% cream
This patient has mild-moderate inflammatory acne. Azelaic acid is the only medication listed which falls
under category B. The others listed are category C, except for tazarotene, which is category X
Q/Q(M)-482157 Report a Problem


Pilomatricomas has been associated with which of the following conditions?
1

Acute myelogenous leukemia
2

Multiple sclerosis
3

Crohns Disease
4

Myotonic dystrophy
5

Pneumothorax
2

Q/Q(M)-477401 Report a Problem


Pilomatricomas has been associated with which of the following conditions?
4

Myotonic dystrophy
Answer D. Pilomatricomas are benign tumors that are derived from hair matrix cells. This tumor presents
more frequently in children as a solitary asympotmatic purple red papule or nodule usually on the head
and neck. Mutations in beta catenin is associated with the development of these tumors. Multiple
pilomatricomas may be a marker for the development of myotonic dystrophy. This rare disorder is AD,
and is associated with difficulty relaxing muscles after contraction. In a review of patients with this
disorder, patients tended to have mulitple pilomatricomas. Pilomatricomas can also be found with Turner
syndrome, Rubinstein-Taybi, and Churg Strauss syndromes.
Q/Q(M)-477401 Report a Problem


Lacrimal gland involvement is characteristically seen in patients with:
1

Lymphoma
2

Sarcoid
3

Rosacea
4

Syphilis
5

Pemphigus
Q/Q(M)-477156 Report a Problem


Lacrimal gland involvement is characteristically seen in patients with:
2

Sarcoid
Sarcoid is granulomatous autoimmune condition characterized by the formation of non-caseating
granulomas which may affect any organ system. Lacrimal gland involvement occurs in 15-28% of
patients. Manifestation of lacrimal gland involvement includes bilateral painless swelling.
Q/Q(M)-477156 Report a Problem


A patient presents requesting removal of a tattoo. Examination reveals a tattoo with red, orange, yellow,
and purple pigment. Which laser would be appropriate to treat this tattoo?
1

Q switched Nd:YAG (532nm)
2

Q switched alexandrite (755nm)
3

Q switched ruby (694nm)
4

Nd:Yag (1064nm)
5

Alexandrite (755nm)
Q/Q(M)-482799 Report a Problem


3

A patient presents requesting removal of a tattoo. Examination reveals a tattoo with red, orange, yellow,
and purple pigment. Which laser would be appropriate to treat this tattoo?
1

Q switched Nd:YAG (532nm)
The Q switched Nd:YAG can also be used to treat tan pigment. QS alexandrite and QS ruby can remove
green pigment. QS Nd:YAG is a good choice for patients with darker skin color.
Q/Q(M)-482799 Report a Problem

A young man presents with explosive onset of severe cystic acne with acute, suppurative nodules and
plaques that ulcerate and form a blackish eschar on the trunk as well as the face. Which of the following
is true regarding this entity?
1

Women are more often affected than men
2

P. acnes osteomyelitis presents with lytic changes on x-rays and bone scans
3

The sternoclavicular joint is often involved in this entity
4

Systemic corticosteroids are contraindicated given risk of sepsis
5

High-dose isotretinoin monotherapy is the treatment of choice
Q/Q(M)-480164 Report a Problem

A young man presents with explosive onset of severe cystic acne with acute, suppurative nodules and
plaques that ulcerate and form a blackish eschar on the trunk as well as the face. Which of the following
is true regarding this entity?
3

The sternoclavicular joint is often involved in this entity
Acne fulminans is a rare, explosive form of severe cystic acne affecting young males. Patients may be
systemically ill, with leukocytosis, fever, arthralgias, and myalgias. Lytic changes, indicative of a sterile
osteomyelitis, can be seen on x-ray and bone scans. The sternoclavicular joint and chest wall are most
frequently affected. Treatment is with oral prednisone, intralesional steroids, antibiotics, and isotretinoin.
Q/Q(M)-480164 Report a Problem


Which of the following is true regarding acquired C1 esterase inhibitor deficiency?
1

This condition generally occurs in the first or second decade of life
2

Serum C1q is normal
3

C2 and C4 are both decreased
4

C1 esterase inhibitor may be at normal levels with functional impairment
5

Positive family history is common
Q/Q(M)-480508 Report a Problem


Which of the following is true regarding acquired C1 esterase inhibitor deficiency?
3

C2 and C4 are both decreased
C1 esterase inhibitor is a protease inhibitor that inhibits the catalytic subunits of the first components of
4

the classical pathway. In the absence of C1 esterase inhibitor, activated C1 and plasmin generate
activated C2 kinin, which mediates angioedema. Acquired C1 esterase inhibitor deficiency generally
affects adults or elderly individuals with no family history. Serum C1q is decreased. It occurs in the
setting of lymphoproliferative disease or rheumatologic illness, where idiotype/anti-idiotype immune
complexes consume available C1q and functionally and quantitatively lower the amounts of C1 esterase
inhibitor. It can also occur in the setting of autoimmunity directed against the C1 esterase protein.
Inherited C1 esterase inhibitor deficiency is detected in the first or second decade of life and is autosomal
dominantly inherited. Serum C1q is normal in the inherited form, but there is a defect in the synthesis
and/or function of C1 esterase inhibitor. In both the inherited and acquired forms levels of C2 and C4 are
decreased because of the uncontrolled actions of C1s.
Q/Q(M)-480508 Report a Problem

A 2 year-old boy with crusted skin papules is found to have osteolytic defects and diabetes insipidus.
These features are seen in:
1

Osteogenesis imperfecta
2

Conradi-Hunnerman disease
3

Metastatic small cell lung carcinoma
4

Hand-Schuller-Christian disease
5

Epidermal nevus syndrome
Q/Q(M)-477165 Report a Problem


A 2 year-old boy with crusted skin papules is found to have osteolytic defects and diabetes insipidus.
These features are seen in:
4

Hand-Schuller-Christian disease
Hand-Schuller-Christian disease is a chronic multifocal form of Langerhans cell histiocytosis.70% of
cases occur between the ages of 2 and 6. The four characteristic clinical findings are bone lesions,
diabetes insipidus, exophthalmus, mucocutaneous lesions. Bones lesions are osteolytic and preferentially
involve the calvarium.
Q/Q(M)-477165 Report a Problem

Probiotics, which are cultures of potentially beneficial gut microflora bacteria, have been studied in the
primary prevention of which of the following diseases?
1

Celiac disease
2

Atopic dermatitis
3

Psoriasis
4

Cutaneous T-cell lymphoma
5

Asthma
Q/Q(M)-474298 Report a Problem


5

Probiotics, which are cultures of potentially beneficial gut microflora bacteria, have been studied in the
primary prevention of which of the following diseases?
2

Atopic dermatitis
Probiotics have been studied in the primary prevention of atopic dermatitis by Kalliomaki et al.
Lactobacillus GG cultures were given to pregnant women with a history of atopy to assess the effect of
potentially beneficial gut flora on the prevention of atopic disease in their children. The frequency of
atopic dermatitis in the children in the probiotic group was half that in the placebo group at two years of
life.
Q/Q(M)-474298 Report a Problem


Mutations in which of the following genes has been implicated in the pathogenesis of disseminated
superficial actinic porokeratosis?
1

BRAF
2

K-RAS
3

PTCH
4

PTEN
5

SART3
Q/Q(M)-482839 Report a Problem

Mutations in which of the following genes has been implicated in the pathogenesis of disseminated
superficial actinic porokeratosis?
5

SART3
Disseminated superficial actinic porokeratosis (DSAP) is a disorder characterized by numerous keratotic
macules and papules often localized in a photodistribution. Clinically, they are distinguished by a rim of
keratotic scale and often may have an atropic appearance centrally. Histologically, they often display a
lichenoid infiltrate flanked by characteristic cornoid lamella (inward-bending tiers of parakeratosis with
underlying hypogranulosis and dyskeratosis). Recently, mutations in SART3 (squamous cell carcinoma
antigen recognized by T-cells 3) have been implicated in a Taiwainese patient cohort affected by DSAP.
Of note, while all varieties of porokeratoses have the potential for malignant degeneration and
development of squamous cell carcinoma, lesions of DSAP have the lowest risk.
Q/Q(M)-482839 Report a Problem


Which contact allergen is found in Krazy glue?
1

Formaldehyde
2

Lanolin alchohol
3

Ethyl cyanoacrylate
4

Octyl-dimethyl-paba
5

Mercaptobenzothiazole
Q/Q(M)-476071 Report a Problem
6



Which contact allergen is found in Krazy glue?
3

Ethyl cyanoacrylate
Ethyl cyanoacrylate is a chemical found in Krazy Glue. It can also be found in nail adhesives causing a
dermatitis on the face and fingers, nail dystrophy, and a generalized rash. Formaldehyde is a widely used
chemical found in a variety of applications and is a common allergen. It can be found in paper, skin and
hair products, cosmetics, and permanent press textiles. Lanolin alcohol is found in wool fat, wool wax,
adhesives, cosmetics and pharmaceuticals. Octyl-dimethyl-paba is a chemical found in sunscreens and is
the most frequently used PABA group sunscreen. Mercaptobenzothiazole is a rubber accelerant and is the
most common allergen found in dermatitis to shoes. It is also found in veterinarian products such as flea
and tick sprays and powders. Mercapotbenzothiazole can also be found in cutting oil, antifreeze,
fungicides and photographic film emulsions.
Q/Q(M)-476071 Report a Problem


In the setting of the lupus erythematosus-associated complement deficiency syndrome, which of the
following would most often be observed?
1

low C3 and C4
2

low C2 and C3
3

low C2 and C4
4

low C2 only
5

low C3 only
Q/Q(M)-480514 Report a Problem


In the setting of the lupus erythematosus-associated complement deficiency syndrome, which of the
following would most often be observed?
3

low C2 and C4
In the complement deficiency syndrome, low C2 and C4 are most commonly seen. Photosensitivity,
annular SCLE lesions, and Ro antibody formation are commonly observed.
Q/Q(M)-480514 Report a Problem


Which of the following is a feature of Schnitzler's Syndrome?
1

Monoclonal IgG gammopathy
2

Thromboembolic events
3

Bronchospasm
4

Hematuria
5

Sensorimotor neuropathy
Q/Q(M)-480534 Report a Problem


Which of the following is a feature of Schnitzler's Syndrome?
7

5

Sensorimotor neuropathy
Schnitzler's syndrome presents as episodes of urticarial vasculitis that occur in association with a
monoclonal IgM M component. Fever, lymphadenopathy, hepatosplenomegaly, bone pain, and
sensorimotor neuropathy also occur.
Q/Q(M)-480534 Report a Problem

Which of the following is a manifestation of psoriasis of the nail matrix?
1

Splinter hemorrhages
2

Oil spots
3

Subungual hyperkeratosis
4

Pits
5

Onycholysis
Q/Q(M)-474319 Report a Problem


Which of the following is a manifestation of psoriasis of the nail matrix?
4

Pits
Psoriatic nail changes may be of nail matrix or nail bed origin. Pits are the common finding; splinter
hemorrhages the least. Psoriatic nail changes of matrix origin include: pits (representing focal psoriasis of
the proximal matrix) and leukonychia. Psoriatic nails changes of nail bed origin include: salmon spots,
oil spots, onycholysis, subungual hyperkeratosis, and splinter hemorrhages.
Q/Q(M)-474319 Report a Problem

Which of the following leukodermas has a normal number of epidermal melanocytes?
1

Albinism
2

Vitiligo
3

Piebaldism
4

Waardenburg's syndrome
5

Ziprowski-Margolis syndrome
Q/Q(M)-480477 Report a Problem


Which of the following leukodermas has a normal number of epidermal melanocytes?
1

Albinism
Patients with albinism have a normal number of epidermal melanocytes, however, these melanocytes
synthesize inadequate amounts of melanin. Vitiligo, piebaldism, Waardenburg's syndrome and
Ziprowski-Margolis syndrome all feature a decreased number or total absence of epidermal melanocytes.
Q/Q(M)-480477 Report a Problem
After being treated for several months with doxycycline, this patient develops a gram negative
folliculitis. What is the next appropriate therapy?
8

1

Tetracycline
2

Bactrim
3

Isotretinoin
4

Ceftriaxone
5

Cefepime
Q/Q(M)-476722 Report a Problem



After being treated for several months with doxycycline, this patient develops a gram negative
folliculitis. What is the next appropriate therapy?
3

Isotretinoin
Gram negative folliculitis may occur after prolonged antibiotic therapy for acne vulgaris. It should be
suspected in patients who are well controlled and then suddenly flare. The treatment of choice is
isotretinoin.
Q/Q(M)-476722 Report a Problem



You are consulted to evaluate this patient with tender, warm plaques on his shins. What other physical
signs should you look for?
1

Exophthalmos
2

Uveitis
3

Tachycardia
4

Clubbing of the fingers
9

5

Cough
Q/Q(M)-480561 Report a Problem


You are consulted to evaluate this patient with tender, warm plaques on his shins. What other physical
signs should you look for?
2

Uveitis
Erythema Nodosum (EN) is represented by tender, warm,nodules and plaques, often but not exclusivley
located on the anterior shin. It represents a reactive panniculitis. Causes include, infections
(Streptococcal, tuberculosis, yersina, mycoplasma, campylobacter,salmonella, histoplasmosis,
blastomycosis, coccidiomycosis), drugs (sulfonamides, gold, and OCP's), enteropathies, pregnancy,
hodgkin's disease and lymphoma and sarcoidosis. Loefgren's disease is a varient of sarcoidosis with EN,
hilar adenopathy, fever, uveitis and arthritis. Differential diagnosis includes: pretibial myxedema and
erythema induratum.
Q/Q(M)-480561 Report a Problem




TH2 immune responses:
1

Are associated with cell-mediated immunity
2

Produce IL-6
3

Produce IFN-gamma
4

Produce TNF-beta
5

Produce IL-2
Q/Q(M)-480484 Report a Problem


TH2 immune responses:
2

Produce IL-6
TH1 cells produce IL-2, IFN-gamma, and TNF-beta, and are associated with cell-mediated immunity.
TH2 cells produce IL-4, IL-5, IL-6, IL-10, and IL-13, and are associated with antibody-mediated immune
responses.
Q/Q(M)-480484 Report a Problem


Monotherapy for acne with topical antibiotics is discouraged because of:
10

1

Slow onset of comedolytic action
2

Potential for irritation
3

Lack of anti-inflammatory action
4

Potential for bacterial resistance
5

Poor patient compliance
Q/Q(M)-474321 Report a Problem


Monotherapy for acne with topical antibiotics is discouraged because of:
4

Potential for bacterial resistance
Topical antibiotics reduce the population of P. acnes on the skin, and thus are indirectly anti-
inflammatory. In contrast to topical retinoids, topical antibiotics are not comedolytic. They are generally
well tolerated by patients.
Q/Q(M)-474321 Report a Problem

The first site in body that shows yellowish pigmentation in carotenoderma is
1

Face
2

Trunk
3

Palms and soles
4

Nails
5

Sclera
Q/Q(M)-482111 Report a Problem


The first site in body that shows yellowish pigmentation in carotenoderma is
1

Face
Carotenoderma is yellowish discoloration of the skin secondary to carotenemia. Carotene is excreted by
sebaceous glands and in sweat, so the yellow pigmentation appears first on the face (especially nasolabial
folds and forehead) and then becomes diffusely distributed with accentuation in palms and soles. In
contrast to jaundice, carotenoderma spares mucous membranes and sclera
Q/Q(M)-482111 Report a Problem
In addition to the lesions seen on the feet of this young man he also had geographic tongue, erythematous
plaques on his penis and arthritis, what HLA type is associated with this syndrome?
1

HLA-B27
2

HLA-B51
3

HLA-DR1
4

HLA-DR3
5

HLA-DQW2
11

Q/Q(M)-482070 Report a Problem

In addition to the lesions seen on the feet of this young man he also had geographic tongue, erythematous
plaques on his penis and arthritis, what HLA type is associated with this syndrome?
1

HLA-B27
Reiter syndrome, now referred to as reactive arthritis (ReA), is a condition that most often occurs
following enteric or urogenital infections. Reactive arthritis is associated with human leukocyte antigen
(HLA)B27, although HLA-B27 is not always present in individuals who are HIV+. Bacteria
associated with reactive arthritis are generally enteric or venereal and include the following: Shigella
flexneri, Salmonella typhimurium, Salmonella enteritidis, Streptococcus viridans, Mycoplasma
pneumonia, Cyclospora, Chlamydia trachomatis, Yersinia enterocolitica, and Yersinia
pseudotuberculosis.
Q/Q(M)-482070 Report a Problem


What is the most common variant of morphea in children?
1

Plaque
2

Generalized
3

Bullous
4

Deep (morphea profunda)
5

Linear
Q/Q(M)-482550 Report a Problem


What is the most common variant of morphea in children?
5

Linear
Linear morphea is the most common presentation in children, comprising between 40% to 70% of
children with morphea. This subtype includes linear morphea of the extremity, en coup de sabre, or
progressive facial hemiatrophy, all of which may be accompanied by underlying tissue atrophy.
Q/Q(M)-482550 Report a Problem


Presence of which of the following autoantibodies is diagnostic of SLE and not reported in patients with
other connective tissue diseases?
1

anti-U1RNP
12

2

anti-dsDNA
3

anti-Ro
4

anti-La
5

anti-Sm
Q/Q(M)-480522 Report a Problem


Presence of which of the following autoantibodies is diagnostic of SLE and not reported in patients with
other connective tissue diseases?
5

anti-Sm
Anti-Sm is diagnostic of SLE and not reported in patients with other connective tissue diseases. It is
found in 15-40% of patients with SLE. Most patients with anti-Sm also have antibodies to U1RNP, but
the converse is not true. anti-U1RNP is found in 100% of patients with MCTD and in 30% of patients
with SLE (the majority of patients with positive U1RNP have SLE rather than MCTD). Anti-dsDNA
correlates with renal involvement in SLE. Anti-Ro and anti-La antibodies are found in LE and Sjogren's,
and strongly associated with photosensitivity.
Q/Q(M)-480522 Report a Problem


Perifolicular depigmentation is a characteristic cutaneous manifestation of:
1

Dermatomyositis
2

Secondary syphilis
3

Scleroderma
4

Discoid lupus
5

Sarcoidosis
Q/Q(M)-477175 Report a Problem
Perifolicular depigmentation is a characteristic cutaneous manifestation of:
3

Scleroderma
Scleroderma is a systemic disease which may affect almost any organ in the body. Cutaneous findings are
characterized by symmetric, thickening of the skin. Dyspigmentation may occur with salt and
pepper appearance due to perifollicular depigmentation.
Q/Q(M)-477175 Report a Problem

Which of the following hormones bind the androgen receptor?
1

Dehydroepiandrosterone
2

Androstenedione
3

Dihydrotestosterone
4

Dehydroepiandrosterone and dihydrotestosterone
5

Dehydroepiandrosterone, androstenedione, and dihydrotestosterone
13

Q/Q(M)-480496 Report a Problem


Which of the following hormones bind the androgen receptor?
3

Dihydrotestosterone
Only testosterone and dihydrotestosterone bind the androgen receptor, thus adrenal androgens
(androstenedione and dehydroepiandrosterone) virilize only in so far as they serve as precursors for
testosterone and dihydrotestosterone.
Q/Q(M)-480496 Report a Problem

A patient presents with hemorrhagic onycholysis. The drug class most commonly associated with this
finding is:
1

Quinolone antibiotics
2

Systemic retinoids
3

Calcineurin inhibitors
4

Taxanes
5

Tetracyclines
Q/Q(M)-482324 Report a Problem


A patient presents with hemorrhagic onycholysis. The drug class most commonly associated with this
finding is:
4

Taxanes
Taxane probably cause nail changes more commonly than other drugs. Cutaneous toxicity has been
reported with taxanes and includes erythema and desquamation, involving primarily the hands. Taxanes
exert their cytotoxic effect by reversibly binding the -subunit of tubulin, thereby inducing tubulin
polymerization and inhibiting microtubule depolymerization. A balance between polymerization and
depolymerization is needed for normal microtubule function. Taxanes disrupt this balance, leading to
arrest at the G2/M phase of the cell cycle.
Q/Q(M)-482324 Report a Problem


Diarrhea, Dementia and a photosensitive dermatitis are associated with a deficiency of which vitamin?
1

Niacin
2

Biotin
3

Thiamine
4

Riboflavin
5

Pyridoxine
Q/Q(M)-480472 Report a Problem
14




Diarrhea, Dementia and a photosensitive dermatitis are associated with a deficiency of which vitamin?
1

Niacin
Niacin (Vitamin B3) deficiency is associated with a photosensitive dermatitis, diarrhea and dimentia. The
photosensitive dermatitis classically involves the face, neck and upper chest (Casal's necklace) and
forearms (as pictured). Other findings may include angular cheilitis and thickening and
hyperpigmentation of skin overlying bony prominences.
Q/Q(M)-480472 Report a Problem



What is the most common presentation of psoriatic arthritis?
1

Asymmetric oligo- or polyarthritis
2

Symmetric polyarthritis
3

Spondylitis (axial)
4

Distal interphalangeal joint (DIP) disease
5

Arthritis mutilans
Q/Q(M)-480461 Report a Problem

What is the most common presentation of psoriatic arthritis?
1

Asymmetric oligo- or polyarthritis
Psoriatic arthritis affects 20-40% of patients with psoriasis. Most (~80%) of psoriatic arthritis patients are
rheumatoid factor negative. All of the answer choices represent possible presentations of psoriatic
arthritis, but asymmetric oligo- or polyarthritis is the most common presentation.
Q/Q(M)-480461 Report a Problem
15




In this patient with an autoimmune disorder, which autoantibody would be indicative of increased risk of
pulmonary disease?
1

U1RNP
2

DsDNA
3

anti-Jo-1 antibody
4

Mi-2 antibody
5

Scl-70 antibody
Q/Q(M)-476825 Report a Problem


In this patient with an autoimmune disorder, which autoantibody would be indicative of increased risk of
pulmonary disease?
3

anti-Jo-1 antibody
The patient depicted has dermatomyositis. Autoantibodies to anti-Jo-1 antibody targets histidyl transfer
RNA synthetase. In dermatomyositis correlates with the development of pulmonary disease.
Q/Q(M)-476825 Report a Problem


All of the following drugs have been reported to cause rash similar to the attached image except
1

Hydroxyurea
2

D-penicillamine
3

Statins
4

Phenytoin
5

Captopril
Q/Q(M)-482135 Report a Problem
16




All of the following drugs have been reported to cause rash similar to the attached image except
5

Captopril
The image shows symmetrical erthematous to violaceous plaque on dorsal hands with some periungual
erythema and telangiectasia. Skin biopsy was consistent with interface dermatitis and mucin. Many drugs
have been reported to cause dermatomyositis-like picture, which include hydroxyurea, D-penicillamine,
statins, phenytoin and alfuzosin (alpha antagonist for BPH). Captopril is not associated with DM-like
rash.
Q/Q(M)-482135 Report a Problem


This woman has restricted ability to open her mouth with tight bound down skin of her hands and
pulmonary fibrosis. In addition to a positive ANA with a nucleolar pattern, what is the most common
autoimmune antibody that is associated with pulmonary fibrosis?
1

Topoisomerase I antibodies (formerly Scl-70)
2

Fibrillarin antibodies
3

Anti-U3RNP antibodies
4

Anti-PM-Scl antibodies
5

Anti-thyroglobulin antibodies
Q/Q(M)-482066 Report a Problem



This woman has restricted ability to open her mouth with tight bound down skin of her hands and
pulmonary fibrosis. In addition to a positive ANA with a nucleolar pattern, what is the most common
autoimmune antibody that is associated with pulmonary fibrosis?
17

1

Topoisomerase I antibodies (formerly Scl-70)
This woman has scleroderma which is a systemic disease characterized by skin induration and
thickening. The cutaneous findings are accompanied by various degrees of tissue fibrosis and chronic
inflammatory infiltration in numerous visceral organs, prominent fibroproliferative vasculopathy, and
humoral and cellular immune alterations. Antinuclear antibodies are present in about 95% of the patients,
usually with a speckled or homogenous pattern. A nucleolar pattern, although less common, is more
specific for systemic sclerosis. Topoisomerase I antibodies (formerly Scl-70) are present in
approximately 30% of patients with diffuse disease (absent in limited disease) and are associated with
pulmonary fibrosis. Anticentromere antibodies are present in about 60-90% of patients with limited
disease and are rare in patients with diffuse disease. Fibrillarin antibodies and antibodies to U3
ribonucleoprotein (RNP) may also be present but are more common in patients with skeletal muscle
involvement. Anti-U3RNP is present mostly in patients with diffuse disease with overlap syndromes.
Anti-ThRNP is present mostly in limited disease and is associated with more extensive visceral disease.
Anti-PM-Scl is present in limited and overlap states and is associated with myositis and renal
involvement.
Q/Q(M)-482066 Report a Problem



Increased chylomicrons are a feature of which type of hyperlipoproteinemia?
1

Type I
2

Type IIa
3

Type IIb
4

Type III
5

Type IV
Q/Q(M)-480549 Report a Problem

Increased chylomicrons are a feature of which type of hyperlipoproteinemia?
1

Type I
Type I (familial lipoprotein lipase deficiency or apoprotein CII deficiency) hyperlipoproteinemia has
increased chylomicrons as a feature, and presents with eruptive xanthomas and lipemia retinalis. For the
other hyperlipoproteinemias, the associated lipid abnormalities are as follows: Type IIa -- increased LDL;
Type IIb -- increased LDL and VLDL; Type III -- increased IDL; Type IV -- increased VLDL. Type V
has increased chylomicrons and increased VLDL.
Q/Q(M)-480549 Report a Problem


A patient presents with cutaneous lesions suggestive of sarcoidosis. You consider ordering a serum ACE
level to help with the diagnosis. What is the sensitivity and specificity of checking an ace level in this pt?
1

Sensitivity: 60 Specificity: 80
2

Sensitivity: 90 Specificity: 60
3

Sensitivity: 95 Specificity: 80
4

Sensitivity: 60 Specificity: 60
18

5

Sensitivity: 90 Specificity: 25
Q/Q(M)-482232 Report a Problem


A patient presents with cutaneous lesions suggestive of sarcoidosis. You consider ordering a serum ACE
level to help with the diagnosis. What is the sensitivity and specificity of checking an ace level in this pt?
1

Sensitivity: 60 Specificity: 80
Serum ACE levels are positive in only approximately 60% of patients with sarcoidosis. However the
specificity is only 80%. Therefore, serum ace levels are less helpful in diagnosis (they can be helpful in
following treatment response.) For example, if your pretest clinical suspicion of sarcoidosis is 25% (i.e. 1
of 4 likely possible etiologies based on differential diagnosis), then of a 1000 patients tested, 250 will
have sarcoidosis. 160 of these will be detected by an abnormal serum ace level, but 150 of the non-
sarcoidosis patients will also have an abnormal ace level giving you a positive predictive value of just
over 50% making it a poor diagnositic test.
Q/Q(M)-482232 Report a Problem



Pyostomatitis vegetans is characteristically associated with which systemic disease?
1

Pemphigus vulgaris
2

Ulcerative colitis
3

Lichen planus
4

Lymphoma
5

Rheumatoid arthritis
Q/Q(M)-477349 Report a Problem


Pyostomatitis vegetans is characteristically associated with which systemic disease?
2

Ulcerative colitis
Pyostomatitis vegetans is a pustular, vegetative variant of pyoderma gangrenosum, found in the oral
mucous membranes. It is most frequently associated with inflammatory bowel disease.
Q/Q(M)-477349 Report a Problem


Best treatment option for this stable type of vitiligo is
1

Phototherapy with narrow band UVB
2

Excimer laser
3

Oral prednisone
4

20% monobenzyl ether of hydroquinone
5

Nitrogen mustard
Q/Q(M)-482122 Report a Problem
19




Best treatment option for this stable type of vitiligo is
4

20% monobenzyl ether of hydroquinone
The picture shows generalized type of vitiligo or vitiligo universalis. Patients who have widespread
disease with only a few areas of normally pigmented skin in exposed sites can be treated with
depigmenting agents. The patients must be carefully chosen, i.e. adults who recognize that their
appearance will be altered significantly and who understand that depigmentation requires lifelong care of
the skin (sunscreens, protective clothing, etc.). The most commonly used agent is monobenzyl ether of
hydroquinone (MBEH) 20% applied twice daily to the affected areas for 9-12 months or longer.
Monobenzyl ether of hydroquinone is a potent irritant and/or allergen, and an open use test should be
performed before more widespread application. It normally takes 1-3 months to initiate a response, and a
loss of pigment can occur at distant sites. Although depigmentation from MBEH is considered
permanent, repigmentation (especially perifollicular) can be seen following a sunburn or even intense sun
exposure. Monomethyl ether of hydroquinone in a 20% cream can be used as an alternative to MBEH.
Side effects include contact dermatitis, exogenous ochronosis and leukomelanoderma en confetti.
Phototherapy and excimer laser are not good or practical choices for this type of vitiligo. Nitrogen
mustard is not used in vitiligo.
Q/Q(M)-482122 Report a Problem



What is the most common malignancy associated with this condition in this female?
1

Brain cancer
2

Thyroid cancer
3

Breast cancer
4

Ovarian cancer
5

Liver cancer
Q/Q(M)-476818 Report a Problem
20




What is the most common malignancy associated with this condition in this female?
4

Ovarian cancer
Dermatomyositis is an autoimmune polymyositis with characteristic cutaneous findings. Patients with
dermatomyositis should be screened for a underlying malignancy. Risk factors for having an underlying
malignancy include a negative ANA, adult age, and female gender. Ovarian cancer is one of the most
frequently associated dermatomyositis.
Q/Q(M)-476818 Report a Problem


What is the treatment of choice for this condition?
1

Nystatin
2

Fluconazole
3

Better oral hygiene
4

Penicillin
5

Acyclovir
Q/Q(M)-476814 Report a Problem



What is the treatment of choice for this condition?
3

Better oral hygiene
Black tongue is associated with poor oral hygiene, the use of medications, and radiation to the head and
21

neck region. In many cases, simply brushing the tongue with a toothbrush or using a commercially
available tongue scraper is sufficient improve the condition.
Q/Q(M)-476814 Report a Problem


Patients with Dermatitis Herpetiformis are most likely to have:
1

Antibodies to BPAg2
2

Antibodies to transglutaminase 3
3

Mutations in transglutaminase I
4

Mutations in laminin 5
5

Mutations in plectin
Q/Q(M)-482146 Report a Problem


Patients with Dermatitis Herpetiformis are most likely to have:
2

Antibodies to transglutaminase 3
In dermatitis herpetiformis, antibodies are found to transglutaminase 3, and the direct immunofluorescent
studies show granular IgA and C3 in the dermal papillae. Antibodies to BPag2 are found in bullous
pemphigoid. Mutations in plectin are found in EBS with muscular dystrophy. Mutations in laminin 5 are
found in patients with JEB,Herlitz type. Mutations in transglutaminase I are found in pateints with
lamellar ichthyosis and non bullous congenital ichthyosiform erythroderma.
Q/Q(M)-482146 Report a Problem



Histologic examination of this chronic pruritic plaque on the back, which of the following stains would
NOT be positive?
1

Von Kossa
2

Cotton dyes
3

Crystal violet
4

PAS
5

Thioflavin T
22

Q/Q(M)-476863 Report a Problem


Histologic examination of this chronic pruritic plaque on the back, which of the following stains would
NOT be positive?
1

Von Kossa
Macular amyloid is a form of keratin derived amyloid, which is typically located between the shoulder
blades. Many stains are used to identify amyloid in the skin including Congo red, cotton dyes, crystal
violet, PAS and thioflavin T.
Q/Q(M)-476863 Report a Problem




The differential diagnosis for the attached image should include all of the followings except
1

Candidiasis
2

Lichen planus
3

Contact dermatitis
4

Inverse psoriasis
5

Erythrasma
Q/Q(M)-482121 Report a Problem



The differential diagnosis for the attached image should include all of the followings except
23

5

Erythrasma
The image shown is inverse type of psoriasis. Although psoriatic lesions are classically distributed on the
extensor surfaces (the elbows, knees, and lumbosacral regions), lesions may also be found in a flexural
distribution with involvement of the axillae, groin, perineum, central chest, and umbilical region.
Differential diagnosis include all other choices in question. Although erythrasma might be included in
differentials in case of groin involvement only, it is unlikly for eryhrasma to presnt on penis with well
demarctaed erythamtos plaques. Erythrasma is a superficial bacterial infection of the skin caused by C.
minutissimum. It is characterized by asymptomatic, well-demarcated, reddish brown, slightly scaly
patches in the groin, axillae, gluteal crease, or inframammary regions, and less often the interdigital
spaces of the feet. Erythrasma is frequently confused with a dermatophyte infection, it can be
differentiated from tinea infection by the characteristic coral red fluorescence seen when viewed under
Wood's lamp illumination (due to the production of porphyrins by the corynebacteria.
Q/Q(M)-482121 Report a Problem


This 10 year-old girl presents to your office. These lesions have been present for months. She denies any
other systemic complaints. What will you tell her parents?
1

They should expect her to get more lesions in non sun-exposed areas
2

She is more likely to develop systemic lupus erythematosus than an adult with these lesions
3

She is less likely than an adult to develop renal disease
4

She is more likely than an adult to develop cardiovascular disease
5

No hematologic labs are required for evaluation
Q/Q(M)-480560 Report a Problem


This 10 year-old girl presents to your office. These lesions have been present for months. She denies any
other systemic complaints. What will you tell her parents?
1

They should expect her to get more lesions in non sun-exposed areas
2

She is more likely to develop systemic lupus erythematosus than an adult with these lesions
3

She is less likely than an adult to develop renal disease
4

She is more likely than an adult to develop cardiovascular disease
24

5

No hematologic labs are required for evaluation
Q/Q(M)-480560 Report a Problem


This 10 year-old girl presents to your office. These lesions have been present for months. She denies any
other systemic complaints. What will you tell her parents?
2

She is more likely to develop systemic lupus erythematosus than an adult with these lesions
Discoid lupus erythematosus (DLE) presents with plaques characterized by scarring, atrophy, follicular
plugging, and scale and photosensitivity. Children presenting with DLE have a higher incident of
developing systemic lupus (SLE) than adults. Because of progression from DLE to SLE, children should
be screened and followed with antinuclear antibodies and anti- DNA antibodies. Children and
adolescents have a higher incidence of renal involvement. Treatment for DLE includes topical steroids,
oral steroids, and hydroxychloroquine.
Q/Q(M)-480560 Report a Problem


You are suspicious for a new diagnosis of lupus in a patient recently treated with penicillamine. Which
antibodies would you expect to have been induced by this medication?
1

anti-dsDNA
2

anti-histone
3

anti-SSA
4

anti-SSB
5

anti-Mi-2
Q/Q(M)-480516 Report a Problem


You are suspicious for a new diagnosis of lupus in a patient recently treated with penicillamine. Which
antibodies would you expect to have been induced by this medication?
1

anti-dsDNA
Penicillamine induces native systemic lupus erythematosus, associated with anti-dsDNA antibodies, in
contrast to drug-induced lupus erythematosus which has been associated with exposure to hydralazine,
procainamide, sulfonamides, penicillin, anticonvulsants, minocycline, and INH and is associated with
anti-histone antibodies.
25

Q/Q(M)-480516 Report a Problem

What autoimmune disease is associated with a PRP-like eruption?
1

Systemic lupus
2

Rheumatoid arthritis
3

Dermatomyositis
4

Scleroderma
5

Pemphigus erythematosus
Q/Q(M)-482872 Report a Problem

What autoimmune disease is associated with a PRP-like eruption?
3

Dermatomyositis
The Wong type of dermatomyositis is characterized by erythematous, hyperkeratotic, follicular papules .
It can have a limited or generalized distribution. It can evolve into full-blown dermatomyositis with
classic cutaneous features. This rare variant bears a striking clinical resemblance to PRP. It can be
distinguished on skin biopsy. It has been reported in both adults and children. Some reports suggest the
follicular papules are confined to the skin overlying the extensor knees and elbows and this variant may
be more common in Asian patients.
Q/Q(M)-482872 Report a Problem

What is the treatment of choice for neurotic excoriations?
1

Risperidone
2

Doxepin
3

Olanzapine
4

Gabapentin
5

Diphenhydramine
Q/Q(M)-482564 Report a Problem


What is the treatment of choice for neurotic excoriations?
2

Doxepin
Doxepin is the treatment of choice for patients with neurotic excoriations. It has both antidepressant and
antipruritic effects. Combined psychiatric and pharmacologic intervention is recommended. Other agents
such as risperidone, olanzapine, gabapentin, and diphenhydramine are not drugs of choice for this
disorder.
Q/Q(M)-482564 Report a Problem



The "flag sign" is seen in which of the following conditions?
26

1

psoriasis
2

sarcoidosis
3

hidradenitis suppurativa
4

kwashiorkor
5

telogen effluvium
Q/Q(M)-480485 Report a Problem


The "flag sign" is seen in which of the following conditions?
4

kwashiorkor
The "flag sign" is a striking physical finding most readily seen in long and dark hair. Hair that has grown
during periods of inadequate nutrition is pale so alternating bands of light and dark can be seen along a
single strand (reflecting periods of adequate and inadequate nutrition). The "flag sign" can be seen with
other nutritional disorders as well.
Q/Q(M)-480485 Report a Problem


A young man treated with minocycline developed blue-black discoloration with acne scars at his cheeks.
A Perls stain would show:
1

Increased melanin at the basal layer of the epidermis
2

Black staining granules within macrophages
3

Blue staining granules within macrophages
4

Muddy brown pigment granules
5

Giant melanosomes
Q/Q(M)-474305 Report a Problem
A young man treated with minocycline developed blue-black discoloration with acne scars at his cheeks.
A Perls stain would show:
3

Blue staining granules within macrophages
Perls stain stains hemosiderin (iron) blue. Fontana Masson stains melanin black. Minocycline
hyperpigmentation often shows positive staining for both iron and melanin.
Q/Q(M)-474305 Report a Problem



Which of the following is true regarding topical therapies for psoriasis?
1

Vitamin D3 analogues deactivate salicylic acid
2

Retinoids are effective in decreasing lesional erythema
3

Anthralin can cause irreversible staining of peri-lesional skin
4

Calcipotriol is deactivated by UV light
27

5

Anthralin inhibits PMNs and monocytes
Q/Q(M)-480389 Report a Problem

Which of the following is true regarding topical therapies for psoriasis?
5

Anthralin inhibits PMNs and monocytes
Anthralin, in addition to possessing antiproliferative activity on human keratinocytes, has strong anti-
inflammatory effects by inhibiting PMNs and monocytes. Vitamin D3 analogues are inactivated by
salicylic acid and should be used after UV light (calcipotriol absorbs UV). Retinoids reduce scaling and
plaque thickness, but do not generally decrease lesional erythema. Anthralin can stain hair purple and
cause reversible brownish discoloration of surrounding skin.
Q/Q(M)-480389 Report a Problem
What is the diagnosis?
1

Psoriasis
2

Lichen planus
3

Balanitis circinata
4

Candida
5

Syphilis
Q/Q(M)-476810 Report a Problem




What is the diagnosis?
3

Balanitis circinata
Balanitis circinata presents as sharply demarcated, serpiginous ulcers or plaques on the penile head.
Balanitis circinata is usually seen in Reiter's syndrome which has a classic triad of arthritis, urethritis, and
conjunctivitis. The syndrome typically occurs post-infection of the GI or urinary tract.
Q/Q(M)-476810 Report a Problem

Almost all patients with SLE have positive ANAs. A patient can have ANA-negative SLE if they only
make antibodies to:
1

ssDNA
2

Sm
28

3

U1RNP
4

Ro
5

dsDNA
Q/Q(M)-480526 Report a Problem

Almost all patients with SLE have positive ANAs. A patient can have ANA-negative SLE if they only
make antibodies to:
1

ssDNA
ANA-negative SLE is uncommon. The ANA has a high negative predictive value and low positive
predictive value. ANA-negative SLE can result if a patient only makes antibodies to ssDNA, which is not
detected by most tests.
Q/Q(M)-480526 Report a Problem



A 62 year-old man with chronic renal failure on hemodialysis presents with carpal tunnel syndrome, bone
cysts, and spondyloarthropathy. A diagnosis of amyloidosis is suspected. Which of the following is true
regarding his most likely diagnosis?
1

AA amyloid is likely causative
2

AL amyloid is likely causative
3

beta 2-microglobulin is likely causative
4

keratin-derived amyloid is likely causative
5

Amyloid P component will not be found in affected tissues
Q/Q(M)-480504 Report a Problem


A 62 year-old man with chronic renal failure on hemodialysis presents with carpal tunnel syndrome, bone
cysts, and spondyloarthropathy. A diagnosis of amyloidosis is suspected. Which of the following is true
regarding his most likely diagnosis?
3

beta 2-microglobulin is likely causative
The patient likely has dialysis-related amyloidosis. In this condition, beta 2-microglobulin is the protein
component altered by uremia. Carpal tunnel syndrome, bone cysts, and spondyloarthropathy commonly
result. Amyloid P component and ground substance are found in all forms of amyloidosis; the protein-
derived amyloid fibers differ among the various forms.
Q/Q(M)-480504 Report a Problem

Which of the following is true regarding atopic dermatitis?
1

Over 80% of affected individuals present in the first year of life
2

Natural measles infection has been noted to improve atopic dermatitis
3

Most children develop worsening of atopic dermatitis if they develop asthma later in childhood
29

4

Staphylococcus aureus is found in ~25% of atopic dermatitis skin lesions
5

Increased expression of cathelicidins such as LL 37 has been noted in atopic dermatitis
Q/Q(M)-480481 Report a Problem

Which of the following is true regarding atopic dermatitis?
2

Natural measles infection has been noted to improve atopic dermatitis
Natural measles infection has been noted to improve atopic dermatitis. Atopic dermatitis typically begins
in infancy, with ~50% in the first year of life and an additional 30% between 1 and 5 years. Most children
with AD eventually develop allergic rhinitis or asthma later in childhood. Many outgrow AD as
respiratory allergy develops. S. aureus is found in over 90% of AD skin lesions. Decreased expression of
innate antimicrobial peptides, such as human beta-defensin and cathelicidins, such as LL 37, may explain
the increased susceptibility to colonization and skin infection with S. aureus in patients with atopic
dermatitis.
Q/Q(M)-480481 Report a Problem


Sarcoidosis presenting as uveitis, facial nerve palsy, fever and parotid gland swelling is referred to as:
1

Heerfordt's syndrome
2

Lofgren's syndrome
3

Lupus pernio
4

Darier-Roussy disease
5

Schaumann syndrome
Q/Q(M)-480468 Report a Problem


Sarcoidosis presenting as uveitis, facial nerve palsy, fever and parotid gland swelling is referred to as:
1

Heerfordt's syndrome
Heerfordt's syndrome is the name given to sarcoidosis presenting with uveitis, facial nerve palsy, fever
and parotid gland swelling. Lofgren's syndrome is an acute presentation of sarcoidosis that presents wth
fever, arthritis, erythema nodosum and hilar adenopathy. Darier-Roussy disease is sarcoidosis presenting
as painless firm subcutaneous nodules. Lupus pernio refers to sarcoidosis presenting as papulonodules
and plaques involving the nose (especially the alar rim), ears and cheeks. Schaumann syndrome was a
distractor as Shaumann bodies are seen in sarcoidosis on histopathology.
Q/Q(M)-480468 Report a Problem


Which of the following is important in the pathogenesis of acne vulgaris?
1

Activation of toll-like receptor-3 by P. acnes
2

Activation of toll-like receptor-2 by M. furfur
3

Activation of toll-like receptor-2 by P. acnes
4

P. acnes produces lipase which cleaves cholesterol into triglycerides
30

5

Demodex activates complement
Q/Q(M)-480419 Report a Problem

Which of the following is important in the pathogenesis of acne vulgaris?
3

Activation of toll-like receptor-2 by P. acnes
Acne vulgaris is a disease of follicular hyperkeratosis and the microcomedone is thought to be the
precursor lesion. P. acnes has lipase that cleaves triglycerides into free fatty acids. P. acnes can activate
complement and PMN chemotaxis. Toll-like receptors (TLR) recognize bacterial patterns and P. acnes
activates TLR-2.
Q/Q(M)-480419 Report a Problem


A child has a cutaneous ossification disorder characterized by endochondral bone formation, deafness,
baldness, and mental retardation. Which form of osteoma cutis does this patient have?
1

Fibrodysplasia ossificans progressiva
2

Progressive osseous heteroplasia
3

Plate-like osteoma cutis
4

Albright's hereditary osteodystrophy
5

Calcinosis cutis
Q/Q(M)-482391 Report a Problem



A child has a cutaneous ossification disorder characterized by endochondral bone formation, deafness,
baldness, and mental retardation. Which form of osteoma cutis does this patient have?
1

Fibrodysplasia ossificans progressiva
This patient has fibrodysplasia ossificans progressiva characterized by endochondral bone formation (the
other types have intramembranous bone formation), noggin gene defects and other systemic symptoms as
listed. This is a progressive and potentially fatal condition. Progressive osseous heteroplasia is also
progressive, seen in mostly females, and demonstrates increased alkaline phosphatase, LDH and CPK
with normal calcium, phosphate, and PTH. Plate-like osteoma cutis is limited and seen in children and
newborns. Albright's hereditary osteodystrophy is due to a mutation in GNAS-1. This disorder is
characterized by a lack of responsiveness to parathyroid hormone, resulting in low serum calcium, high
serum phosphate, and appropriately high serum parathyroid hormone. Individuals with Albright's
hereditary osteodystrophy have short stature, characteristically shortened fourth and fifth metacarpals,
rounded facies, and often mild mental retardation. Calcinosis cutis is not a form of osteoma cutis.
Q/Q(M)-482391 Report a Problem


The mechanism of action of azithromycin is:
1

Inhibition of bacterial cell wall synthesis
2

Inhibition of RNA-dependent protein synthesis by binding to the 30s ribosomal subunit
3

Inhibition of RNA-dependent protein synthesis by binding to the 50s ribosomal subunit
31

4

Inhibition of DNA-dependent RNA polymerase
5

Inhibition of bacterial topoisomerase
Q/Q(M)-474301 Report a Problem

The mechanism of action of azithromycin is:
3

Inhibition of RNA-dependent protein synthesis by binding to the 50s ribosomal subunit
Azithromycin is a macrolide antibiotic. It binds the bacterial 50s ribosomal subunit and inhibits RNA-
dependent protein synthesis.
Q/Q(M)-474301 Report a Problem


A pregnant woman in her third trimester presents with nonpruritic erythematous plaques and pustules in
the intertriginous regions, the trunk, and the extremities as well as systemic symptoms of malaise and
fever. Which complication is most associated with this condition?
1

Premature labor
2

Placental insufficiency
3

No risk to fetus or mother
4

Small for gestational age
5

Microcephaly
Q/Q(M)-482246 Report a Problem



A pregnant woman in her third trimester presents with nonpruritic erythematous plaques and pustules in
the intertriginous regions, the trunk, and the extremities as well as systemic symptoms of malaise and
fever. Which complication is most associated with this condition?
2

Placental insufficiency
This patient has pustular psoriasis of pregnancy which is also called "impetigo herpetiformis." It is
characterized by red plaques with a peripheral ring of pustules that are distributed symmetrically in
flexural areas and trunk and extremities. Patients can have elevated ESR and leukocytosis as well as
hypocalcemia. There is a risk of fetal morbidity and mortality secondary to placental insufficiency and
maternal mortality secondary to cardiac or renal failure. Premature labor may be associated with
cholestasis or pregnancy. Patients with herpes gestationis may have a risk of premature labor and small
for gestational age. Microcephaly may occur with isotretinoin taken during pregnancy.
Q/Q(M)-482246 Report a Problem

Acrokeratosis paraneoplastica is most commonly associated with which of the following malignancies:
1

Squamous cell carcinoma
2

Adenocarcinoma
3

Transitional cell carcinoma
4

T-cell lymphoma
32

5

Breast cancer
Q/Q(M)-480556 Report a Problem


Acrokeratosis paraneoplastica is most commonly associated with which of the following malignancies:
1

Squamous cell carcinoma
Acrokeratosis paraneoplastica, also known as Bazexs syndrome, is a rare paraneoplastic syndrome.
Clinically, it appears as symmetric, hyperkeratotic lesions on red base, in an acral distribution, nose and
helices of the ears. Most commonly associated with upper aerodigestive tract tumors. Treatment of the
tumor leads to disappearance of lesions.
Q/Q(M)-480556 Report a Problem


Which of the following is an example of a large vessel vasculitis?
1

Takayasu arteritis
2

Wegener's granulomatosis
3

Chrug-Strauss syndrome
4

Polyarteritis nodosa
5

Henoch-Schonlein purpura
Q/Q(M)-480469 Report a Problem

Which of the following is an example of a large vessel vasculitis?
1

Takayasu arteritis
Takayasu arteritis is a large vessel vasculitis that manifests as progressive granulomatous inflammation
of the aorta and its major branches. The systemic vasculitidies are classified into three categories: large
vessel, medium-sized vessel and small vessel vasculitis. Takayasu arteritis and Giant cell (temporal)
arteritis are the two large vessel vasculitidies. Polyarteritis nodosa and Kawasaki disease are medium-
sized vasculitidies. Wegener's granulomatosis, Churg-Strauss syndrome, microscopic polyangiitis,
Henoch-Schonlein purpura and cutaneous leukocytoclastic vasculitis are examples of small vessel
vasculitidies.
Q/Q(M)-480469 Report a Problem

What mast cell mediators are preformed, rather than synthesized upon exposure to a trigger?
1

tryptase
2

leukotriene C4
3

thromboxane
4

prostaglandin D2
5

platelet activating factor
Q/Q(M)-482861 Report a Problem


33

What mast cell mediators are preformed, rather than synthesized upon exposure to a trigger?
1

tryptase
Mast cell mediators can be grouped into two classes: preformed and newly formed. Preformed mediators
include tryptase, histamine, serotonin, and heparin. Newly formed mediators include prostaglandin D2,
leukotriene C4, and platelet activating factor.
Q/Q(M)-482861 Report a Problem


What is the treatment of choice for this condition:
1

Topical antifungal medication
2

High potency topical steroid
3

Topical antibiotic cream
4

Oral antibiotic
5

Excision
Q/Q(M)-476812 Report a Problem


What is the treatment of choice for this condition:
2

High potency topical steroid
The condition depicted is lichen sclerosus et atrophicus which most commonly affects the genital region.
High potency topical steroids are the treatment of choice.
Q/Q(M)-476812 Report a Problem



RNP antibodies include:
1

anti-SSA, anti-centromere, anti-SSB
2

anti-SSA, anti-SSB, anti-Sm, anti-U1RNP
3

anti-SSA, anti-SSB, anti-Sm, anti-dsDNA
4

ANA, anti-dsDNA
5

anti-SSA, anti-U1RNP, anti-centromere
Q/Q(M)-480521 Report a Problem


34

RNP antibodies include:
2

anti-SSA, anti-SSB, anti-Sm, anti-U1RNP
RNP antibodies target small ribonucleoproteins. These include SS-A (Ro), SS-B (La), Sm, and U1RNP.
The total amount of antibody has more diagnostic value than the mere presence of antibody.
Q/Q(M)-480521 Report a Problem

Which systemic anesthetic can be safely used in mastocytosis?
1

Lidocaine
2

Succinylcholine
3

D-tubocurarine
4

Thiopental
5

Propofol
Q/Q(M)-482221 Report a Problem

Which systemic anesthetic can be safely used in mastocytosis?
5

Propofol
Local lidocaine can be safely used but systemic lidocaine can precipitate anaphylaxis in patients with
mastocytosis. Other systemic analgesics that are counter-indicated in mastocytosis include
succinylcholine, d-tubocurarine, thiopental, metocurine, enflurane, isoflurane, and etomidate. Propofol,
vecuronium bromide, and fentanly are safe options for systemic anesthesia in patients with mastocytosis.
Q/Q(M)-482221 Report a Problem



A 48 year old woman was recently diagnosed with dermatomyositis. Which examination would be most
important in a work up for malignancy?
1

Breast exam
2

Chest xray
3

Thyroid exam
4

Pelvic exam
5

Stool guiac
Q/Q(M)-477130 Report a Problem

A 48 year old woman was recently diagnosed with dermatomyositis. Which examination would be most
important in a work up for malignancy?
4

Pelvic exam
Dermatomyositis is an idiopathic inflammatory disease with myositis and characteristic cutaneous
manifestations. There is an increased incidence of malignancy in these patients which may precede, occur
with or follow the diagnosis of dermatomyositis. The most common form of malignancy in adult women
35

is ovarian cancer. Other malignancies that have been associated include testicular cancer, gastrointestinal,
lung and nasopharyngeal carcinomas.
Q/Q(M)-477130 Report a Problem



A 17 year old high school cheerleader returns to clinic for follow up of her acne and management of her
isotretinoin. She denies depression or any GI complications. She is pleased with her progress but does
mention painful red bumps on her lower legs for 1 week. She attributes these to her cheer practice. What
is the most likely cause of her lower extremity lesions.
1

Training regimen
2

Abusive boyfriend
3

Oral contraceptives
4

Allergy medication
5

Recent strep infection
Q/Q(M)-482789 Report a Problem


A 17 year old high school cheerleader returns to clinic for follow up of her acne and management of her
isotretinoin. She denies depression or any GI complications. She is pleased with her progress but does
mention painful red bumps on her lower legs for 1 week. She attributes these to her cheer practice. What
is the most likely cause of her lower extremity lesions.
3

Oral contraceptives
The patient is likely experiencing erythema nodosum (EN), a delayed hypersensitivity response to a wide
variety of eliciting factors. They consist of an eruption of erythematous, tender nodules, typically over
the anterior tibial areas. Common causes include oral contraceptives, which is a probable choice given
that the patient is likely taking them as part of her isotretinoin treatment. Strep infection is also a common
cause of EN but with no mention of symptoms is less likely in this case. Other causes include TB, fungal
infections, sarcoidosis, ulcerative colitis, and regional enteritis.
Q/Q(M)-482789 Report a Problem

A 48 year-old man with a long history of alcohol abuse and dependence presents with angular cheilitis,
atrophic glossitis, a scrotal dermatitis that spares the midline and extends to the thighs, photophobia, and
blepharitis. Which nutritional deficiency do you suspect?
1

Vitamin B12
2

Vitamin C
3

Pyridoxine
4

Vitamin B2
5

Iron
Q/Q(M)-480554 Report a Problem


36

A 48 year-old man with a long history of alcohol abuse and dependence presents with angular cheilitis,
atrophic glossitis, a scrotal dermatitis that spares the midline and extends to the thighs, photophobia, and
blepharitis. Which nutritional deficiency do you suspect?
4

Vitamin B2
The patient presents with classic findings of the oral-ocular-genital syndrome seen in the setting of
vitamin B2 (riboflavin) deficiency. This deficiency occurs most often in alcoholics. It characteristically
presents with angular cheilitis, atrophic glossitis (magenta), a seborrheic-like dermatitis around the nose,
genital dermatitis (scrotal dermatitis sparing the midline and extending to the thighs), photophobia, and
blepharitis.
Q/Q(M)-480554 Report a Problem


Which of the following medications is NOT associated with exacerbating or causing this eruption?
1

Lithium
2

Prednisone
3

Phenytoin
4

Isoniazid
5

Propranolol
Q/Q(M)-476826 Report a Problem


Which of the following medications is NOT associated with exacerbating or causing this eruption?
5

Propranolol
Acne and acneiform eruptions can be caused or exacerbated by cortiosteroids, oral contraceptives,
androgens, ACTH, lithium, phenytoin, halogens, INH, and haloperidol.
Q/Q(M)-476826 Report a Problem



Which of the following is true regarding this entity?
1

The subcutaneous variant is most common in children
2

75% of localized lesions are still present 2 years after diagnosis
3

Localized lesions often ulcerate and heal with scarring
4

The perforating variant classically involves the lower extremities
5

Diabetes is present in the majority of patients with the generalized variant
37

Q/Q(M)-480492 Report a Problem



Which of the following is true regarding this entity?
1

The subcutaneous variant is most common in children
Granuloma annulare is the entity pictured. Subtypes include localized GA, generalized GA, macular GA,
subcutaneous GA, and perforating GA. The subcutaneous variant is most common in children; there is
often a history of trauma to the area. Localized lesions never ulcerate and heal without scarring; 75% of
localized lesions clear within 2 years. The perforating variant classically involves the dorsa of the hands.
Diabetes is present in 20% of patients with the generalized variant.
Q/Q(M)-480492 Report a Problem



A 35 year-old woman presents with predominantly lower facial inflammatory acne, hirsutism, and
irregular menses. She has failed multiple conventional treatments. Laboratory work-up reveals a very
high level of DHEA-S. You are most concerned about:
1

Congenital adrenal hyperplasia
2

Adrenal tumor
3

Polycystic ovary syndrome
4

Toxicity from prolonged use of spironolactone
5

Progestin excess from oral contraceptives
Q/Q(M)-480381 Report a Problem



A 35 year-old woman presents with predominantly lower facial inflammatory acne, hirsutism, and
irregular menses. She has failed multiple conventional treatments. Laboratory work-up reveals a very
high level of DHEA-S. You are most concerned about:
2

Adrenal tumor
A patient whose acne fails to respond to conventional therapy, whose acne flares cyclically, with
hirsutism, alopecia, or irregular menses warrants an endocrine work-up, including free and total
testosterone, LH, FSH, and DHEA-S. Although rare, very high levels of DHEA-S may suggest an
adrenal adrogen-secreting tumor.
Q/Q(M)-480381 Report a Problem
38



The dermatosis pictured is most commonly associated with:
1

Insulin resistance
2

Cushing's syndrome
3

Acromegaly
4

Polycystic ovarian syndrome
5

Gastric carcinoma
Q/Q(M)-480479 Report a Problem


The dermatosis pictured is most commonly associated with:
1

Insulin resistance
Insulin resistance is the most common cause of acanthosis nigricans. Insulin-like growth factors,
produced by the liver in response to high levels of circulating insulin, bind epidermal growth factor
receptors to produce thickening of the epidermis and hyperkeratosis. Cushing's syndrome, acromegaly
and polycystic ovarian syndrome are associated with acanthosis nigricans, although less common than
insulin resistance. Paraneoplastic acanthosis nigricans is associated with gastrointestinal carcinoma,
classically gastric carcinoma.
Q/Q(M)-480479 Report a Problem


Eyebrow growth:
1

Is androgen-dependent in men but not in women
2

Is regulated by dehydroepiandrosterone but not androstenedione
3

Is not androgen-dependent
4

Requires conversion of testosterone to dihydrotestosterone in the hair follicle
5

Is regulated only by testosterone and dihydrotestosterone
Q/Q(M)-480498 Report a Problem


Eyebrow growth:
39

3

Is not androgen-dependent
Eyebrows, eyelashes, and vellus hair are not androgen-dependent, thus there is no difference between
these areas of hair growth in men and women.
Q/Q(M)-480498 Report a Problem


Leukokeratosis nicotina palati:
1

Is not attributed to smoking
2

Represents a premalignant condition
3

Responds to intralesional steroids
4

Represents inflamed minor salivary glands
5

Resolves with tetracycline therapy
Q/Q(M)-477172 Report a Problem

Leukokeratosis nicotina palati:
4

Represents inflamed minor salivary glands
Leukokeratosis nicotina palati, also called smokers palate, is a complication of tobacco use, especially
pipe smokers. Lesions develop as a result of inflamed minor salivary glands and manifest as uniform
keratosis of the hard palate with multiple red, umbilicated papules.
Q/Q(M)-477172 Report a Problem


Which of the following statements comparing inflammatory lesions from patients with psoriasis and
patients with atopic dermatitis is true?
1

Human -defensin 2 (HBD-2) and LL 37 levels in psoriasis lesions are significantly lower than in
atopic dermatitis lesions
2

Expression of HBD-2 and LL 37 is negligible in psoriatic lesions
3

HBD-2 and LL 37 are significantly elevated in atopic dermatitis lesions
4

Expression of HBD-2 and LL 37 in psoriasis is similar to normal skin, whereas it is decreased in
atopic dermatitis lesions
5

HBD-2 and LL 37 are significantly decreased in atopic dermatitis lesions compared to psoriasis
lesions
Q/Q(M)-474308 Report a Problem


Which of the following statements comparing inflammatory lesions from patients with psoriasis and
patients with atopic dermatitis is true?
5

HBD-2 and LL 37 are significantly decreased in atopic dermatitis lesions compared to psoriasis
lesions
Ong et al found a deficiency of HBD-2 and LL 37, which are innate antimicrobial peptides, in lesions
from patients with atopic dermatitis compared to lesions from patients with psoriasis. IL-4 and IL-13,
which are Th2-type cytokines, were found to inhibit expression of HBD-2 and LL 37. Decreased
expression of HBD-2 and LL 37 may account for the increased susceptibility to S. aureus infection in
40

patients with atopic dermatitis.
Q/Q(M)-474308 Report a Problem

Pernio or Chilblains is an inflammatory skin condition which is triggered by cold, wet, non-freezing
environmental conditions. Acral skin has violaceous discoloration accompanied by burning or itching.
While avoidance and prevention is best, the most effective pharmacologic treatment is:
1

Nifedipine
2

Nicotinamide
3

Phenoxybenzamine
4

Psoralen+UVA
5

Aspirin
Q/Q(M)-477646 Report a Problem


Pernio or Chilblains is an inflammatory skin condition which is triggered by cold, wet, non-freezing
environmental conditions. Acral skin has violaceous discoloration accompanied by burning or itching.
While avoidance and prevention is best, the most effective pharmacologic treatment is:
1

Nifedipine
Nifedipine is effective in about 70% of patients with pernio in prevention of the development of new skin
lesions. The other options listed, other than aspirin, are anecdotally suggested to be helpful.
Q/Q(M)-477646 Report a Problem

A patient develops pulmonary fibrosis, Raynaud's phenomenon and skin changes similar to systemic
sclerosis. These changes resolve following discontinuation of which of the following medications?
1

Bleomycin
2

Penicillamine
3

Hydralazine
4

Minocycline
5

Isoniazide
Q/Q(M)-480066 Report a Problem


A patient develops pulmonary fibrosis, Raynaud's phenomenon and skin changes similar to systemic
sclerosis. These changes resolve following discontinuation of which of the following medications?
1

Bleomycin
The changes described above are similar to findings of scleroderma. Patients receiving bleomycin can
develop a reversible syndrome similar to scleroderma. The other four listed options are related to drug-
induced lupus erythematosus and are not linked to scleroderma-like syndromes.
Q/Q(M)-480066 Report a Problem


41

What is the most likely diagnosis?
1

Voight line
2

Linea nigra
3

Fuchter line
4

Phytophotodermatitis
5

Incontinentia pigmenti
Q/Q(M)-476887 Report a Problem


What is the most likely diagnosis?
2

Linea nigra
Linea nigra is a hyperpigmented, linear patch that often becomes apparent during pregnancy. It is thought
to be secondary to hormonal fluctuations and usually spontaneously resolves after pregnancy. Voight and
Fuchter lines delineate a vestigial hyperpigmentation of the dorsal aspect of organisms.
Q/Q(M)-476887 Report a Problem


Which of the following viruses is associated the most with lichen planus?
1

Hepatitis C virus
2

Hepatitis B virus
3

Human immunodeficiency virus
4

Hepatitis A virus
5

Epstein-barr virus
Q/Q(M)-482811 Report a Problem


Which of the following viruses is associated the most with lichen planus?
1

Hepatitis C virus
Lichen planus classically involves the wrists, ankles, and oral mucosa. It is characterized by violaceous
polygonal, purple, pruritic papules. It has a strong association with hepatitis C virus. Many other
associations exist, including other viruses and medications, but hepatitis C is the best choice.
42

Q/Q(M)-482811 Report a Problem


Which of the following is NOT true regarding primary cutaneous amyloidosis?
1

AA is not the protein component
2

Amyloid is present around blood vessels
3

AL is not the protein component
4

Notalgia paresthetica may be associated with the macular form
5

Amyloid found in benign and malignant neoplasms does not represent primary cutaneous
amyloidosis
Q/Q(M)-480505 Report a Problem


Which of the following is NOT true regarding primary cutaneous amyloidosis?
1

AA is not the protein component
2

Amyloid is present around blood vessels
3

AL is not the protein component
4

Notalgia paresthetica may be associated with the macular form
5

Amyloid found in benign and malignant neoplasms does not represent primary cutaneous
amyloidosis
Q/Q(M)-480505 Report a Problem



Which of the following is NOT true regarding primary cutaneous amyloidosis?
2

Amyloid is present around blood vessels
Keratin is the protein component in primary cutaneous amyloidosis. No amyloid is found around blood
vessels. Macular amyloidosis (which may have associated notalgia paresthetica) and lichen amyloidosis
are forms of primary cutaneous amyloidosis. Secondary cutaneous amyloidosis presents with keratin-
derived amyloid and is seen following PUVA therapy and in benign and malignant neoplasms.
Q/Q(M)-480505 Report a Problem



The best diagnosis for this congenital melanopenic lesion without extracutaneous associations is
1

Hypomelanosis of Ito
2

Segmental vitiligo
3

Ash leaf spots
4

Nevus anemicus
5

Nevus achromicus
Q/Q(M)-482125 Report a Problem
43




The best diagnosis for this congenital melanopenic lesion without extracutaneous associations is
5

Nevus achromicus
Nevus achromicus is another name for nevus depigmentosus. It usually presents at birth or appear during
early infancy as normal pigmentation increases. Most individuals will have a solitary lesion of nevus
depigmentosus, but multiple lesions and segmental forms of nevus depigmentosus have been described.
Nevus depigmentosus tends to persist lifelong, but remains unchanged after onset. The hypopigmented
white spots of tuberous sclerosis are most difficult to distinguish from nevus depigmentosus, but lack of
other cutaneous or systemic manifestaions exclude tubrous sclerosis. Lesions of vitiligo tend to be
depigmented (melanocytopenic not melanopenic as in the question), and show a bright white coloration
with Wood's lamp examination. Nevus anemicus is a distinct vascular birthmark characterized by
blanching of cutaneous blood vessels, hence presenting as a "white" patch of skin that becomes
unnoticeable when the surrounding skin is blanched with a glass slide ("diascopy").
Q/Q(M)-482125 Report a Problem


Scalp biopsy of a 44-year-old female with suspected discoid lupus erythematosis would likely reveal
inflammation around which portion of the hair follicle:
1

Dermal papilla
2

Hair bulb
3

Matrix
4

Isthmus
5

Infundibulum
Q/Q(M)-482937 Report a Problem


Scalp biopsy of a 44-year-old female with suspected discoid lupus erythematosis would likely reveal
inflammation around which portion of the hair follicle:
4

Isthmus
On biopsy, alopecia areata exhibits a peribulbar lymphocytic swarm of bees. Discoid lupus typically
exhibits inflammation surrounding the isthmus, along with a perivascular dermatitis and vacuolar
interface changes. Lichen planopilaris exhibits inflammation most densely concentrated about the
infundibulum.
Q/Q(M)-482937 Report a Problem


44

A 22 year-old woman notes irregular menses and acne that has been refractory to several over-the-
counter and prescription regimens. Appropriate initial work-up includes
1

Serum prolactin levels
2

24-hour urine cortisol
3

Serum free and total testosterone, LH, FSH, androstenedione
4

Serum free and total testosterone, LH, FSH, DHEA-S
5

Serum free and total testosterone, LH, FSH, 17-OH, hydroxylase
Q/Q(M)-474890 Report a Problem

A 22 year-old woman notes irregular menses and acne that has been refractory to several over-the-
counter and prescription regimens. Appropriate initial work-up includes
4

Serum free and total testosterone, LH, FSH, DHEA-S
The history and clinical appearance of this patient suggest a possible hormonal basis to her acne. The
work-up in answer d assesses for adrenal hormonal production and screens for PCOS. Androstenedione is
of ovarian origin.
Q/Q(M)-474890 Report a Problem

This radiograph shows a characteristic finding of severe psoriatic arthritis: bone proliferation at the base
of the distal phalanx with resorption of the tufts. What is the most common form of psoriatic arthritis?
1

Arthritis mutilans
2

Axial
3

Symmetric polyarthritis
4

Asymmetric oligoarthritis
5

Symmetric oligoarthritis
Q/Q(M)-474895 Report a Problem


This radiograph shows a characteristic finding of severe psoriatic arthritis: bone proliferation at the base
of the distal phalanx with resorption of the tufts. What is the most common form of psoriatic arthritis?
4

Asymmetric oligoarthritis
45

Approximately 70 % of psoriatic arthritis is asymmetric oligoarthritis. The slide depicts arthritis mutilans,
which occurs in about 5% of patients with psoriatic arthritis.
Q/Q(M)-474895 Report a Problem


Which HLA type is more commonly associated with this clinical finding?
1

HLA-B7
2

HLA-B15
3

HLA-B27
4

HLA-B51
5

HLA-DR4
Q/Q(M)-476811 Report a Problem



Which HLA type is more commonly associated with this clinical finding?
3

HLA-B27
The condition shown is balnitis circinata which is part of the constellation of findings in Reiter's
syndrome in addition to arthritis, urethritis, and conjunctivitis. There is a higher incidence of this
condition people with HLA-B27.
Q/Q(M)-476811 Report a Problem


This patient was started on isotretinoin but failed to discontinue the tetracycline. He is at risk for what
complication?
1

Acne fulminans
2

Pseudotumor cerebri
3

Diarrhea
4

Depression
5

Myositis
Q/Q(M)-476889 Report a Problem
46



This patient was started on isotretinoin but failed to discontinue the tetracycline. He is at risk for what
complication?
2

Pseudotumor cerebri
Both isotretinoin and tetracycline are known to cause pseudotumor cerebri. In combination, the risk is
significantly elevated. Symptoms of pseudotumor cerebri include headaches, pulsatile tinnitus, diplopia,
and blurred vision.
Q/Q(M)-476889 Report a Problem


Bullous lupus erythematosus is most commonly associated with antibodies to:
1

Type IV collagen
2

Laminin 5
3

Desmoglein I
4

Plectin
5

Type VII collagen
Q/Q(M)-482145 Report a Problem



Bullous lupus erythematosus is most commonly associated with antibodies to:
5

Type VII collagen
Type VII collagen is found in the sublamina densa and patients with bullous lupus erythematosus have
been found to have antibodies to this protein. Bullous LE and EBA share antibasement membrane zone
antibodies of identical specificity (type VII collagen) as well as clinical and histologic overlap. A
differentiating feature between bullous LE and EBA is that bullous LE has a dramatic response to
dapsone.
Q/Q(M)-482145 Report a Problem


What is the most common infection that can cause guttate psoriasis?
1

Streptococcus pyogenes
2

Coccidiomycosis
3

Herpes
4

Mycoplasma pneumonia
47

5

Tuberculosis
Q/Q(M)-482069 Report a Problem



What is the most common infection that can cause guttate psoriasis?
1

Streptococcus pyogenes
Guttate psoriasis refers to a distinctive, acute clinical presentation of an eruption characterized by small,
droplike, 1-10 mm in diameter, salmon-pink papules, usually with a fine scale. It is more commonly seen
in individuals younger than 30 years with a history of upper respiratory infection that precedes the
eruption by 2-3 weeks. The most common organism is group A beta-hemolytic streptococci (eg,
Streptococcus pyogenes). Although recurrent episodes may occur, especially those due to pharyngeal
carriage of streptococci, isolated bouts are known to occur. Guttae psoriasis may also occur with other
infections or in isolation without any identifiable infection.
Q/Q(M)-482069 Report a Problem


Which class of medication works by inhibits DNA gyrase?
1

Quinolones
2

Tetracyclines
3

Aminoglycosides
4

Cephalosporins
5

Carbapenems
Q/Q(M)-482797 Report a Problem

Which class of medication works by inhibits DNA gyrase?
1

Quinolones
Penicillins, cephalosporins, carbapenems, and vancomycin work on the cell wall. Quinolones inhibit
DNA gyrase. Tetracyclines and aminoglycosides work at the 30 S ribosomal subunit.
Q/Q(M)-482797 Report a Problem

Which of the following disorders is most strongly associated with a monoclonal gammopathy of the IgG-
lambda type?
1

Scleredema
48

2

POEMS syndrome
3

Necrobiotic xanthogranuloma
4

EED
5

Scleromyxedema
Q/Q(M)-482477 Report a Problem


Which of the following disorders is most strongly associated with a monoclonal gammopathy of the IgG-
lambda type?
5

Scleromyxedema
All of the above conditions are assocaited with some type of monoclonal gammopathy. Scleromyxedema
is associated with an IgG-lambda monoclonal gammopathy. Scleredema and NXG are most often
associated with IgG-kappa, and both POEMS syndrome and EED are usually associated with IgA
monoclonal gammopathies.
Q/Q(M)-482477 Report a Problem

A pregnant woman presents with mild inflammatory acne which is very bothersome to her. Which of the
following is the most appropriate treatment option based on FDA classifications of medication in
pregnancy?
1

Topical erythromycin/benzoyl peroxide gel
2

Topical tretinoin
3

Tazarotene 0.1% cream
4

Azelaic acid 20% cream
5

Bactrim
Q/Q(M)-474341 Report a Problem


A pregnant woman presents with mild inflammatory acne which is very bothersome to her. Which of the
following is the most appropriate treatment option based on FDA classifications of medication in
pregnancy?
4

Azelaic acid 20% cream
Azeleic acid is category B in pregnancy. The others are category C (benzoyl peroxide, topical tretinoin,
and bactrim) or category X (tazarotene). The categories for safety of drugs in pregnancies are as follows:
Category A: controlled studies in humans show no risk to fetus. Category B: controlled human studies
show no risk to fetus but may show risk to animals, or no risk in animal studies but no human studies
conducted. Category C: risk to human fetus cannot be ruled out, studies are lacking; animal studies
equivocal. Category D: controlled suties show risk to human fetus, benefits may sometimes outweigh
risk. Cateogory X: contraindicated in pregnancy.
Q/Q(M)-474341 Report a Problem

An 8 year old healthy boy complaining of changes on his nails for the past 10 months. What is the most
likely diagnosis?
49

1

Psoriasis
2

Alopecia
3

Pachynonychia congenita
4

Chronic mucocutaneous candidiasis
5

Atopic dermatitis
Q/Q(M)-482880 Report a Problem



An 8 year old healthy boy complaining of changes on his nails for the past 10 months. What is the most
likely diagnosis?
1

Psoriasis
Nail involvement in psoriasis is common. It affects about 25-50% of pediatric patients with psoriasis,
more commonly during the second decade of life.
Q/Q(M)-482880 Report a Problem


Spindle cell lipoma is commonly found on the:
1

Head
2

Lower extremities
3

Buttocks
4

Breast
5

Posterior shoulder
Q/Q(M)-477681 Report a Problem

Spindle cell lipoma is commonly found on the:
5

Posterior shoulder
Spindle cell lipoma is a solitary benign tumor seen in adult male patients, and is most often located on the
posterior shoulder and neck regions.The tumor histologically consists of mature collagen, adpose tissue,
spindle cells, and mast cells. Treatment is with local excision.
Q/Q(M)-477681 Report a Problem

Which of the following forms of angioedema is mediated by kinin release?
1

Hereditary angioedema
2

Acute allergic angioedema
3

Angioedema associated with urticarial vasculitis
4

NSAID angioedema
5

Infectious angioedema
50

Q/Q(M)-482506 Report a Problem


Which of the following forms of angioedema is mediated by kinin release?
1

Hereditary angioedema
Types of angioedema mediated through kinin release include hereditary angioedema, acquired C1
inhibitor deficiency, and ACE inhibitor associated angioedema. These conditions are not associated with
concurrent urticaria.
Q/Q(M)-482506 Report a Problem


What is the most common site of extracutaneous involvement in this non-infectious disease?
1

Lungs
2

Eyes
3

Bone Marrow
4

Liver
5

Nasal mucosa
Q/Q(M)-476631 Report a Problem


What is the most common site of extracutaneous involvement in this non-infectious disease?
1

Lungs
Sarcoidosis is a non-infectious granulomatous disorder of unclear etiology. It has a predilection for the
lungs (90%), lymph nodes (75-90%), eyes (25%), nasal mucosa (20%), bone marrow (25-40%), and liver
(15-40%).
Q/Q(M)-476631 Report a Problem


A positive ANA with a speckled staining pattern correlates with staining of what cellular component?
1

Ribonucleoproteins
2

Nucleolar RNA
3

Native DNA
4

Kinetochore
5

Histones
Q/Q(M)-480464 Report a Problem
51



A positive ANA with a speckled staining pattern correlates with staining of what cellular component?
1

Ribonucleoproteins
A speckled ANA pattern correlates with staining of ribonucleoproteins and is seen in Mixed Connective
Tissue Disease (MCTD), Systemic Lupus Erythematosus (SLE), systemic sclerosis and Sjogren's
syndrome. A nucleolar staining pattern correlates with staining of nucleolar RNA. Centromere staining
correlates with staining of kinetochore. A peripheral staining pattern correlates with staining of native
DNA and a homogeneous staining pattern correlates with staining of native DNA and histones.
Q/Q(M)-480464 Report a Problem


A common site for chloracne is the:
1

Occipital scalp
2

Forehead
3

Scrotum
4

Forearms
5

Shoulders
Q/Q(M)-474330 Report a Problem

A common site for chloracne is the:
3

Scrotum
Common sites for chloracne include the malar cheek, the post-auricular scalp and in men, the scrotum.
Q/Q(M)-474330 Report a Problem




An 8 year old boy with seasonal allergies presents with red brown macules on the trunk present for
several months. Parent mentions that when these lesions are scratched they form an irritated, red wheal.
Which of the following sign is likely described?
1

Asboe-Hansen sign
2

Homans sign
3

Dariers sign
4

Auspitz sign
5

Hutchinson sign
Q/Q(M)-482790 Report a Problem


An 8 year old boy with seasonal allergies presents with red brown macules on the trunk present for
several months. Parent mentions that when these lesions are scratched they form an irritated, red wheal.
Which of the following sign is likely described?
52

3

Dariers sign
Dariers sign is seen in patients with urticarial pigmentosa, a common form of mastocytosis. Erythema
and wheals are commonly elicited with stroking or rubbing secondary to mast cell degranulation.
Symptoms can range from very mild (flushing, hives, no treatment needed) to life-threatening (vascular
collapse). Asboe Hansen, commonly seen in pemphigus vulgaris, refers to extension of a bulla to
adjacent unblistered skin when pressure is put on top of a bulla.
Q/Q(M)-482790 Report a Problem


The diagnosis is:
1

Onychomycosis
2

Trauma-induced nail changes
3

Chronic paronychia
4

Pseudomonal infection
5

Psoriasis arthritis
Q/Q(M)-474891 Report a Problem


The diagnosis is:
5

Psoriasis arthritis
This slide shows lateral onycholysis and subtle swelling of the distal interphalangeal joint (DIP), both
characteristic findings of psoriasis and psoriatic arthritis.
Q/Q(M)-474891 Report a Problem



You suspect that a patient has acquired angioedema. Levels of which of the following would you expect
to be low?
1

C1q, C3
2

C1q, bradykinin
3

C4, C1q
4

bradykinin, C3
53

5

C4, C3
Q/Q(M)-482452 Report a Problem


You suspect that a patient has acquired angioedema. Levels of which of the following would you expect
to be low?
3

C4, C1q
Initial screening test in angioedema for both inherited and acquired should include C4, which is low. C3
is normal in angioedema. C1q is low in acquired angioedema but normal in the hereditary type.
Bradykinin is elevated in both inherited and acquired angioedema.
Q/Q(M)-482452 Report a Problem


A 40 year-old man presents with a complaint of nail changes for several years. He was treated by an
outside physician with terbinafine without improvement. On further questioning, he reports morning
shoulder stiffness and back pain that lasts 1-2 hours and improves with activity. Which of the following
is true regarding his condition?
1

50% of patients present with joint disease prior to skin involvement
2

Bony erosions are not commonly seen on radiographs
3

Cyclosporine is effective in treating the arthritis in this condition
4

A positive rheumatoid factor may be seen
5

Joint disease correlates with severity of skin involvement
Q/Q(M)-480388 Report a Problem



A 40 year-old man presents with a complaint of nail changes for several years. He was treated by an
outside physician with terbinafine without improvement. On further questioning, he reports morning
shoulder stiffness and back pain that lasts 1-2 hours and improves with activity. Which of the following
is true regarding his condition?
4

A positive rheumatoid factor may be seen
This patient has psoriatic nail changes and a history suggestive of psoriatic arthritis. Psoriatic arthritis is
an inflammatory arthropathy associated with psoriasis. In 80% of patients the rheumatoid factor is
negative; however a positive rheumatoid factor may sometimes be seen. 80% of patients present with
54

skin disease first. Large eccentric erosions are classically present on radiographs. Cyclosporine is not
effective in treating psoriatic arthritis. Mild skin disease may be associated with moderate-to-severe joint
disease, and vice versa.
Q/Q(M)-480388 Report a Problem



Which form of sarcoidosis is associated with camptodactyly?
1

Lofgren's syndrome
2

Heerfordt's syndrome
3

Darier-Roussy
4

Mikulicz syndrome
5

Blau syndrome
Q/Q(M)-477494 Report a Problem

Which form of sarcoidosis is associated with camptodactyly?
5

Blau syndrome
Camptodactyly is a flexion contracture of the 3rd through the 5th proximal interphalangeal joints and
elbows bilaterally. Blau syndrome is an autosomal dominant form of sarcodosis due to a defect in the
CARD15 gene manifesting also with arthritis, cutaneous sarcoid, uveitis, and synovial cysts. It does not
have lung or visceral involvement.
Q/Q(M)-477494 Report a Problem

A patient complained of sudden appearance of multiple papules on chest and upper arms. Preliminary
report by the dermpath fellow are cystic spaces lines by two layers of cuboidal cells and epithelial strands
of similar cells. What is the most likely diagnosis?
1

Syringomas
2

Hydrocystomas
3

Acrospiromas
4

Spiradenomas
5

Mixed tumors
Q/Q(M)-482885 Report a Problem



A patient complained of sudden appearance of multiple papules on chest and upper arms. Preliminary
report by the dermpath fellow are cystic spaces lines by two layers of cuboidal cells and epithelial strands
of similar cells. What is the most likely diagnosis?
1

Syringomas
The eruptive type of syringomas. In addition to the description above, some of the cysts have small
comma-like tails, which is known as a tadpole pattern. Eruptive syringomas are histologically
55

identical to those on the eyelid, but appear suddenly on the neck, chest, axillae, upper arms, and
periumbilically. It usually occurs in young people. The other options do not have an eruptive form and
tend to occur in different locations of the body other than the arms and thorax.
Q/Q(M)-482885 Report a Problem

What is the most common paraproteinemia in scleromyxedema
1

IgA
2

IgG kappa
3

IgG lambda
4

IgM
5

It is uncommon to see paraproteinemia with scleromyxedema
Q/Q(M)-482148 Report a Problem


What is the most common paraproteinemia in scleromyxedema
3

IgG lambda
An abnormal paraproteinemia is found in approximately 90% of cases with scleromyxedema, generally
IgG lambda. IgA paraproteinemia can be seen in pyoderma gangrenosum and Sneddon-Wilkinson
disease. Amyloidosis and NXG can be associated with IgG kappa paraproteinemia, and Schnitzler
syndrome is associated with an IgM paraproteinemia.
Q/Q(M)-482148 Report a Problem

What mast cell mediators are synthesized upon exposure to a trigger, rather than preformed?
1

tryptase
2

histamine
3

serotonin
4

prostaglandin D2
5

heparin
Q/Q(M)-482849 Report a Problem

What mast cell mediators are synthesized upon exposure to a trigger, rather than preformed?
4

prostaglandin D2
Mast cell mediators can be grouped into two classes: preformed and newly formed. Preformed mediators
include tryptase, histamine, serotonin, and heparin. Newly formed mediators include prostaglandin D2,
leukotriene C4, and platelet activating factor.
Q/Q(M)-482849 Report a Problem


Which of the following is not a common food or environmental allergen implicated in atopic dermatitis?
56

1

Dermatophagoides pteronyssimus
2

Wheat
3

Corn
4

Eggs
5

Fish
Q/Q(M)-480483 Report a Problem

Which of the following is not a common food or environmental allergen implicated in atopic dermatitis?
3

Corn
While most AD patients do not have food allergy, food allergens exacerbate AD in at least a subset of
patients, particularly infants and young children. Eggs, milk, peanuts, soybeans, tree nuts, fish, and wheat
are the most common food allergens implicated. Dust mites (Dermatophagoides pteronyssimus) are
among the environmental allergens that may exacerbate AD.
Q/Q(M)-480483 Report a Problem
Necrobiotic xanthogranuloma is most commonly associated with which of the following:
1

IgA monoclonal gammopathy
2

IgM monoclonal gammopathy
3

IgG lambda monoclonal gammopathy
4

IgG kappa monoclonal gammopathy
5

IgG/IgA polyclonal gammopathy
Q/Q(M)-482684 Report a Problem



Necrobiotic xanthogranuloma is most commonly associated with which of the following:
4

IgG kappa monoclonal gammopathy
Necrobiotic xanthogranuloma is associated with a monoclonal IgG kappa paraproteinemia in
approximately 80% of cases. It is less commonly associated with an IgA gammopathy. Myeloma or
myelodysplastic syndromes may resultantly occur.
Q/Q(M)-482684 Report a Problem
The main cause of nutritional disease in developed nations is:
1

Unusual diets
2

Inflammatory bowel disease
3

Malabsorption syndromes
4

Alcoholism
5

Psychiatric illness
57

Q/Q(M)-480552 Report a Problem


The main cause of nutritional disease in developed nations is:
4

Alcoholism
Alcoholism is the main cause of nutritional disease in developed nations. Other conditions that cause
nutritional disease include: unusual diets, postoperative state, psychiatric illness, inflammatory bowel
disease, cystic fibrosis, surgical bowel dysfunction, and inborn errors of metabolism.
Q/Q(M)-480552 Report a Problem
A 45-year-old woman develops bilateral poikilodermatous patches without atrophy on her lateral thighs.
This is most likely a manifestation of what disease?
1

Lupus Erythematosus
2

Dermatomyositis
3

Mycosis Fungoides
4

Hepatitis C
5

Diabetes Mellitus
Q/Q(M)-482218 Report a Problem


A 45-year-old woman develops bilateral poikilodermatous patches without atrophy on her lateral thighs.
This is most likely a manifestation of what disease?
2

Dermatomyositis
Poikilodermatous changes on the lateral thigh represent the "holster sign" seen in dermatomyositis.
The poikilodermatous patches seen in dermatomyositis are often symmetric macular violaceous
erythemas with dyspigmentation. Other characteristic poikilodermatous manifestations of
dermatomyositis include the "V-sign" on the anterior neck and upper chest and the "shawl sign" on
the posterior neck, upper back, and the posterior shoulders. Lesions of Mycosis Fungoides typically have
some component of epidermal atrophy.
Q/Q(M)-482218 Report a Problem

Which of the following is characteristic of lichen planus pigmentosus?
1

Lesions are typically hypopigmented
2

Most cases present in Caucasians
3

Oral involvement is pathognomonic
4

Can occasionally see epidermotropic T-cells
5

Trunk is typically spared
Q/Q(M)-482521 Report a Problem


Which of the following is characteristic of lichen planus pigmentosus?
4

Can occasionally see epidermotropic T-cells
58

Occasionally epidermotropic T-cells are seen in the lichenoid reaction and thus may raise concern for
mycosis fungoides. Lichen planus pigmentosus inversus, in particular, presents in classic sites of mycosis
fungoides, including the axilla, inguinal, and inframammary areas. The individual lesions of lichen
planus pigmentosis are typically smaller, however, than those encountered in mycosis fungoides, thus
helping with the differential. Lesions of lichen planus pigmentosus are hyperpigmented, with most cases
prsenting in skin of color. Oral involvement is rare and the trunk can be involved.
Q/Q(M)-482521 Report a Problem


A concerned 23 year old male presents to clinic with multiple verrucoid lesions on his penis. He is
anxious to have them removed. You explain that these are commonly treated cryosurgically. What is the
boiling point of liquid nitrogen in Celsius?
1

-89.5
2

-40.8
3

3.8
4

-195.8
5

-78.5
Q/Q(M)-482791 Report a Problem



A concerned 23 year old male presents to clinic with multiple verrucoid lesions on his penis. He is
anxious to have them removed. You explain that these are commonly treated cryosurgically. What is the
boiling point of liquid nitrogen in Celsius?
4

-195.8
The boiling point of liquid nitrogen is -195.8. Nitrous oxide is -89.5. Carbon dioxide is -78.5.
Dichlorotrifluoroethane (Freon 114) is 3.8. Chlorodifluoromethane (Freon 22) is 40.8. Colder cryogen is
needed for adequate deep destruction, making liquid nitrogen the only reliable agent of the five listed.
The Freons, solid CO2 and nitrous oxide are used only for topical anesthesia and superficial destruction.
Q/Q(M)-482791 Report a Problem


Sneddon-Wilkinson Disease:
1

Most often occurs in elderly women
2

Rarely involves intertriginous areas
3

May occur in association with an IgG monoclonal gammopathy
4

Can be treated with narrow band UVB
5

Is usually an acute, self-limited condition
Q/Q(M)-480395 Report a Problem

Sneddon-Wilkinson Disease:
4

Can be treated with narrow band UVB
59

Sneddon-Wilkinson disease, or subcorneal pustular dermatosis, presents with superficial pustules in
annular and serpiginous patterns in the axillae, groin, and abdomen. Middle-aged women are most often
affected. This condition rarely occurs in association with an IgA monoclonal gammopathy. It is a chronic
condition, possibly related to psoriasis, with remissions of variable duration. Treatments including
dapsone, acitretin, and narrow band UVB.
Q/Q(M)-480395 Report a Problem

Which of the following HLA types is associated with early onset psoriasis?
1

HLA-B13 or HLA-B17
2

HLA-B27
3

HLA-B57, HLA-Cw6, or HLA-DR7
4

HLA-Cw2
5

HLA-DR3
Q/Q(M)-482152 Report a Problem

Which of the following HLA types is associated with early onset psoriasis?
3

HLA-B57, HLA-Cw6, or HLA-DR7
HLA-B57, HLA-Cw6, or HLA-DR7 are most commonly associated with early onset type I psoriasis. The
presence of HLA-B13 or B17 is associated with a 5-fold risk of developing psoriasis and are increase in
guttate and erythrodermic psoriasis. HLA-B27 may be seen in pustular psoriasis. HLA-Cw2 is seen with
late onset psoriasis, or type II. HLA-DR3 is commonly found with subacute cutaneous lupus
Q/Q(M)-482152 Report a Problem
Which enzyme can be a useful adjunct test to diagnose zinc deficiency where the serum zinc level is
normal or near normal?
1

Zinc sulfatase
2

Alkaline phosphatase
3

AST
4

ALT
5

Creatinine kinase
Q/Q(M)-477430 Report a Problem

Which enzyme can be a useful adjunct test to diagnose zinc deficiency where the serum zinc level is
normal or near normal?
2

Alkaline phosphatase
A low serum alkaline phosphatase, a zinc dependent enzyme, may be a valuable adjuctive test where the
serum zinc level is normal or near normal. The diagnosis of zinc deficiency should be suspected in at-risk
individuals with acral or periorificial dermatitis. It is usually confirmed by low serum zinc levels.
Q/Q(M)-477430 Report a Problem
60


This patient has similar lesions on his distal extremities. Which laboratory test can be done in order to
make a diagnosis?
1

Hemogram
2

Alkaline phosphatase
3

Creatinine
4

Potassium
5

TSH
Q/Q(M)-480486 Report a Problem


This patient has similar lesions on his distal extremities. Which laboratory test can be done in order to
make a diagnosis?
2

Alkaline phosphatase
This patient has acrodermatitis enteropathica, a rare, inhertied disorder caused by an inability to absorb
zinc. This disease is characterized by a traid of acral dermatitis, diarrhea, and alopecia. AE is rapidly
reversed by zinc supplementation. Alkaline phosphatase is a zinc-dependent enzyme; it is a moderately-
sensitive marker for zinc deficiency (although not an early marker).
Q/Q(M)-480486 Report a Problem



Acute hemorrhagic edema of childhood is distinguished from Henoch-Schonlein Purpura based on:
1

The presence of pupura on the upper trunk
2

The lack of an antecedent infection
3

The involvement of the synovia
4

The neurologic complications
5

The lack of systemic features
Q/Q(M)-480531 Report a Problem
61



Acute hemorrhagic edema of childhood is distinguished from Henoch-Schonlein Purpura based on:
5

The lack of systemic features
Acute hemorrhagic edema of childhood affects children and infants < 2 years of age. It presents with
painful, edematous petechiae and ecchymoses on the head and distal extremities. Facial edema may be
the initial sign. Triggering factors include infection, drugs, and immunization. It lacks the systemic
features of HSP, and resolves in 1-3 weeks without sequelae. HSP occurs mostly in children. There is an
antecedent URI in 75% of cases. HSP involves the skin, synovia, GI tract, and kidneys. Long-term
morbidity results from renal disease, which is predicted by the spread of purpura to the upper trunk.
Q/Q(M)-480531 Report a Problem


Which of the following is true about nevus anemicus:
1

Usually occurs in association with vitiligo
2

Occurs more frequently in men than in women
3

Most commonly involves the upper chest
4

Results from locally decreased vascular reactivity to catecholamines
5

Contains dilated blood vessels
Q/Q(M)-481904 Report a Problem


Which of the following is true about nevus anemicus:
3

Most commonly involves the upper chest
Nevus anemicus is caused by localized hypersensitivity to catecholamines and most commonly found in
the upper chest
Q/Q(M)-481904 Report a Problem


A 32 year-old woman presents with moderate hirsutism. She has normal menses, normal-sized ovaries,
no evidence of tumors of the adrenal or ovary, and normal adrenal function, but does have slight
elevations of plasma androstenedione and testosterone. What is the most likely diagnosis?
1

Stein-Leventhal syndrome
2

Cushing syndrome
3

Idiopathic hirsutism
4

Occult virilizing tumor
5

Kruckenberg tumor
Q/Q(M)-480499 Report a Problem


A 32 year-old woman presents with moderate hirsutism. She has normal menses, normal-sized ovaries,
no evidence of tumors of the adrenal or ovary, and normal adrenal function, but does have slight
elevations of plasma androstenedione and testosterone. What is the most likely diagnosis?
62

3

Idiopathic hirsutism
Idiopathic hirsutism is diagnosed in women with evidence of androgen excess but with normal menses,
normal-sized ovaries, no evidence of tumors of the adrenal or ovary, and normal androgen function.
Slight elevations of plasma androstenedione and testosterone are common.
Q/Q(M)-480499 Report a Problem


Most common cause of these tender lesions on this 12-year old child is
1

Inflammatory bowel disease
2

Tuberculosis
3

beta-hemolytic streptococcal infection
4

Yersinia
5

Infectious mononucleosis
Q/Q(M)-482124 Report a Problem

Most common cause of these tender lesions on this 12-year old child is
3

beta-hemolytic streptococcal infection
The picture shows classical case of erythema nodosum (EN). Although all other choices in question
cause EN, the most common in the pediatric patient is beta-hemolytic streptococcal infection (especially
pharyngitis). The management of erythema nodosum is directed at identification and treatment of the
underlying cause. Minimal evaluation usually involves obtaining an ASO/DNase B titer, chest
radiograph, and tuberculin testing. Bed rest, with elevation of the patient's legs, helps reduce pain and
edema. When pain, inflammation, or arthralgia is prominent, NSAIDs can be prescribed. Salicylates,
colchicine, and potassium iodide are the most commonly used alternative therapies.
Q/Q(M)-482124 Report a Problem


This variant of amyloidosis is derived from degenerated tonofilaments of keratinocytes:
1

Lichen amyloidosis
2

Nodular amyloidosis
3

Primary systemic amyloidosis
4

Dialysis-related amyloidosis
5

Secondary systemic amyloidosis
63

Q/Q(M)-480476 Report a Problem


This variant of amyloidosis is derived from degenerated tonofilaments of keratinocytes:
1

Lichen amyloidosis
Lichen amyloidosis and Macular amyloidosis are derived from degenerated tonofilaments of
keratinocytes. Primary systemic amyloidosis results from deposition of protein AL derived from Ig
Lambda light chains. Nodular amyloidosis is also associated with AL type protein. Secondary systemic
amyloidosis is associated with AA amyloid fibrils derived from SAA protein. Dialysis-related
amyloidosis is associated with beta-2-microglobulin protein deposition.
Q/Q(M)-480476 Report a Problem


Patients with chronic idiopathic urticaria should avoid:
1

Latex products
2

Alcohol
3

Nickel sulfate
4

-blockers
5

aspirin
Q/Q(M)-477167 Report a Problem


Patients with chronic idiopathic urticaria should avoid:
5

aspirin
Chronic idiopathic urticaria is defined by the presence of urticaria of unknown etiology lasting greater
than 6 weeks. Patient with chronic idiopathic urticaria should avoid aspirin as it aggravates urticaria in
about 30% of patients.
Q/Q(M)-477167 Report a Problem

Which of the following is a feature of Sneddon's syndrome?
1

Venous thromboses
2

Hepatosplenomegaly
3

anti-Scl-70 autoantibodies
4

M. tuberculosis infection
5

C2 deficiency
Q/Q(M)-480535 Report a Problem


Which of the following is a feature of Sneddon's syndrome?
1

Venous thromboses
Sneddon's syndrome presents as livedo reticularis and livedoid vasculitis associated with ischemic
64

cerebrovascular lesions, hypotension, and extracerebral arterial and venous thromboses.
Q/Q(M)-480535 Report a Problem


What organism caused this disease?
1

Micrococcus sedenditaris
2

Candida albicans
3

Staphylococcus aureus
4

Corynebacterium tenuis
5

Corynebacterium diptheria
Q/Q(M)-482079 Report a Problem


What organism caused this disease?
1

Micrococcus sedenditaris
Pitted keratolysis is a skin disorder characterized by crateriform pitting that primarily affects the
pressure-bearing aspects of the plantar surface of the feet. It is caused by a cutaneous infection with
micrococcus sedentarius which is included in the Corynebacteria genus. These are gram-positive,
catalase-positive, aerobic or facultatively anaerobic, generally nonmotile rods. Corynebacterium
diphtheriae infection is classically characterized by a local inflammation, usually in the upper respiratory
tract, associated with toxin-mediated cardiac and neural disease. Corynebacterium tenuis causes
trichomycosis Patients typically present with yellow, black, or red pinpoint nodules on the hair shafts in
the inguinal region. Staphylococcus aureus is a common bacterium that can result in formation of
pustules, furuncles, cellulitis and abscesses. Candida species are responsible for superficial infections
such as oropharyngeal candidiasis (thrush) and vulvovaginal candidiasis (vaginal Candidiasis) and is also
occurs as an opportunistic infection
Q/Q(M)-482079 Report a Problem


Acneiform eruptions have been associated with which of the following vitamins?
1

Vitamin C
2

Vitamin E
3

Vitamin A
4

Biotin
5

Vitamin B12
Q/Q(M)-474320 Report a Problem
65


Acneiform eruptions have been associated with which of the following vitamins?
5

Vitamin B12
Vitamin B12 can cause acneiform eruptions.
Q/Q(M)-474320 Report a Problem


Which of the following is true regarding this diagnosis?
1

The protein deposited is derived from Ig light chains, kappa subtype
2

Skin is involved in <10% of cases
3

A different protein is found in skin lesions associated with a plasmacytoma
4

Bullae, when present, are subepidermal
5

Factor V deficiencies are commonly associated with this entity
Q/Q(M)-480502 Report a Problem


Which of the following is true regarding this diagnosis?
4

Bullae, when present, are subepidermal
Primary sytemic amyloidosis involves the skin in 40% of cases. The tongue, heart, and GI tract are
commonly involved. The protein AL is derived from Ig light chains (lambda subtype); AL is also found
in nodular or tumefactive cutaneous amyloidosis produced by a plasmacytoma. Glossitis is common, and
may lead to dysphagia; the lateral aspects of the tongue often show indentations from teeth. Purpuric
lesions result from amyloid infiltration of blood vessels, and occur after trauma (pinch purpura). Bullous
amyloidosis presents with tense, hemorrhagic bullae at areas of trauma; lesions are subepidermal.
Arthropathies of small joints, enlarged deltoids (shoulder pad sign), factor IX and X deficiencies, cardiac
arrythmias, and CHF may all result.
Q/Q(M)-480502 Report a Problem



What condition does this patient have given the extent of arthritis seen in this X-ray?
1

Antiphospholipid syndrome
2

Multicentric reticulohistiocytosis
3

Alpha-1-antitrypsin syndrome
4

Eosinophilic fasciitis
66

5

Dermatomyositis
Q/Q(M)-476821 Report a Problem

What condition does this patient have given the extent of arthritis seen in this X-ray?
2

Multicentric reticulohistiocytosis
Multicentric reticulohistiocytosis is a non-Langerhans histiocytosis associated with mutilating arthritis.
Dermatologically, patient may present with coral beading around the fingers. There is a 30% incidence of
malignancy in these patients.
Q/Q(M)-476821 Report a Problem


Phrynoderma is associated with deficiency of which vitamin?
1

Vitamin A
2

Vitamin B1
3

Vitamin B6
4

Vitamin D
5

Vitamin K
Q/Q(M)-480471 Report a Problem



Phrynoderma is associated with deficiency of which vitamin?
1

Vitamin A
Phrynoderma (toad skin) presents clinically as keratotic papules on the extremities and shoulders and
results from a deficiency of Vitamin A. Vitamin A is a fat soluble vitamin. Hypovitaminosis A is often
seen in diseases such as Crohn's disease, celiac disease and cystic fibrosis which manifest with fat
malabsorbtion. Hypovitaminosis A is additionally associated with night blindness, xeropthalmia and
keratomalacia.
Q/Q(M)-480471 Report a Problem


Carcinoid tumors in which site do not lead to flushing or other manifestations of the carcinoid syndrome?
67

1

Appendix
2

Ileum
3

Ascending colon
4

Rectum
5

Brochus
Q/Q(M)-480510 Report a Problem


Carcinoid tumors in which site do not lead to flushing or other manifestations of the carcinoid syndrome?
4

Rectum
Carcinoid tumors are derived from enterochromaffin (Kulchitsky) cells. The appendix is the most
common site, followed by the ileum. Less than 4% of abdominal carcinoid tumors have the carcinoid
syndrome; the presence of the syndrome implies hepatic metastases, extraabdominal carcinoid tumor, or
a large enough tumor burden such that the liver cannot degrade the increased level of hormone. Foregut
tumors (bronchus, stomach, pancreas) produce histamine, cause peptide ulcers, and produce more
persistent, intense flushing, and lacrimation, sweating, vomiting, and asthma. Midgut tumors (small-
intestine to mid-colon) cause cyanotic flush, hypotension, and bronchoconstriction more commonly.
Hindgut tumors (descending colon and rectal) do not lead to flushing or other manifestations.
Q/Q(M)-480510 Report a Problem


The first step in the formation of a closed comedone is:
1

Obstruction of the pilosebaceous duct
2

Increased sebum production
3

Colonization by P. acnes
4

Increased hormonal stimulation of the pilosebaceous unit
5

Clogging of the pore with make-up
Q/Q(M)-478076 Report a Problem



The first step in the formation of a closed comedone is:
1

Obstruction of the pilosebaceous duct
The first step in the formation of a microcomedone is the obstruction of the pilosebaceous unit by
keratinocytes. After that has occured, increased sebum and P. acnes can contribute to the further
formation of the comedone.
Q/Q(M)-478076 Report a Problem

Increased LDL and VLDL is seen in which type of hyperlipoproteinemia?
1

Type IIb
2

Type I
68

3

Type III
4

Type IV
5

Type V
Q/Q(M)-480466 Report a Problem



Increased LDL and VLDL is seen in which type of hyperlipoproteinemia?
1

Type IIb
Type IIb hyperlipoproteinemia is autosomal dominant and features increased LDL and VLDL.
Associated clinical findings include xanthelasma, planar, tendinous and tuberous xanthomas. Type I
hyperlipoproteinemia is autosomal recessive and manifests with increased chylomicrons. Clinically,
patients develop eruptive xanthomas and lipemia retinalis. Type IIa hyperlipoproteinemia is autosomal
dominant. Patients have increased LDL and may develop tendinous, tuberous, planar or eruptive
xanthomas in addition to xanthelasma and arcus juvenilis. Type III hyperlipoproteinemia is autosomal
recessive and features increased IDL. Clinically, these patients are prone to develop palmar, planar,
tendinous, tuberous and eruptive xanthomas. Type IV hyperlipoproteinemia is autosomal dominant and
manifests with increased VLDL. Patients develop eruptive, tendinous and tuberous xanthomas. Type V
hyperlipoproteinemia is autosomal dominant. Patients have increased chylomicrons and VLDL and may
develop eruptive xanthomas and lipemia retinalis.
Q/Q(M)-480466 Report a Problem


A healthy 50 year old man presents to your office with 3 red-brown indurated plaques on his forehead.
Biopsy reveals a focal leukocytoclastic vasculitis, with a diffuse infiltration of neutrophils, eosinophils,
and lymphocytes. Which of the following is a potential treatment option for this patient?
1

Q-switched alexandrite laser
2

Pulse dye laser
3

Q-switched ruby laser
4

Nd:YAG laser
5

Frequency-doubled Nd:YAG laser
Q/Q(M)-482542 Report a Problem


A healthy 50 year old man presents to your office with 3 red-brown indurated plaques on his forehead.
Biopsy reveals a focal leukocytoclastic vasculitis, with a diffuse infiltration of neutrophils, eosinophils,
and lymphocytes. Which of the following is a potential treatment option for this patient?
2

Pulse dye laser
The question stem describes a case of granuloma faciale. This condition presents with red-brown papules
and plaques on the face, most commonly in middle-aged Caucasian males. While traditional treatment
options include intralesional steroids with or without cryotherapy, pulse dye laser (595 nm) represents
another treatment option for this condition, with successful clearance in a number of case reports.
Q/Q(M)-482542 Report a Problem


69

Child abuse is often incorrectly suspected when a young girl presents with which cutaneous finding?
1

Molluscum contagiosum
2

Lichen sclerosus et atrophicus
3

Condyloma accuminata
4

Ecchymoses on face and hands
5

Genital herpes simplex
Q/Q(M)-477372 Report a Problem

Child abuse is often incorrectly suspected when a young girl presents with which cutaneous finding?
2

Lichen sclerosus et atrophicus
Lichen sclerosus et atrophicus is an inflammatory disease of the skin that most often occurs oin the
genital region. Clinically, lichen sclerosus may appear as a hypopigmented, well demarcated patch with a
crinkled appearance. It is often pruritic and if left untreated may result in progressive scarring. The
treatment of choice is ultra potent topical steroids.
Q/Q(M)-477372 Report a Problem


The primary component of human sebum is:
1

Triglycerides
2

Free fatty acids
3

Ceramides
4

Cholesterol
5

Squalene
Q/Q(M)-474325 Report a Problem


The primary component of human sebum is:
1

Triglycerides
Human sebum is rich in triglycerides. The lipase made by P. acnes cleaves triglycerides to free fatty
acids.
Q/Q(M)-474325 Report a Problem



The most important mediator of retinoid activity in the skin is:
1

RAR-alpha
2

RAR-beta
3

RAR-gamma
4

RXR-alpha
70

5

RXR-beta
Q/Q(M)-474314 Report a Problem


The most important mediator of retinoid activity in the skin is:
3

RAR-gamma
Retinoid activity in humans is mediated by retinoid receptors. Two groups exist: RA receptors (RAR)
and RX receptors (RXR). Each has three receptor subtypes: alpha, beta, and gamma. RAR-gamma is the
most important mediator of retinoid activity in the skin.
Q/Q(M)-474314 Report a Problem


Which of the following are risk factor(s) for post-transplant CTCL?
1

Renal transplant
2

Liver transplant
3

Cyclosporine therapy
4

Female Sex
5

Renal transplant and Cyclosporine therapy
Q/Q(M)-482191 Report a Problem

Which of the following are risk factor(s) for post-transplant CTCL?
5

Renal transplant and Cyclosporine therapy
In a recent case series and review (Arch Dermatol. 2010; 146 (5): 513-516) it was shown that the
following are associated with post-transplant CTCL: renal transplant, cyclosporine, tacrolimus, and male
sex. There have been 29 cases of post-transplant CTCL documented in the literature to date.
Q/Q(M)-482191 Report a Problem


Which of the following receptor is the most important mediator of retinoid activity in the skin?
1

RAR-gamma
2

RXR-alpha
3

RXR-gamma
4

RAR-beta
5

RXR-beta
Q/Q(M)-480375 Report a Problem


Which of the following receptor is the most important mediator of retinoid activity in the skin?
1

RAR-gamma
Retinoid activity is mediated by retinoid receptors. Two groups exist: RA receptors (RAR) and RX
receptors (RXR). Each has three receptor subtypes: alpha, beta, gamma. RAR-gamma is the most
71

important mediator of retinoid activity in the skin.
Q/Q(M)-480375 Report a Problem

A patient presents to the ER with skin pain and large sheets of epidermal detachment on greater than 10%
of his body surface area after starting bactrim. No purpuric macules or target lesions are noted. Which of
the following can help predict mortality in this patient?
1

glucose greater than 150 mg/dL
2

age over 40 years
3

heart rate greater than 100
4

history of diabetes
5

BUN greater than 15 mg/dL
Q/Q(M)-482517 Report a Problem

A patient presents to the ER with skin pain and large sheets of epidermal detachment on greater than 10%
of his body surface area after starting bactrim. No purpuric macules or target lesions are noted. Which of
the following can help predict mortality in this patient?
2

age over 40 years
The patient described has TEN. A diagnosis of TEN can be made if the patient presents with epidermal
detachment of >30% of the body surface with widespread purpuric macules or flat atypical targets, or if
there are large sheets of epidermal detachment involving >10% of the body surface without purpuric
macules or target lesions. The SCORTEN scoring system was developed to assess severity of illness and
predict mortality in TEN, and is meant to be calculated within the first 24 hours after admission and again
on day three. The score is the sum of the following clinical variables: (1) age over 40 years; (2) heart rate
>120 beats per minute; (3) the presence of cancer or hematologic malignancy; (4) epidermal detachment
involving body surface area >10% on day one; (5) blood urea nitrogen >28 mg/dL (10 mmol/L); (6)
glucose >252 mg/dL (14 mmol/L); and (7) bicarbonate >20 mEq/L. The mortality increases sharply with
each additional point, with a score of 5 or greater having a 90% mortality.
Q/Q(M)-482517 Report a Problem


Which of the following is not associated with this condition?
1

Poor oral hygeine
2

Antibiotic use
3

Aging
4

Thrush
5

Smoking
Q/Q(M)-476885 Report a Problem
72



Which of the following is not associated with this condition?
3

Aging
Black hairy tongue is a common reactive condition due to the enlongation of filiform papillae. The most
common etiologies include poor oral hygeine, use of oral antibiotics, HIV infection, use of tobacco,
radiation, and alcohol.
Q/Q(M)-476885 Report a Problem



The most common laboratory abnormality in patients treated with isotretinoin is:
1

Decreased white blood cell count
2

Increased cholesterol
3

Elevated liver enzymes
4

Hypertriglyceridemia
5

Elevated CPK
Q/Q(M)-474790 Report a Problem


The most common laboratory abnormality in patients treated with isotretinoin is:
4

Hypertriglyceridemia
The most common laboratory abnormality seen in patients taking isotretinoin is increased triglycerides,
followed by elevation of ALT and AST.
Q/Q(M)-474790 Report a Problem


Lichen planus pemphigoides has been associated with an antigen to which structure?
1

Type XVII collagen
2

Hemidesmosome plaque
3

Desmoglein 3
4

Desmoglein 1
73

5

Type VII collagen
Q/Q(M)-482156 Report a Problem



Lichen planus pemphigoides has been associated with an antigen to which structure?
1

Type XVII collagen
Type XVII collagen, or BPag2 (180 kD) has been associated with LP pemphigoides. The
hemisdesomosome plauqe is also known as BPag1 (230 - kD), desmoglein 1 and 3 are associated with
the Pemphigous family, and antibodies to Type VII collagen is found in Epidermolysis Bullosa Aquisita
Q/Q(M)-482156 Report a Problem


A 3 year-old girl has a red-yellow papule on her cheek which her mother is extremely anxious about.
Skin biopsy reveals a juvenile xanthogranuloma. Which specialist should you refer the patient to?
1

Plastic surgeon
2

Endocrinologist
3

Neurologist
4

Ophthalmologist
5

Oncologist
Q/Q(M)-477314 Report a Problem



A 3 year-old girl has a red-yellow papule on her cheek which her mother is extremely anxious about.
Skin biopsy reveals a juvenile xanthogranuloma. Which specialist should you refer the patient to?
4

Ophthalmologist
Juvenile xanthogranulomas are the most common form of non-Langerhans cell histiocytoses. 80% of
these lesions appear within the first year of life and they present as firm, red-brown to yellowish papule.
The most common noncutaneous site is the iris, so referral to ophthalmologist is warranted.
Q/Q(M)-477314 Report a Problem

What is the eponym used for this inherited, congenital condition?
1

Meige disease
2

Milroy disease
3

Proteus syndrome
4

Klippel-Trenaunay syndrome
5

Stuart-Treves syndrome
Q/Q(M)-476882 Report a Problem
74



What is the eponym used for this inherited, congenital condition?
2

Milroy disease
Milroy disease in an autosomal dominantly inherited form of primary lymphedema. Unlike Meige
disease, Milroy disease is congenital and due to a failure of the development of lymphatic vessels in
utero.
Q/Q(M)-476882 Report a Problem

The lesions shown here are most likely to:
1

Occur in infants and elderly patients
2

Occur as an adverse reaction to medication
3

Be refractory to treatment
4

Follow a streptococcal infection
5

Erupt after the appearance of a herald patch
Q/Q(M)-474892 Report a Problem

The lesions shown here are most likely to:
4

Follow a streptococcal infection
This slide shows guttate psoriasis.
Q/Q(M)-474892 Report a Problem

A patient begins to lose the fat in her face and upper torso acutely after a viral illness. Which internal
organ may be affected by this disease?
75

1

Lungs
2

CNS
3

Bone marrow
4

Kidney
5

Heart
Q/Q(M)-482795 Report a Problem


A patient begins to lose the fat in her face and upper torso acutely after a viral illness. Which internal
organ may be affected by this disease?
4

Kidney
Barraquer-Simons syndrome, aka acquired partial lipodystrophy, presents typically in females and is
secondary to a mutation in LMNB2. It is often preceded by a viral illness. More than one third of patients
will have glomerulonephritis, which may lead to chronic renal sequelae.
Q/Q(M)-482795 Report a Problem


What is the most common paraproteinemia in scleromyxedema?
1

IgA
2

IgG kappa
3

IgG lambda
4

IgM
5

It is extremely uncommon to see a paraproteinemia with scleromyxedema
Q/Q(M)-477447 Report a Problem


What is the most common paraproteinemia in scleromyxedema?
3

IgG lambda
An abnormal paraproteinemia is found in 90% of cases of scleromyxedema, usually IgG lamda.
Q/Q(M)-477447 Report a Problem



All of the following are true regarding Reiter's syndrome except:
1

The classic clinical triad is urethritis, conjunctivitis and arthritis
2

Usually occurs in young women of HLA-B27 genotype
3

May be associated with keratoderma belnnorrhagicum
4

May be associated with Chlamydia trachomatis
5

Nails may become thick and brittle with heavy subungual hyperkeratotic deposits
76

Q/Q(M)-482286 Report a Problem


All of the following are true regarding Reiter's syndrome except:
2

Usually occurs in young women of HLA-B27 genotype
Reiter's syndrome is a chronic inflammatory disease similar to psoriasis with psoriatoc arthritis.
Reiter's syndrome usually occurs in men of HLA-B27 genotype and rarely occurs in women. Few
patients present with the classic triad, thus can be diagnosed with peripheral arthritis >1 month and
associated urethritis. Keratoderma blennorrhagicum is crusted, hyperkeratotic papules and plaques on
plantar surfaces. Painful and bloody urination and pyuria, cystitis, prostatitis and seminal vesiculitis may
occur secondary to Chlamydia trachomatis. Nail lesions as described may occur.
Q/Q(M)-482286 Report a Problem

The most common associated disorder in a patient with elastosis perforans serpiginosa is:
1

Marfan syndrome
2

Down syndrome
3

Osteogenesis imperfecta
4

Ehlers-Danlos syndrome
5

Rothmund-Thomsom syndrome
Q/Q(M)-477633 Report a Problem


The most common associated disorder in a patient with elastosis perforans serpiginosa is:
2

Down syndrome
About 1/3 of case of elastosis perforans serpiginosa occur in patients with other concomitant disorders.
All of the above can occur in patients with EPS; however the most common is Down syndrome. One
mnemonic to remember the associated disorders is: PROMEDA. This stands for PXE, Rothmund-
Thomson, Osteogenesis imperfecta, Marfan syndrome, Ehlers-Danlos syndrome, Down syndrome,
acrogeria.
Q/Q(M)-477633 Report a Problem


The condition shown here may be exacerbated by the inappropriate use of which of the following?
1

Comedogenic make-up foundation
2

Sunscreens
3

Erythromycin 2% ointment
4

Topical corticosteroids
5

Mineral oil-containing moisturizers
Q/Q(M)-474886 Report a Problem
77



The condition shown here may be exacerbated by the inappropriate use of which of the following?
4

Topical corticosteroids
This slide shows perioral dermatitis, characterized by small, erythematous papules in a perioral, often
periorificial, distribution. Topical corticosteroids can exacerbate or trigger perioral dermatitis. Note the
absence of comedones, which distinguishes rosacea and its subsets from acne.
Q/Q(M)-474886 Report a Problem


A 50-year-old female with sarcoid has enlargement of the parotid, submandibular, and lacrimal glands
that is also known as:
1

Mikulicz's syndrome
2

Heerfordt-Waldenstromn syndrome
3

Darier-Roussy sarcoid
4

Lofgren's syndrome
5

Blau's syndrome
Q/Q(M)-482263 Report a Problem



A 50-year-old female with sarcoid has enlargement of the parotid, submandibular, and lacrimal glands
that is also known as:
1

Mikulicz's syndrome
This patient has Mikulicz's syndrome which is sarcoid of the parotid, submandibular and lacrimal glands.
Heerfordt-Waldenstromn syndrome demonstrates a combination of fever, parotid enlargement, anterior
uveitis, and facial nerve palsy. Darier-Roussy sarcoid is characterized by subcutaneous nodular sarcoid
on the trunk and extremities. Lofgren's syndrome features acute sarcoid, erythema nodosum and
migratory polyarthritis, fever, iritis, and bilateral hilar adenopathy. Blau's syndrome is a rare autosomal
dominant familial granulomatous syndrome that presents with arthritis, uveitis, and skin lesions that
appear as "red dots;" there is no pulmonary involvement.
Q/Q(M)-482263 Report a Problem
What is the diagnosis of this lesion on the penile shaft?
78

1

Sclerosing lymphangitis
2

Loaloa
3

Varicose vein
4

Syphilis
5

Lymphogranuloma venereum
Q/Q(M)-476867 Report a Problem



What is the diagnosis of this lesion on the penile shaft?
1

Sclerosing lymphangitis
Sclerosing lymphangitis is a self-limited, cord-like structure on the penile shaft, which typically occurs
24-48 hours after vigorous sexual intercourse.
Q/Q(M)-476867 Report a Problem



Angiolymphoid hyperplasia with eosinophilia most commonly occurs on the:
1

Ear
2

Lip
3

Dorsal hands
4

Trunk
5

Lower extremities
Q/Q(M)-482676 Report a Problem


Angiolymphoid hyperplasia with eosinophilia most commonly occurs on the:
1

Ear
Angiolymphoid hyperplasia with eosinophilia often presents with red to reddish-brown papules or
nodules on the head or neck. While lesions can occur on any body site, most common locations are the
79

ears and scalp.
Q/Q(M)-482676 Report a Problem


Concomitant discoid lupus is found in what percentage of patients with subacute cutaneous lupus?
1

5%
2

20%
3

40%
4

60%
5

80%
Q/Q(M)-480512 Report a Problem

Concomitant discoid lupus is found in what percentage of patients with subacute cutaneous lupus?
2

20%
Sub acute cutaneous lupus presents with scaly papules that evolve into either psoriasiform or polycyclic
annular lesions (more commonly). Sun-exposed surfaces of the face and neck are commonly involves.
Photo sensitivity is seen in 40%, the hard palate is involved in 40%, and concomitant DLE is seen in
20%.
Q/Q(M)-480512 Report a Problem

Eosinophilia, asthma, neuropathy and sinus abnormalities are associated with which of the following
systemic vasculitidies?
1

Churg-Strauss syndrome
2

Polyarteritis nodosa
3

Microscopic polyangiitis
4

Wegener's granulomatosis
5

Kawasaki disease
Q/Q(M)-480470 Report a Problem

Eosinophilia, asthma, neuropathy and sinus abnormalities are associated with which of the following
systemic vasculitidies?
1

Churg-Strauss syndrome
Churg-Strauss syndrome has six diagnostic criteria as set by the American College of Rheumatology:
Blood Eosinophilia, Asthma, Neuropathy, Sinus abnormalities, Allergies and Perivascular eosinophils.
The presence of four of these six criteria yields a diagnostic sensitivity of 85% and specificity of 99.7%.
Q/Q(M)-480470 Report a Problem


Which statement best describes the mechanism of action of spironolactone?
80

1

Non-steroid inhibitor of DHT binding to the androgen receptor
2

Progestin
3

GnRH agonist
4

Mineralocorticoid
5

Steroid inhibitor of androgen receptor and androgen biosynthesis
Q/Q(M)-474304 Report a Problem

Which statement best describes the mechanism of action of spironolactone?
5

Steroid inhibitor of androgen receptor and androgen biosynthesis
Spironolactone is a steroid molecule. It acts as an anti-androgen by blocking the androgen receptor and
inhibiting androgen synthesis.
Q/Q(M)-474304 Report a Problem



The most common finding in patients with systemic sclerosis is:
1

Migratory polyarthritis
2

Esophageal dysfunction
3

Pulmonary fibrosis
4

Renal disease
5

Cardiac conduction defects
Q/Q(M)-480067 Report a Problem


The most common finding in patients with systemic sclerosis is:
2

Esophageal dysfunction
Esophageal dysfunction is the most common systemic finding in systemic sclerosis. A migratory
polyarthritis is the first manifestation of disease in many. Pulmonary fibrosis, renal disease and cardiac
conduction defects are all manifestations but not the most common.
Q/Q(M)-480067 Report a Problem


What is the treatment of choice for this patient who developed acne fulminans one month after starting
isotretinoin therapy?
1

Add oral prednisone
2

Increase isotretinoin dose
3

Add doxycycline
4

Acne surgery
5

Check lipid levels
81

Q/Q(M)-476875 Report a Problem


What is the treatment of choice for this patient who developed acne fulminans one month after starting
isotretinoin therapy?
1

Add oral prednisone
Acne fulminans is an explosive form of acne in which patients may develop systemic symptoms,
ulcerated nodules on the face and trunk. Acne fulminans may be triggered by initiation of isotretinoin
therapy. The treatment of choice is systemic steroids at a dosage of 1 mg/kg/day tapered over the course
of 6 weeks.
Q/Q(M)-476875 Report a Problem


Mutations in the MEFV gene product, pyrin, produce an autosomal recessive syndrome characterized by
recurrent fevers, peritonitis, pleuritis, arthritis and erysipelas-like erythema. Which of the following
syndromes is described above?
1

Familial Mediterranean Fever
2

PAPA syndrome
3

TNF receptor associated periodic syndrome
4

Hyper IgD syndrome
5

Familial cold autoinflammatory syndrome
Q/Q(M)-479411 Report a Problem

Mutations in the MEFV gene product, pyrin, produce an autosomal recessive syndrome characterized by
recurrent fevers, peritonitis, pleuritis, arthritis and erysipelas-like erythema. Which of the following
syndromes is described above?
1

Familial Mediterranean Fever
Familial Mediterranean Fever is described above. It is autosomal recessive with incomplete penetrance.
The main therapy is Colchicine. PAPA syndrome, TNF receptor associated periodic syndrome, hyper-
IgD syndrome and familial cold autoinflammatory syndrome are related autoinflammatory syndromes.
PAPA syndrome is Pyogenic Arthritis, Pyoderma gangrenosum and Acne and is caused by proline serine
82

threonine phosphatase-interacting protein (PSTPIP1) or CD2-binding protein 1 (CD2BP1) which interact
with pyrin. TNF receptor associated periodic syndrome (TRAPS) has similar findings to FMF, but the
attacks are of longer duration, is autosomal dominant in transmission, and does not respond to colchicine.
TRAPS is caused by a mutation in the TNFRSF1A gene which results in a decrease in soluble TNF
receptor. Hyper IgD syndrome is associated with defects in the mevalonate kinase gene, which presents
with a hereditary periodic fever. Familial cold autoinflammatory syndrome is associated with cryopyrin
defects and is characterized by fever, rash, conjunctivitis and arthralgia upon exposure to cold.
Q/Q(M)-479411 Report a Problem


Which of the following is a cause of immunologic urticaria?
1

Polymyxin B
2

Amoxicillin
3

Ibuprofen
4

Opiates
5

Tartrazine
Q/Q(M)-479412 Report a Problem


Which of the following is a cause of immunologic urticaria?
2

Amoxicillin
Amoxicillin is in the penicillin family of antibiotics and is an immunologic cause of urticaria.
Immunologic urticaria is most commonly caused by exposure to this family and other related beta-lactam
antibiotics. Patients with reaction to penicillins have an increased risk of cross-reacting to cephalosporins,
mostly the earlier generations. The third-generation cephalosporins are less likely to cause reactions in a
penicillin allergic patient. The other listed options are causes of non-immunologic urticaria. They alter
prostaglandin metabolism which increases degranulation of mast cells.
Q/Q(M)-479412 Report a Problem


This 57-year old male complains of weakness when climbing the stairs and this clinical presentation.
Which cytokine has been imnplicated in the etiology of this condition?
1

IL-1
2

Il-10
3

Interferon-alpha
4

Interferon-gamma
5

Tumor necrosis factor alpha
Q/Q(M)-476817 Report a Problem
83




This 57-year old male complains of weakness when climbing the stairs and this clinical presentation.
Which cytokine has been imnplicated in the etiology of this condition?
5

Tumor necrosis factor alpha
Dermatomyositis is an autoimmune condition which presents with typical skin findings and muscle
weakness. Age-appropriate screening should be done for internal malignancy as there is a higher
incidence of cancer in these patients. Polymorphisms of tumor necrosis factor-alpha have been implicated
in the etiology.
Q/Q(M)-476817 Report a Problem


A 35yo patient was complaining of skin eruption on her both ears. What is the most likely culprit?
1

Imitanib
2

Panitumumab
3

Adalimumab
4

Infliximab
5

Rituximab
Q/Q(M)-482884 Report a Problem



A 35yo patient was complaining of skin eruption on her both ears. What is the most likely culprit?
2

Panitumumab
It is an epidermal growth factor receptor inhibitor(EGFRI). It is used in various types of cancers such as
colorectal cancer. Epidermal growth factors inhibitors are associated with acneiform skin eruptions.
There is some evidence to suggest that the appearance of skin rash may be useful as a surrogate marker of
EGFRI efficacy. Examples of EGFRI are erlotinib, cetuximab, panitumumab.
Q/Q(M)-482884 Report a Problem

The most common associated malignancy with dermatomyositis is:
1

Colon carcinoma
2

Hepatocellular carcinoma
3

Renal cell carcinoma
84

4

Esophageal carcinoma
5

Ovarian carcinoma
Q/Q(M)-474337 Report a Problem

The most common associated malignancy with dermatomyositis is:
5

Ovarian carcinoma
Patients with dermatomyositis (DM) may have cancer that precedes, occurs simultaneously as, or follows
the diagnosis of DM. Ovarian cancer is overrepresented in this population.
Q/Q(M)-474337 Report a Problem

Which of the following cytokines is most characteristic of atopic dermatitis in its chronic phase?
1

Interferon-gamma
2

IL-4
3

IL-5
4

IL-10
5

IL-13
Q/Q(M)-479343 Report a Problem


Which of the following cytokines is most characteristic of atopic dermatitis in its chronic phase?
1

Interferon-gamma
While acute atopic dermatitis is a TH2 state, chronic eczematous AD is most often a TH1 cytokine
environment with interferon-gamma being the best choice. Initial lesions TH2 dominant, cytokines IL-
4,5,10,13. IL-4/5 produce elevated IgE and eosinophilia, IL-10 inhibits delayed-type hypersensitivity. IL-
4 downregulates IFN-gamma.
Q/Q(M)-479343 Report a Problem

The presentation of a foregut carcinoid tumor involves:
1

The production of histamine, cyanotic flush, and bronchoconstriction
2

The production of serotonin, intense flushing, peptic ulcer, and lacrimation
3

To cutaneous findings
4

The production of kallikrein with bronchial asthma and angioedema
5

Frequent episodes of tongue swelling and urticaria
Q/Q(M)-474335 Report a Problem


The presentation of a foregut carcinoid tumor involves:
2

The production of serotonin, intense flushing, peptic ulcer, and lacrimation
85

Carcinoid syndrome involves tumors derived from the enterochromaffin cells. The appendix is the most
common site. Tumors can be foregut (bronchus, stomach, or pancreas), midgut (small intestine to mid-
colon), or hindgut (descending colon and rectal). Tumors from the foregut area produce serotonin, cause
peptic ulcers, and produce intense, persistent flushing and lacrimation.
Q/Q(M)-474335 Report a Problem

What is the condition which is a diagnostic cutaneous manifestation of sarcoid?
1

Discoid lupus
2

Lupus pernio
3

Lupus vulgaris
4

Granulomatous rosacea
5

Rhinoscleroma
Q/Q(M)-476819 Report a Problem


What is the condition which is a diagnostic cutaneous manifestation of sarcoid?
2

Lupus pernio
Sarcoidosis is a systemic disease charcterized by non-caseating granulomas. Organ involved include the
skin, lung, liver and eyes. Lupus pernio manifests as indurated, red-brown, swollen plaques of the nose,
lips, cheeks and ears.
Q/Q(M)-476819 Report a Problem
What characteristic body would be found on histopathology of this disease?
1

Kamino
2

Civatte
3

Henderson-Patterson
4

Donovan
5

Dutcher
Q/Q(M)-476879 Report a Problem


86


What characteristic body would be found on histopathology of this disease?
2

Civatte
Hypertrophic lichen planus is a type of lichen planus characterized by hyperkeratosis. They are usually
pruritic and often found on the extremities. Civatte bodies are degenerated keratinocytes often seen in
lichen planus.
Q/Q(M)-476879 Report a Problem

A young woman presents with tender, erythematous nodules an the anterior lower extremities. Which of
the following would not be an appropriate test to consider?
1

TSH
2

ESR
3

ASO
4

Fungal cultures
5

PPD
Q/Q(M)-482164 Report a Problem




A young woman presents with tender, erythematous nodules an the anterior lower extremities. Which of
the following would not be an appropriate test to consider?
1

TSH
Erythema nodosum can be triggered by several medical conditions, including drugs, IBD, infections
(strep, hepatitis, TB, fungus), sarcoidosis, and malginancy. Thyroid disease is not a known trigger for
erythema nodosum
Q/Q(M)-482164 Report a Problem



A 35 year old man with plaque psoriasis well-controlled with twice daily application of calcipotriene
0.005% cream noticed a flare of his psoriasis shortly after he started using a prescription moisturizer
87

lotion for psoriasis. Which is the most likely explanation for the apparent decreased efficacy of
calcipotriene?
1

The moisturizer impaired penetration of the calcipotriene
2

The moisturizer diluted the calcipotriene
3

The two events are unrelated
4

The prescription moisturizer likely contains lactic acid.
5

The decreased efficacy is due to tachyphylaxis
Q/Q(M)-474300 Report a Problem


A 35 year old man with plaque psoriasis well-controlled with twice daily application of calcipotriene
0.005% cream noticed a flare of his psoriasis shortly after he started using a prescription moisturizer
lotion for psoriasis. Which is the most likely explanation for the apparent decreased efficacy of
calcipotriene?
4

The prescription moisturizer likely contains lactic acid.
The active ingredient in Dovonex is easily inactivated, particularly by acidic compounds like salicylic
acid and lactic acid.
Q/Q(M)-474300 Report a Problem


A 28 year old previously healthy man presents with thick crusting/hyperkeratosis of the palms and soles,
urethritis, and one month of peripheral arthritis. What of the following is true regarding his condition
1

Females and males are equally affected
2

A chronic deforming arthritis occurs in 20%
3

TNF-alpha inhibiting agents have no role in the treatment of this condition
4

Patients must have urethritis, conjunctivitis, and arthritis for diagnosis
5

Clamydia cervicitis is not associated with this condition
Q/Q(M)-482151 Report a Problem


A 28 year old previously healthy man presents with thick crusting/hyperkeratosis of the palms and soles,
urethritis, and one month of peripheral arthritis. What of the following is true regarding his condition
2

A chronic deforming arthritis occurs in 20%
The patient has Reiter's syndrome. Reiter's syndrome is a chronic inflammatory disease similar to
psoriasis with psoriatic arthritis, and is though to be a variant form. The classic triad includes urethritis,
conjunctivitis, and arthritis. Few patients present with the classic triad, and thus the syndrome can be
88

diagnosed with peripheral arthritis for greater then one month duration and rarely occurs in women. Skin
findings include keratoderma blennorrhagicum and circinate balanitis (in men), as well as oral erosions,
sever stomatitis, and nail changes. Chronic deforming arthritis occurs in 20% of patients. Treatments
include topical steroids, NSAIDs, methotrexate, acitretin, cyclosporin, and TNF-inhibiting biologics such
as etanercept.
Q/Q(M)-482151 Report a Problem


Which of the following medications is most likely to induce a flare of this patient's disease?
1

ciprofloxacin
2

captopril
3

diphenhydramine
4

sertraline
5

acetominophen
Q/Q(M)-480426 Report a Problem




Which of the following medications is most likely to induce a flare of this patient's disease?
2

captopril
Medications that have been reported to cause flares of psoriasis include antimalarials, beta blockers,
NSAIDS, penicillin, tetracycline, ACE inhibitors, G-CSF, interferons and lithium. In addition,
withdrawal from systemic corticosteroids can induce a flare.
Q/Q(M)-480426 Report a Problem

Acquired angioedema is characterized by:
1

Inheritance
2

Decreased levels of C1
3

Self-limited course
4

Association with underlying malignancy
5

Increased C1 esterase inhibitor
89

Q/Q(M)-475872 Report a Problem


Acquired angioedema is characterized by:
4

Association with underlying malignancy
Hereditary angioedema is an autosomal dominant condition associated with recurrent attacks of
angioedema of the skin, respiratory, and gastrointestinal tract, without urticaria and is caused by a
functional deficiency in C1 esterase inhibitor. In contrast, acquired angioedema is caused by a depletion
in C1 esterase inhibitor. There are two forms of acquired angioedema; one results from the production of
antibodies directed against C1 esterase inhibitor, and the other is seen in association with underlying
malignancies, especially B cell lymphomas, with antibody production against overexpressed
paraproteins. Acquired angioedema can be distinguished from hereditary angioedema by differences in
serum levels of C1. While the level is markedly decreased in acquired angioedema, a normal C1 is seen
in patients with hereditary angioedema.
Q/Q(M)-475872 Report a Problem

The constellation of erythema nodosum, bilateral hilar lymphadenopathy, uveitis, fever, and arthritis in
patients with sarcoidosis is known as:
1

Loeffler's syndrome
2

Schnitzler's syndrome
3

Lofgren's syndrome
4

Heerfordt's syndrome
5

Mikulicz's syndrome
Q/Q(M)-477754 Report a Problem


The constellation of erythema nodosum, bilateral hilar lymphadenopathy, uveitis, fever, and arthritis in
patients with sarcoidosis is known as:
3

Lofgren's syndrome
The presence of EN, bilateral hilar lymphadenopathy, uveitis, fever, and arthritis in patients with
sarcoidosis is known as Lofgren's syndrome. Heerfordt's syndrome, also known as uveoparotid fever,
frequently occurs in patients with sarcoidosis of the central nervous syndrome. It consists of uveitis,
facial nerve palsy, fever, and parotid gland involvement. Milulicz's syndrome is bilateral sarcoidosis of
the parotid, submandibular, sublingual, and lacrimal glands. Loeffler's syndrome is characterized by a
patchy infiltrate in the lungs and eosinophilia of the blood and sputum. Loeffler's syndrome can be with
creeping eruption (larva migrans). Schnitzler's syndrome is a rare disorder of chronic urticaria, fever,
disabling bone pain, hyperostosis, increased erythrocyte sedimentation rate, and monoclonal IgM
gammopathy. Pruritus is not a feature of Schnitzler syndrome.
Q/Q(M)-477754 Report a Problem

Which of the following is a progestin used in oral contraceptives with low intrinsic androgenic
properties?
1

Norgestrel
90

2

Norethindrone
3

Norgestimate
4

Levonorgestrel
5

Spironolactone
Q/Q(M)-474312 Report a Problem

Which of the following is a progestin used in oral contraceptives with low intrinsic androgenic
properties?
3

Norgestimate
Desogestril, norgestimate and gestodone are progestins with low intrinsic androgenic properties.
Q/Q(M)-474312 Report a Problem

The "follicular occlusion triad" consists of:
1

Acne conglobata, hidradenitis suppurativa, kerion
2

Acne conglobata, chloracne, pyoderma faciale
3

Acne fulminans, hidradenitis suppurativa, dissecting cellulites of the scalp
4

Acne comedonicus, hidradenitis suppurativa, dissecting cellulites of the scalp
5

Acne conglobata, hidradenitis suppurativa, dissecting cellulites of the scalp
Q/Q(M)-482282 Report a Problem


The "follicular occlusion triad" consists of:
5

Acne conglobata, hidradenitis suppurativa, dissecting cellulites of the scalp
Acne conglobata is a sever variant of acne vulgaris characterized by large, often multiple comedones,
abscesses with sinus formation and inflammatory nodules. Hidradenitis suppurativa is a chronic condition
characterized by swollen, painful, inflamed lesions in the axillae, groin, and other parts of the body that
contain apocrine glands. Dissecting cellulites of the scalp, also known as perifolliculitis capitis abscedens
et Suffodiens of Hoffman, consists of deep inflammatory boggy nodules +/- sinus tracts on the occipital
scalp, most commonly in African American males.
Q/Q(M)-482282 Report a Problem

The most common association with the generalized lesions shown is:
1

Hodgkin's disease
2

Non-Hodgkin's lymphoma
3

Granulomatous mycosis fungoides
4

HIV infection
5

Diabetes mellitus
Q/Q(M)-481907 Report a Problem
91




The most common association with the generalized lesions shown is:
5

Diabetes mellitus
In a large retrospective study of 1383 patients, diabetes mellitus was diagnosed in 21% of patients with
generalized GA, compared with 9.7% of patients with localized GA. Othere choices also have been
reported to be associated with granuloma annulare. Dabski K, Winkelmann RK. Generalized granuloma
annulare: clinical and laboratory findings in 100 patients. J Am Acad Dermatol. 1989;20:39-47
Q/Q(M)-481907 Report a Problem


Which of the following statements is true regarding Morbihan's Disease?
1

It is often misdiagnosed as cellulitis
2

It presents with blepharitis, conjunctivitis, iritis, and keratitis
3

Histopathology reveals perifollicular and perivascular noncaseating epithelioid granulomas
4

It occurs around the mouth and/or nose and eyes and may be triggered by topical steroid use
5

It presents with large coalescent nodules and confluent draining sinuses occupying most of the
face
Q/Q(M)-480383 Report a Problem



Which of the following statements is true regarding Morbihan's Disease?
1

It is often misdiagnosed as cellulitis
Morbihan's Disease, also known as rosacea lymphedema or persistent edema of rosacea, presents with
hard, nonpitting edema. It is often misdiagnosed as cellulitis.
Q/Q(M)-480383 Report a Problem


The causative organism for mucocutaneous leishmaniasis is:
1

Leishmania aethiopica
2

Leishmania infantum
3

Leishmania brasiliensis
4

Leishmania tropica
92

5

Leishmania major
Q/Q(M)-482321 Report a Problem


The causative organism for mucocutaneous leishmaniasis is:
3

Leishmania brasiliensis
Mucocutaneous disease is most commonly caused by New World species, although Old World L
aethiopica has been reported to cause this syndrome. Infection by Leishmania viannia braziliensis may
lead to mucosal involvement in up to 10% of infections depending on the region in which it was
acquired. Initial infection is characterized by a persistent cutaneous lesion that eventually heals, although
as many as 30% of patients report no prior evidence of leishmaniasis. Several years later, oral and
respiratory mucosal involvement occurs, causing inflammation and mutilation of the nose, mouth,
oropharynx, and trachea.
Q/Q(M)-482321 Report a Problem

Which of the following treatments for acne inhibits RNA-dependent protein synthesis by binding to the
50s ribosomal subunit?
1

Erythromycin
2

Tetracycline
3

Trimethoprim-sulfamethoxazle
4

Benzoyl peroxide
5

Azeleic acid
Q/Q(M)-480377 Report a Problem



Which of the following treatments for acne inhibits RNA-dependent protein synthesis by binding to the
50s ribosomal subunit?
1

Erythromycin
Macrolide antibiotics inhibit RNA-dependent protein synthesis by binding to the 50s ribosomal subunit,
and include erythromycin and azithromycin. Tetracyclines inhibit RNA-dependent protein synthesis by
binding to the bacterial 30s ribosomal subunit. TMP-SMX inhibits bacterial folic acid synthesis. Benzoyl
peroxide is a bactericidal agent with direct oxidizing effects. Azeleic acid is a dicarboxylic acid that
inhibits tyrosinase; the mechanism of action against P. acnes is not completely understood.
Q/Q(M)-480377 Report a Problem


Which of the following HLA types is associated with psoriasis and predicts earlier onset and more severe
disease?
1

HLA-B13
2

HLA-B17
3

HLA-Bw57
93

4

HLA-Cw6
5

HLA-B27
Q/Q(M)-480385 Report a Problem


Which of the following HLA types is associated with psoriasis and predicts earlier onset and more severe
disease?
2

HLA-B17
Psoriasis has HLA associations with all of these HLA types. HLA-B17 is associated with earlier onset
and more serious disease.
Q/Q(M)-480385 Report a Problem


Which of the following is associated with Reiters syndrome:
1

Ulcerative colitis
2

Multiple sclerosis
3

HLA-Cw6
4

Asymmetric arthritis
5

Perioral dermatitis
Q/Q(M)-474339 Report a Problem

Which of the following is associated with Reiters syndrome:
4

Asymmetric arthritis
Reiters syndrome is a chronic inflammatory disease similar to psoriatic arthritis. Commonly, patients
present with either a peripheral, asymmetric arthritis, or a urethritis, or conjunctivitis. Not all of these
findings need be present. Patients are usually men with the HLA-B27 genotype. Other clinical findings
include fever, weight loss, keratitis, iritis, and cardiac disease. Skin lesions are often found on the palms
(keratoderma blenorrhagicum _ hyperkeratotic papules and plaques) or the penis (balanitis circinata).
Reiters follows an infection of either the genitourinary tract or the gastrointestinal tract.
Q/Q(M)-474339 Report a Problem



Menopausal flushing can be effectively treated with:
1

Nadolol
2

Tryptophan
3

Clonidine
4

Danazol
5

Tamoxifen
Q/Q(M)-480509 Report a Problem
94



Menopausal flushing can be effectively treated with:
3

Clonidine
Menopausal flushing occurs at menopause or perimenstrual when estrogen levels are low. Pharmacologic
menopause may be caused by drugs, including danazol, tamoxifen, clomiphene citrate, decapeptyl,
leuprolide, and 4-hydroxyandrostenedione. Treatment is with oral estrogen replacement or clonidine
hydrochloride 0.05mg bid. Nadolol is an effective treatment of emotional flushing.
Q/Q(M)-480509 Report a Problem

The most common non-specific cutaneous manifestation associated with this disease is:
1

Leukocytoclastic vasculitis
2

Ertythema multiforme
3

Acne vulgaris
4

Erythema nodosum
5

Lichen planus
Q/Q(M)-477752 Report a Problem


The most common non-specific cutaneous manifestation associated with this disease is:
4

Erythema nodosum
The most common non-specific cutaneous manifestation of sarcoidosis is erythema nodosum. The
clinical hallmark of leukocytoclastic vasculitis is palpable purpura. There are numerous causes of
erythema nodosum including drugs, infection, malignancy, and idiopathic. Lichen planus can be
associated with hepatitis C.
Q/Q(M)-477752 Report a Problem



Which auto-antibody correlates with this finding, shawl sign and an overall favorable prognosis in
Dermatomyositis?
1

Anti-Mi-2 antibody
2

Anti-Jo-1 antibody
3

Anti-SRB antibody
4

Anti-KU antibody
95

5

Anti-PL7 antibody
Q/Q(M)-480462 Report a Problem



Which auto-antibody correlates with this finding, shawl sign and an overall favorable prognosis in
Dermatomyositis?
1

Anti-Mi-2 antibody
Anti-Mi-2 antibodies correlate with shawl sign, cuticular changes (as seen in the image) and an overall
favorable prognosis. Anti-Jo-1 antibodies correlate with pulmonary fibrosis, Raynaud's and polyarthritis.
Anti-SRB antibodies correlate with cardiac disease and a poor prognosis. Anti-KU antibodies correlate
with sclerodermatomyositis. Anti-PL7 and Anti-PL12 antibodies (anti-synthetase antibodies) also
correlate with pulmonary disease.
Q/Q(M)-480462 Report a Problem



Which of the following is true regarding the components of amyloid?
1

Ground substance is present only in secondary cutaneous or tumor-associated amyloidosis
2

The amyloid P component is present in all forms
3

Protein AA is present in primary systemic amyloidosis
4

The amyloid present in secondary systemic amyloid does not lose its birefringence after treatment
with potassium permanganate
5

Protein AL loses its biregringence after treatment with potassium permanganate
Q/Q(M)-480501 Report a Problem


Which of the following is true regarding the components of amyloid?
2

The amyloid P component is present in all forms
All forms of amyloidosis contain amyloid P component and ground substance. The protein-derived
amyloid fibers differ among various forms of amyloid. Protein AL is found in primary systemic
amyloidosis. Protein AA is found in secondary systemic amyloidosis. Secondary systemic amyloid (AA)
loses its birefringence after treatment with potassium permanganate, but amyloid in primary and localized
forms do not.
96

Q/Q(M)-480501 Report a Problem


A patient with holocarboxylase synthetase deficiency is most at risk for developing a deficiency in:
1

Niacin
2

Pyridoxine
3

Biotin
4

Zinc
5

Essential fatty acids
Q/Q(M)-482162 Report a Problem

A patient with holocarboxylase synthetase deficiency is most at risk for developing a deficiency in:
3

Biotin
There are three autosomal recessive syndromes that can lead to biotin deficiency: holocarboxylase
synthetase deficiency, biotinidase deficiency, and an inability to transport biotin into cells. All of these
syndromes present with a dermatitis similar to that of zinc deficiency as well as neurologic abnormalities
Q/Q(M)-482162 Report a Problem

Which cellular organelle is affected in the partial lipodystrophies?
1

Nucleolus
2

Rough endoplasmic reticulum
3

Golgi apparatus
4

Cell membrane
5

Nuclear lamina
Q/Q(M)-482453 Report a Problem

Which cellular organelle is affected in the partial lipodystrophies?
5

Nuclear lamina
Both the congenital (Dunnigan - Type 1) and acquired (Barraquer-Simons) lipodystrophies are caused by
mutations involving the nuclear lamina of the cell. Dunnigan lipodystrophy is characterized by a
mutation in LMNA and Barraquer-Simons is caused by a LMNB2 mutation.
Q/Q(M)-482453 Report a Problem


Which of the following is the most common type of melanoma found in black patients?
1

Acral lentiginous melanoma
2

Superficial spreading melanoma
3

Nodular melanoma
97

4

Amelanotic melanoma
5

Lentigo maligna melanoma
Q/Q(M)-482819 Report a Problem


Which of the following is the most common type of melanoma found in black patients?
1

Acral lentiginous melanoma
Overall, superficial spreading melanomas are most frequently seen. However, acral lentiginous
melanoma is most frequently seen in patients with darker skin types. It is seen on the palms, soles, and
nail units.
Q/Q(M)-482819 Report a Problem


A premature infant who is being weaned off breast milk develops vesicobullous and eczematous skin
lesions and diarrhea. Which of the following is not another classic precipitant for this condition
1

Parenteral nutrition
2

Stress (i.e. infection)
3

Diets with mainly cereal grains
4

Liver disease
5

Alcoholism
Q/Q(M)-482163 Report a Problem


A premature infant who is being weaned off breast milk develops vesicobullous and eczematous skin
lesions and diarrhea. Which of the following is not another classic precipitant for this condition
4

Liver disease
Zinc deficiency can be seen in premature or term infants being weaned off breast milk, which is usually
high in zinc content, as well as in parenteral nutrition use, alcoholism because of poor nutritional intake,
malabsorption, IBD, diets high in grains containing phytate which binds zinc, and metabolic stress
Q/Q(M)-482163 Report a Problem


What is the most likely diagnosis?
1

Dermatofibromasarcoma protuberans
2

Dermatofibroma
3

Lobomycosis
4

Blastomycosis
5

Keloid
Q/Q(M)-476878 Report a Problem
98





What is the most likely diagnosis?
5

Keloid
Keloids are dense overgrowths of fibrous tissue that generally form a response to tissue injury.
Lobomycosis and dermatofibromasarcoma protuberans may mimic keloids.
Q/Q(M)-476878 Report a Problem


Macroglossia can present in all of the following disorders except
1

Primary systemic amyloidosis
2

Down's syndrome
3

Mucopolysaccharidoses
4

Cretinism
5

Behchets disease
Q/Q(M)-481909 Report a Problem


Macroglossia can present in all of the following disorders except
5

Behchets disease
Macroglossia is not a feature of Behchets disease. Macroglossia, a classic feature that occurs in about
20% of patients with primary systemic amyloidosis due to deposition of amyloid in the tongue leading to
firm and enlarged tongue. These patients can also have hemorrhagic papules, plaques and blisters on its
surface. Macroglossia is seen in many diseases and syndromes including Down's syndrome,
mucopolysaccharoidoses, cretinism, hypothyroidism, lipoid proteinosis and Beckwith-Wiederman
Syndrome.
Q/Q(M)-481909 Report a Problem

A patient with this cutanous finding along with peri-ocular erythema, deltoid weakness and pulmonary
disease is most likely to have which laboratory finding?
1

Anti-Jo-1 antibodies against histidyl-tRNA synthetase
99

2

Anti-Jo-1 antibodies against nuclear helicase
3

c-ANCA against proteinase-3
4

Lupus anticoagulant
5

Anti-histone antibodies
Q/Q(M)-480428 Report a Problem


A patient with this cutanous finding along with peri-ocular erythema, deltoid weakness and pulmonary
disease is most likely to have which laboratory finding?
1

Anti-Jo-1 antibodies against histidyl-tRNA synthetase
Anti-Jo-1 antibodies against histidyl-tRNA synthetase are described in patients with dermatomyositis and
pulmonary disease. Anti-Mi-2 antibodies against nuclear helicase are described in classic DM with a
good prognosis. Patients with Wegener's disease display c-ANCA antibodies against proteinase-3 in a
cytoplasmic pattern. Antiphospholipid antibodies are also known as anticardiolipin antibodies and lupus
anticoagulant. They can cause a false positive VDRL. Clinical features include livedo reticularis,
thrombotic events and spontaneous abortions. Anti-histone antibodies are seen in drug-induced lupus.
Q/Q(M)-480428 Report a Problem


What name is commonly used to describe this condition of nodular elastosis with cysts and comedones?
1

Actinic granuloma
2

Chlorance
3

Cutis rhomboidalis nuchae
4

Colloid milium
5

Favre-Racouchot
Q/Q(M)-476435 Report a Problem
100


What name is commonly used to describe this condition of nodular elastosis with cysts and comedones?
5

Favre-Racouchot
Favre-Racouchot syndrome is a condition that generally affects elderly white males with history of
chronic sun exposure and smoking. It is characterized by multiple open comedones on the temples and
forehead with a background of actinincally damaged skin.
Q/Q(M)-476435 Report a Problem


What is the best muscle to biopsy in dermatomyositis?
1

triceps
2

biceps
3

quadriceps
4

gluteus maximus
5

deltoid
Q/Q(M)-482447 Report a Problem


What is the best muscle to biopsy in dermatomyositis?
1

triceps
The triceps muscle is involved early in the disease; therefore, it has the highest yield in a muscle biopsy
looking for the changes seen in dermatomyositis. Surgeons have traditionally biopsied the deltoid
mulscle, but it is not involved until late in the disease.
Q/Q(M)-482447 Report a Problem


Teenage boys with severe, eruptive cystic acne and fever, leukocytosis, myalgias and other constitutional
symptoms can have which of the following bony change?
1

Dimpling above the 5th MCP
2

Osteolytic lesions of the clavicle
3

Osteopoikilosis
101

4

Stippled epiphyses
5

Jaw cysts
Q/Q(M)-480182 Report a Problem

Teenage boys with severe, eruptive cystic acne and fever, leukocytosis, myalgias and other constitutional
symptoms can have which of the following bony change?
2

Osteolytic lesions of the clavicle
The syndrome described is that of acne fulminans. Osteolytic bone lesions may accompany this
syndrome, most commonly of the clavicle. Dimpling above the 5th MCP is a feature of Albrights
Hereditary Osteodystrophy. Osteopoikilosis is a feature of Buschke-Ollendorf syndrome. Stippled
epiphyses occurs in chondrodysplasia punctata. Jaw cysts are a feature of Gorlins syndrome.
Q/Q(M)-480182 Report a Problem

Which of the following drugs is bactericidal?
1

Clindamycin
2

Tetracycline
3

Trimethoprim
4

Penicillin
5

Chloramphenicol
Q/Q(M)-482796 Report a Problem


Which of the following drugs is bactericidal?
4

Penicillin
Other bactericidal drugs include bacitracin, monobactams, quinolones, vancomycin, and polymyxin B.
Clindamycin, tetracycline, trimethoprim, and chloramphenicol and bacteriostatic.
Q/Q(M)-482796 Report a Problem

Which allergen is the most likely cause of this eyelid dermatitis?
1

Mercaptobenzothiazole
2

Ethyleneurea melamine formaldehyde
3

Ethylenediamine dihydrochloride
4

Tosylonamide formaldehyde resin
5

Benzalkonium chloride
Q/Q(M)-476816 Report a Problem
102




Which allergen is the most likely cause of this eyelid dermatitis?
4

Tosylonamide formaldehyde resin
Tosylonomide formaldehyde resin is found nail polish and is a common cause of eyelid dermatitis in
women.
Q/Q(M)-476816 Report a Problem


A patient's biopsy is found to have curled hair shafts with hyperkeratotic plugs. Which of the following is
not an associated feature of this condition
1

Hypochondriasis
2

Gingivitis
3

Delayed wound healing
4

Nail changes
5

Tender nodules on the lower extremities
Q/Q(M)-482161 Report a Problem


A patient's biopsy is found to have curled hair shafts with hyperkeratotic plugs. Which of the following is
not an associated feature of this condition
4

Nail changes
The patient's biopsy demonstrates the corkscrew hairs of scurvy, or vitamin C deficiency, which is
characterized by the 4 "H"s: hemorrhagic signs (tender nodules on the lower extremities), hyperkeratosis
of hair follicle, hypochondriasis, and hematology abnormalities
Q/Q(M)-482161 Report a Problem


A 42 year-old woman has the findings pictured, as well as erythematous patches on the upper chest and
shoulders. No Raynaud's, polyarthritis, pulmonary, or cardiac involvement is noted. Which antibody
might you expect to be present in this patient?
1

Anti-SRB
2

Anti-Jo-1
3

Anti-Mi-2
4

Anti-Ku
103

5

Anti-La
Q/Q(M)-480519 Report a Problem



A 42 year-old woman has the findings pictured, as well as erythematous patches on the upper chest and
shoulders. No Raynaud's, polyarthritis, pulmonary, or cardiac involvement is noted. Which antibody
might you expect to be present in this patient?
3

Anti-Mi-2
This patient has dermatomyositis. Anti-Mi-2 antibodies in DM correlate with the presence of a shawl
sign, cuticular changes, and good prognosis. Anti-Jo-1 antibodies correlate with pulmonary fibrosis,
Raynaud's, and polyarthritis. Anti-SRP antibodies correlate with cardiac disease and poor prognosis.
Anti-Ku antibodies correlate with sclerodermatomyositis. Anti-La antibodies correlate with Sjogren's
syndrome.
Q/Q(M)-480519 Report a Problem

Which of the following is true regarding syndromes associated with amyloidosis?
1

Familial mediterranean fever involves AL protein
2

MEN IIa involves AA protein
3

The syndrome that presents with renal amyloidosis, fevers, limb pains, and deafness involves AA
protein
4

Familial amyloidotic polyneuropathy type IV involves mutations in apolipoprotein A-1
5

Familial amyloidotic polyneuropathy type III involves mutations in gelsolin
Q/Q(M)-480506 Report a Problem



Which of the following is true regarding syndromes associated with amyloidosis?
3

The syndrome that presents with renal amyloidosis, fevers, limb pains, and deafness involves
AA protein
Familial mediterranean fever and Muckle-Wells (renal amyloidosis, urticaria, fevers, limb pains, and
deafness) involve AA. MENA IIa involves keratin-derived amyloid. Familial amyloidotic
polyneuropathy (FAP) type III involves mutations in apolipoprotein A-1; FAP type IV involves
mutations in gelsolin.
Q/Q(M)-480506 Report a Problem


The diagnosis is:
104

1

Psoriasis
2

Alopecia areata
3

Onychomycosis
4

Lichen planus
5

Tetracycline-induced photo-onycholysis
Q/Q(M)-474888 Report a Problem



The diagnosis is:
1

Psoriasis
This slide depicts nail changes consistent with psoriasis, including onycholysis, subungual debris and
pits. Pits in alopecia areata are usually more regularly spaced than in psoriasis. Fungal culture of nail
clipping for histologic evidence of onychomycosis is warranted. The main feature of tetracycline-induced
photo-onycholysis is onycholysis, rather than chronic nail bed and matrix involvement with psoriasis.
Q/Q(M)-474888 Report a Problem

A 61 year old man presents with painful and pruritic unilateral vesicles on his lateral back. Patient
described pain for 1 week prior to development of the rash. Diagnosis was confirmed with a Tzank
smear. Which of the following is a frequently seen complication of this disease?
1

Blindness
2

Infertility
3

Postherpetic neuralgia
4

Renal failure
5

Hepatic insufficiency
Q/Q(M)-482787 Report a Problem

A 61 year old man presents with painful and pruritic unilateral vesicles on his lateral back. Patient
described pain for 1 week prior to development of the rash. Diagnosis was confirmed with a Tzank
smear. Which of the following is a frequently seen complication of this disease?
3

Postherpetic neuralgia
105

Postherpetic neuralgia (PHN) is a common complication seen in herpes zoster. Defined as the presence of
pain after skin lesions have healed. Occurs in 10-15% of patients but incidence may increase to 50% in
patients over the age of 60 who develop herpes zoster. In most cases PHN resolves within the first 12
months but may persist for years. While generally self-limited, antiretroviral therapy is recommended for
patients over the age of 50, those who are immunocompromised, or have ophthalmic zoster.
Q/Q(M)-482787 Report a Problem

A teenage girl presents with recurrent upper eyelid edema and upper vermillion and cutaneous lip
swelling, giving her the appearance of having a double upper lip. She reports that the swelling
eventually self-resolves over the course of several days. Additionally, she is noted to have an enlarged,
non-tender thyroid gland upon palpation. She is most likely to suffer from which of the following
conditions?
1

Ascher syndrome
2

Hereditary angioedema
3

Sarcoidosis
4

Airborne allergic contact dermatitis
5

Granulomatous cheilitis
Q/Q(M)-482837 Report a Problem

A teenage girl presents with recurrent upper eyelid edema and upper vermillion and cutaneous lip
swelling, giving her the appearance of having a double upper lip. She reports that the swelling
eventually self-resolves over the course of several days. Additionally, she is noted to have an enlarged,
non-tender thyroid gland upon palpation. She is most likely to suffer from which of the following
conditions?
1

Ascher syndrome
Ascher syndrome is a rare syndrome characterized by recurrent upper (and sometimes lower) eyelid
swelling as well as upper vermillion and cutaneous lip swelling. The latter often yields the appearance of
a double upper lip, while multiple episodes of the former results in stretching and redundancy of the
eyelid skin, causing blepharochalasis. Ascher syndrome is frequently seen in association with a euthryoid
(non-toxic) goiter, although up to 50% of patients may have no associated thyroid abnormality. The
pathogenesis of this condition remains unknown.
Q/Q(M)-482837 Report a Problem

Neutrophilic dermatoses en plaque is often associated with which of the following?
1

IgA monoclonal gammopathy and a benign course
2

IgA monoclonal gammopathy and a malignant course
3

Myeloma and B lymphomas and a benign course
4

IgG monoclonal gammopathy and a benign course
5

IgG monoclonal gammopathy and a malignant course
Q/Q(M)-480557 Report a Problem


Neutrophilic dermatoses en plaque is often associated with which of the following?
106

1

IgA monoclonal gammopathy and a benign course
Neutrophilic dermatoses en plaque are well defined, sharply demarcated intensely red plaques. Myeloma
and B cell lymphomas are rarely associated. These resolve with treatment of the gammopathy.
Q/Q(M)-480557 Report a Problem


A young girl presents with recurrent severe arthritis of the ankles. She also has a large ulcer on her leg
and severe acne. Which gene is mutated?
1

PSTPIP1
2

NOD2
3

CIAS1
4

AIRE
5

FOXP3
Q/Q(M)-482448 Report a Problem

A young girl presents with recurrent severe arthritis of the ankles. She also has a large ulcer on her leg
and severe acne. Which gene is mutated?
1

PSTPIP1
This patient has PAPA (pyogenic arthritis-pyoderma gangrenosum-acne) syndrome. The gene mutated is
PSTPIP1, also known as CD2 binding protein 1 (CD2BP1) which encodes proline-serine-threonine
phosphatase-interacting protein 1. NOD2 is the gene involved in Blau syndrome; CIAS1 with
CINCA/Muckle-Wells/Familial cold autoinflammatory syndromes; AIRE with APECED (autoimmune
polyendocrinopathy-candidiasis-ectodermal dystrophy); FOXP3 with IPEX (immune dysregulation,
polyendocrinopathy, enteropathy, X-linked) syndrome.
Q/Q(M)-482448 Report a Problem

Immunologic abnormalities in atopic dermatitis include:
1

Increased CD8 T-cell number and function
2

Increased secretion of IFN-gamma
3

Decreased expression of CD23 on B cells and monocytes
4

Increased secretion of IL-4
5

Accentuated DTH response
Q/Q(M)-480482 Report a Problem

Immunologic abnormalities in atopic dermatitis include:
4

Increased secretion of IL-4
Immunologic abnormalities in atopic dermatitis include increased synthesis of IgE, increased specific IgE
to multiple allergens, increased expression of CD23 (low-affinity IgE receptor) on B cells and
monocytes, increased basophil histamine release, impaired DTH response, decreased CD8
suppressor/cytotoxic T-cell number and function, increased secretion of IL-4 and IL-5 by TH2 cells, and
decreased secretion of IFN-gamma by TH1 cells.
107

Q/Q(M)-480482 Report a Problem


A 45-year old woman from the United States develops erythema and swelling around her eyelids and
symmetric weakness of her shoulders and hips. What malignancy is overrepresented in patients with this
condition compared to the general population?
1

Ovarian Cancer
2

Lung Cancer
3

Leukemia
4

Uterine Cancer
5

Breast Cancer
Q/Q(M)-482230 Report a Problem

A 45-year old woman from the United States develops erythema and swelling around her eyelids and
symmetric weakness of her shoulders and hips. What malignancy is overrepresented in patients with this
condition compared to the general population?
1

Ovarian Cancer
This patient has a heliotrope rash and proximal muscle weakness, consistent with the diagnosis of
dermatomyositis. 18-32% of patients with dermatomyositis will develop malignancy. The risk of
malignancy is greatest within the first 3 years of diagnosis. Ovarian cancer is overrepresented. In
Southeast Asia, nasopharyngeal cancer is overrepresented.
Q/Q(M)-482230 Report a Problem

A 30 year old pregnant female presented with this lesion that has rapidly appeared. She was in her third
trimester with normal pregnancy course to date. What is the most likely diagnosis?
1

Hemangioma
2

Glomus tumor
3

Bacillary angiomatosis
4

Pyogenic granuloma
5

Inflammed wart
Q/Q(M)-482887 Report a Problem

A 30 year old pregnant female presented with this lesion that has rapidly appeared. She was in her third
trimester with normal pregnancy course to date. What is the most likely diagnosis?
4

Pyogenic granuloma
It is one of several vascular lesions that can present on the skin during pregnancy. Other common
vascular lesions include palmar erythema, spider angiomas, varicose veins, cavernous hemangiomas,
glomus tumors, and hemorrhoids. Etiologic factors in development of vascular lesions during pregnancy
thought to be a combination of increased hormones and increased intravascular pressure. Pyogenic
granulomas may develop as a reaction to trauma. They often regress after delivery and treatment is not
usually required unless they cause excessive bleeding.
Q/Q(M)-482887 Report a Problem
108



An atypical finding in patients with Lofgren syndrome is?
1

Fever
2

Arthritis
3

Subcutaneous Tender Nodules
4

Acute Iritis
5

Beaded papules on nasal rim
Q/Q(M)-482231 Report a Problem


An atypical finding in patients with Lofgren syndrome is?
5

Beaded papules on nasal rim
Lofgren syndrome is a form of acute, self resolving, sarcoidosis manifested with hilar adenopathy, fever,
migrating polyarthritis, acute iritis, and erythema nodosum. In general these patients do not have other
manifestations of cutaneous saroidosis. Papules along the nasal rim are suggestive of Lupus Pernio
sarcoidosis and suggest a worse brognosis with bronchial involvement.
Q/Q(M)-482231 Report a Problem

What systemic condition is often associated with this disease?
1

Diabetes mellitus
2

Tobacco use
3

Alopecia areata
4

Coronary artery disease
5

Hepatitis C
Q/Q(M)-476505 Report a Problem



What systemic condition is often associated with this disease?
5

Hepatitis C
109

Oral lichen planus is often found in patients with hepatitis C. The association between cutaneous lichen
planus and hepatitis C is not clear.
Q/Q(M)-476505 Report a Problem


A patient with spontaneous occurrence of petechiae and purpura, particularly around the eyes, will also
most likely have which of the following?
1

Diffuse scaling of the scalp
2

Acne
3

Cheilits
4

Macroglossia
5

Uveitis
Q/Q(M)-480487 Report a Problem


A patient with spontaneous occurrence of petechiae and purpura, particularly around the eyes, will also
most likely have which of the following?
4

Macroglossia
This patient has primary systemic amyloidosis. Mucocutaneous findings can be seen in up to 40% of
patients. The surface of the tongue may be smooth and dry or covered with waxy papules and nodules.
Teeth indentations may be seen along the lateral borders. The most common cutaneous signs are
petechiae and purpura that occur spontaneous or after minor trauma ("pinch purpura" about the eyelids).
The most characteristic skin lesions are waxy, shiny, smooth papules and nodules which are usually
hemorrhagic or flesh-colored. Flexural areas are common sites. These patients may also present with
carpal tunnel syndrome, hepatomegaly, and edema.
Q/Q(M)-480487 Report a Problem

The risk of developing systemic involvement in a patent with nodular amyloidosis is:
1

1-4%
2

5-10%
3

25-30%
4

50-60%
5

>95%
Q/Q(M)-477422 Report a Problem


The risk of developing systemic involvement in a patent with nodular amyloidosis is:
2

5-10%
Nodular amyloidosis is a rare type of primary cutaneous amyloidosis made up of AL protein. The risk for
systemic involvement is 7%.
Q/Q(M)-477422 Report a Problem
110



The most common systemic manifestations of systemic sclerosis are:
1

Gastrointestinal
2

Cardiovascular
3

Pulmonary
4

Renal
5

Neurologic
Q/Q(M)-480518 Report a Problem


The most common systemic manifestations of systemic sclerosis are:
1

Gastrointestinal
Esophageal dysfunction is the most systemic finding in systemic sclerosis (>90%). Dysphagia results
from decreased peristalsis and may occur before skin findings are seen. Small intestinal involvement is
also common. Pulmonary fibrosis, myocardial fibrosis (seen in 50-70%), cardiac conduction defects,
heart failure, pericarditis with effusion, and renal disease with slowly progressive uremia may all be seen.
Q/Q(M)-480518 Report a Problem

What is the most photosensitive form of all cutaneous lupus subtypes?
1

Acute cutaneous lupus
2

Subacute cutaneous lupus
3

Classic discoid lupus
4

Lupus tumidus
5

Chiblain
Q/Q(M)-482871 Report a Problem

What is the most photosensitive form of all cutaneous lupus subtypes?
4

Lupus tumidus
It is a rare entity. Patients complaint of edematous and erythematous plaques, usually on the trunk. The
lesions generally respond to antimalarials.
Q/Q(M)-482871 Report a Problem


A 16 year old developmentally normal male presents to his pediatrician intermittent vague epidodes of
hand and feet paresthesias and non specific episode of GI distress. He is referred to you to evaluate
numerous punctate to 5 slightly verrucous, deep-red to blue-black papules distributed diffusely on his
trunk in a bathing suit distribution. Polarization microscopy of the sediment of his urine demonstrates
birefringent lipid globules (ie, renal tubular epithelial cells or cell fragments with lipid inclusions) with
the characteristic Maltese cross configuration. What is the classic ocular finding in this disorder?
1

Pseudo-herpetic corneal ulcerations
111

2

Brushfield spots
3

Hyperpigmentation of the retinal epithelium
4

Corneal opacities
5

Lester irides
Q/Q(M)-481633 Report a Problem



A 16 year old developmentally normal male presents to his pediatrician intermittent vague epidodes of
hand and feet paresthesias and non specific episode of GI distress. He is referred to you to evaluate
numerous punctate to 5 slightly verrucous, deep-red to blue-black papules distributed diffusely on his
trunk in a bathing suit distribution. Polarization microscopy of the sediment of his urine demonstrates
birefringent lipid globules (ie, renal tubular epithelial cells or cell fragments with lipid inclusions) with
the characteristic Maltese cross configuration. What is the classic ocular finding in this disorder?
4

Corneal opacities
Whorled corneal opacities are classically seen in Fabry's disease. Pseudo-herpetic ulcerations are seen in
Richner-Hanhart. Brushfield spots can be seen in Down's syndrome. Congenital hyperpigmentation of the
retinal epithelium is seen in Gardner's syndrome. Lester irides are seen in Nail-Patella syndrome.
Q/Q(M)-481633 Report a Problem



Pyoderma gangrenosum is most commonly reported with which malignancy?
1

Leukemia
2

Medullary thyroid carcinoma
3

Lung carcinoma
4

Breast carcinoma
5

Prostate carcinoma
Q/Q(M)-475865 Report a Problem

Pyoderma gangrenosum is most commonly reported with which malignancy?
1

Leukemia
Pyoderma gangrenosum, (PG), is an uncommon, ulcerative inflammatory skin condition characterized by
boggy ulcerations with undermined borders preceded by the breakdown of a painful nodule or pustule.
These lesions characteristically enlarge progressively over time and display marked tenderness. The
diagnosis of PG is a diagnosis of exclusion and infectious etiologies should be excluded. Pyoderma
gangrenosum can arise in the absence of an underlying disorder or it can be seen in association with
underlying systemic conditions. These conditions include inflammatory bowel disease such as ulcerative
colitis and Crohn's disease, polyarthritis, chronic active hepatitis and Behcet's disease. Pyoderma
gangrenosum has also been associated with an underlying paraproteinemia, mostly of the IgA type,
although IgM and IgG types have also been described. In addition, some patients have myeloma at
presentation or develop it subsequent to their diagnosis of PG. Pyoderma gangrenosum has also been
described in association with myelodysplasia, and in acute myeloblastic, myelomonocytic, and chronic
myeloid leukemia. Treatment of pyoderma gangrenosum should include therapy directed at the
112

underlying systemic disorder.
Q/Q(M)-475865 Report a Problem


A patient presents with lesion that can be herniated into the skin on palpation. What is thought to be
important in the pathogenesis of this condition?
1

Calcification of the elastic fibers
2

Formation of excessive collagen
3

Focal loss of elastic tissue
4

Atrophy of the epidermis
5

Inflammation of the panniculus
Q/Q(M)-476884 Report a Problem



A patient presents with lesion that can be herniated into the skin on palpation. What is thought to be
important in the pathogenesis of this condition?
3

Focal loss of elastic tissue
Anetoderma is a benign condition caused by focal loss of elastic tissue. Primary or idiopathic anetoderma
originates from previously healthy skin with unknown pathogenesis. Various ocular, bony, cardiac, and
other abnormalities have been reported with primary anetoderma. Secondary anetoderma occurs after the
resolution of an inflammatory disease of the skin.
Q/Q(M)-476884 Report a Problem

Kidney disease in Henoch-Schonlein Purpura may be predicted by:
1

Limited skin involvement
2

Spread of purpura to the upper trunk
3

Synovial involvement
4

Bullous lesions
5

Facial edema
Q/Q(M)-480533 Report a Problem
113



Kidney disease in Henoch-Schonlein Purpura may be predicted by:
2

Spread of purpura to the upper trunk
Henoch-Schonlein Purpura occurs mostly in children. There is an antecedent URI in 75% of cases. HSP
involves the skin, synovia, GI tract, and kidneys. Long-term morbidity results from renal disease, which
is predicted by the spread of purpura to the upper trunk. Skin lesions of adults show blisters and necrosis.
Q/Q(M)-480533 Report a Problem

You prescribe doxycyline to a 22 year-old woman with acne. Your patient takes an oral contraceptive to
prevent pregnancy. Your patient read in a magazine that the doxycycline may decrease the efficacy of her
contraceptive, and asks you about this. Which of the following antibiotics has been definitively shown to
reduce contraceptive efficacy?
1

Tetracycline
2

Minocycline
3

Azithromycin
4

TMP-SMX
5

Rifampin
Q/Q(M)-480379 Report a Problem

You prescribe doxycyline to a 22 year-old woman with acne. Your patient takes an oral contraceptive to
prevent pregnancy. Your patient read in a magazine that the doxycycline may decrease the efficacy of her
contraceptive, and asks you about this. Which of the following antibiotics has been definitively shown to
reduce contraceptive efficacy?
5

Rifampin
Theoretically, decreased enterohepatic absorption of hormones due to altered gut flora could decrease
contraceptive efficacy. However, this theory has not been borne out in studies. Only rifampin, which is a
potent hepatic microenzyme inducer, has been definitively shown to reduce contraceptive efficacy.
Q/Q(M)-480379 Report a Problem

Which of the following is correct about eosinophilic folliculitis?
1

painful
2

only seen in adults
3

classified as an AIDS-defining illness
4

more common in females
5

associated with P. acnes infection of hair follicles
Q/Q(M)-482530 Report a Problem

Which of the following is correct about eosinophilic folliculitis?
3

classified as an AIDS-defining illness
Eosinophilic pustular folliculitis is a non-infectious eosinophilic infiltrate of the hair follicle. It is
characterized by intense pruritus. The 3 variants of eosinophilic folliculitis include classic eosinophilic
114

pustular folliculitis, immunosuppression-associated (mostly HIV-related), and infancy-associated
eosinophilic folliculitis. The classic type (Ofujis) is more common in Japanese men. The
male-to-female ratio of eosinophilic folliculitis is 5:1. It is associated with immunosuppresion and has
been classified as an AIDS defining illness. Eosinophilic pustular folliculitis should be viewed as a
possible cutaneous sign of immunosuppression.
Q/Q(M)-482530 Report a Problem


Patients with Gottrons papules and periungual telangiectasias will likely have a positive serologic test
to which antibody?
1

Anti-SM
2

Scl-70
3

Anti-Ro
4

Anti-centromere
5

Anti- Jo-1
Q/Q(M)-477140 Report a Problem

Patients with Gottrons papules and periungual telangiectasias will likely have a positive serologic test
to which antibody?
5

Anti- Jo-1
Gottron's papules and periungual telangiectasias (Samitz's sign) are among the cutaneous features of
dermatomyositis, which also include heliotrope rash, photosensitive poikiloderma of the upper back
(shawl sign). Anti-Jo-1 antibody is present in 20-30% of these patients. The target antigen is anti-histidyl-
tRNA synthetase (Jo-1). The presence of this antibody corresponds to the development of pulmonary
disease.
Q/Q(M)-477140 Report a Problem


What type of amyloid is deposited into the skin of this pruritic disorder?
1

Amyloid AA
2

Amyloid AL
3

Keratin derived
4

Beta-2 microglobulin
5

Transthyretin
Q/Q(M)-476536 Report a Problem
115




What type of amyloid is deposited into the skin of this pruritic disorder?
3

Keratin derived
Lichen amyloidosis is a pruritic eruption that often occurs in areas of chronic rubbing. Clinically, it has a
rippled appearance. Keratin derived amyloid is the type deposited in the skin.
Q/Q(M)-476536 Report a Problem

A 48 year old African American woman presents with 1-3 cm red-brown plaques on the extremities,
fevers, bilateral knee pain, and tender nodules on bilateral shins. What is her presentation most consistent
with?
1

Lofgrens syndrome
2

Rheumatoid arthritis
3

Graves disease
4

Psoriatic arthritis
5

Gout
Q/Q(M)-482823 Report a Problem

A 48 year old African American woman presents with 1-3 cm red-brown plaques on the extremities,
fevers, bilateral knee pain, and tender nodules on bilateral shins. What is her presentation most consistent
with?
1

Lofgrens syndrome
This patient has Lofgrens syndrome, a variant of sarcoidosis, with erythema nodosum, hilar
adenopathy, fever, and arthritis. Rheumatoid arthritis, psoriatic arthritis, and gout result in joint pain, but
are not consistent with this patients presentation.
Q/Q(M)-482823 Report a Problem
Ganciclovir and valganciclovir would be most appropriate to treat which infection?
1

HSV-1
116

2

HSV-2
3

VZV
4

CMV
5

EBV
Q/Q(M)-482798 Report a Problem

Ganciclovir and valganciclovir would be most appropriate to treat which infection?
4

CMV
CMV is most susceptible to cidofovir, ganciclovir, and valganciclovir. HSV-1 and HSV-2 are more
susceptible to acyclovir and famciclovir. Foscarnet can be used to treat resistant strains of CMV, HSV-1
and HSV-2.
Q/Q(M)-482798 Report a Problem


First line therapy for subcorneal pustular dermatosis is:
1

Dapsone
2

Tetracycline
3

Vitamin E
4

Topical retinoids
5

Topical steroids
Q/Q(M)-479619 Report a Problem


First line therapy for subcorneal pustular dermatosis is:
1

Dapsone
First line therapy for subcorneal pustular dermatosis or Sneddon-Wilkinson disease is dapsone 50-
200mg/day. Sulfapyridine, acitretin, PUVA, NB UVB, topical and systemic steroids, vitamin E and
antibiotics have been reported to be helpful also.
Q/Q(M)-479619 Report a Problem


The most common cause of internal manifestation of scleroderma is:
1

Esophageal dysmotility
2

Sepsis
3

Pulmonary hypertension
4

Acute renal failure
5

Raynaud's phenomenon
Q/Q(M)-475861 Report a Problem

The most common cause of internal manifestation of scleroderma is:
117

1

Esophageal dysmotility
Extracutaneous manifestations of scleroderma include esophageal dysmotility, pulmonary fibrosis with
resultant pulmonary hypertension, cardiac involvement with conduction defects, pericarditis or heart
failure, and renal involvement with acute renal failure. Esophageal dysmotility is the most common
systemic manifestation with up to 90% of systemic sclerosis patients affected. Dysphagia can precede
cutaneous involvement thereby giving rise to the term systemic sclerosis sine scleroderma. Dysmotility is
manifested by reduced peristalsis, especially in the lower two-thirds of the esophagus. This can be
demonstrated on a radiologic or manometric study.
Q/Q(M)-475861 Report a Problem

Xanthoma striata palmaris are diagnostic of:
1

Familial hypertriglyceridemia (type IV)
2

Familial dysbetalipoproteinemia (type III)
3

Familial lipoprotein lipase deficiency (AR)
4

Familial lipoprotein lipase deficiency (AD)
5

Apoprotein CII deficiency
Q/Q(M)-480550 Report a Problem

Xanthoma striata palmaris are diagnostic of:
2

Familial dysbetalipoproteinemia (type III)
Xanthoma striata palmaris are diagnostic of type III dysbetalipoproteinemia (AR; broad beta disease).
This condition also presents with palmar, planar, tendinous, tuberous, eruptive, and intertriginous
xanthomas, increased IDL, and atherosclerosis. It is associated with diabetes, gout, and obesity.
Q/Q(M)-480550 Report a Problem


What is the major constituent of sebaceous gland lipid?
1

Triglyceride
2

Wax esters
3

Squalene
4

Free cholesterol
5

Cholesterol esters
Q/Q(M)-480478 Report a Problem

What is the major constituent of sebaceous gland lipid?
1

Triglyceride
Triglyceride is the major constituent of sebaceous lipid, accounting for over 50% of the lipid. Wax esters
constitute about 25% of sebaceous lipid and Squalene accounts for about 15%. The remainder is free
cholesterol and cholesterol esters.
Q/Q(M)-480478 Report a Problem
118



Which of the following is associated with dermatitis herpetiformis?
1

Gluten-sensitive enteropathy
2

Inflammatory bowel disease
3

Autoimmune hepatitis
4

Herpes labialis
5

Rheumatoid arthritis
Q/Q(M)-482807 Report a Problem


Which of the following is associated with dermatitis herpetiformis?
1

Gluten-sensitive enteropathy
Gluten-sensitive enteropathy or Celiac disease is demonstrated on small bowel biopsy of all patients with
dermatitis herpetiformis. However, most of these patients do not demonstrate symptoms of
gastrointestinal disease. Dermatitis herpetiformis (DH) is not associated with inflammatory bowel
disease, autoimmune hepatitis, herpes labialis, or rheumatoid arthritis.
Q/Q(M)-482807 Report a Problem

Which autoantibodies are associated with an increased risk of malignancy in dermatomyositis?
1

anti-SRP
2

anti-Mi2
3

Anti-Jo1
4

anti-155/140
5

anti-PL-7
Q/Q(M)-482446 Report a Problem


Which autoantibodies are associated with an increased risk of malignancy in dermatomyositis?
4

anti-155/140
Anti-155/140 has been associated with an increased risk of malignancy in dermatomyositis patients.
Anti-SRP is associated with fulminant dermatomyositis/polymyositis and cardiac involvement; anti-Mi2
is associated with the shawl sign, periungual telangiectasias, cuticular overgrowth, and Gottrons papules;
anti-Jo1 and anti-PL-7 are associated with antisynthetase syndrome and interstitial lung disease.
Q/Q(M)-482446 Report a Problem


Which of the following is true regarding relapsing polychondritis?
1

Involvement is often bilateral
2

Pathogenic antibodies have not yet been identified
3

The course is chronic, yet mortality is low
119

4

Both sexes are equally affected
5

Migratory arthralgias are uncommonly seen
Q/Q(M)-480530 Report a Problem


Which of the following is true regarding relapsing polychondritis?
4

Both sexes are equally affected
Relapsing polychondritis involves intermittent episodes of inflammation of the articular and nonarticular
cartilage, resulting in chondrolysis, dystrophy, and atrophy of the cartilage. Both sexes are equally
affected. IgG anti-type II collagen antibodies are pathogenic, with titers corresponding to disease activity
found in up to 50% of patients with relapsing polychondritis (and in only 15% of those with RA).
Involvement is often unilateral. Migratory arthralgias are seen in 50-80%. The course is unpredictable,
often chronic and variable with episodic flares. Relapsing polychondritis causes death in 1/3 of patients
secondary to airway collapse, cardiovascular complications, and infection (secondary to systemic
steroids).
Q/Q(M)-480530 Report a Problem


Anti-Jo-1 antibodies in patients with Dermatomyositis are associated with:
1

Pulmonary fibrosis
2

Cardiac disease
3

Photosensitivity
4

Calcinosis
5

Heliotrope rash
Q/Q(M)-480463 Report a Problem

Anti-Jo-1 antibodies in patients with Dermatomyositis are associated with:
1

Pulmonary fibrosis
Anti-Jo-1 (histidyl TRNA syntetase) antibodies are seen in a minority of patients with Dermatomyositis
and are associated with pulmonary fibrosis. Anti-Jo-1 Abs are also associated with Raynaud's and
polyarthritis. Treatment of dermatomyositis may include systemic corticosteroids, antimalarials,
methotrexate, azatioprine and photoprotection.
Q/Q(M)-480463 Report a Problem


Which of the following dermatoses occurs at the latest stage of pregnancy?
1

Darkening of nevi
2

Linea nigra
3

Melasma
4

Areolar hyperpigmentation
5

Psoriasis exacerbation
120

Q/Q(M)-477320 Report a Problem

Which of the following dermatoses occurs at the latest stage of pregnancy?
2

Linea nigra
Linear nigra is one the latest skin changes in pregnancy. It is a hyperpigmented, linear streak which
exctends from the pubic symphysis to the xiphoid process. It usually appears from the 20th week.
Q/Q(M)-477320 Report a Problem

The diagnosis is
1

Psoriasis:
2

Alopecia areata
3

Onychomycosis
4

Lichen planus
5

Tetracycline-induced photo-onycholysis
Q/Q(M)-474887 Report a Problem



The diagnosis is
4

Lichen planus
Lichen planus-related nail changes seen here include thinning of the nail plate with onychorrhexis,
ridging and pterygium formation.
Q/Q(M)-474887 Report a Problem


What marker is specific for acral melanocytic lesions?
1

b-raf
2

c-myc
3

c-kit
4

Ras
121

5

p53
Q/Q(M)-482848 Report a Problem


What marker is specific for acral melanocytic lesions?
3

c-kit
C-kit staining is specific for acral melanocytic lesions. Ras is found in all melanocytic lesions. P53
mutations are found in actinic keratoses, SCCIS, and SCC.
Q/Q(M)-482848 Report a Problem


Which of the following MOST favors a diagnosis of Sweets over atypical (bullous) pyoderma
gangrenosum?
1

Recurrence of lesions
2

Presence of lesions on the H/N and hands
3

Association with infections
4

Presence of constitutional symptoms
5

Presence of hemorrhagic bulla and ulcerations
Q/Q(M)-482846 Report a Problem


Which of the following MOST favors a diagnosis of Sweets over atypical (bullous) pyoderma
gangrenosum?
4

Presence of constitutional symptoms
Both Sweets syndrome and atypical pyoderma gangrenosum fall within the category of neutrophilic
dermatoses with bullous and ulcerated lesions. Both favor the head and neck and extremities. Both can
recur, and both demonstrate a heavy neutrophilic infiltrate on histopathology. Both are associated with
infections and hematologic disorders, though atypical PG is more likely associated with hematologic
malignancy. The presence of constitutional symptoms such as fever, arthralgia, and arthritis are strongly
associated with Sweets and can assist in differentiation between these entities.
Q/Q(M)-482846 Report a Problem

Which of the following medications would likely exacerbate your patient's psoriasis?
1

Hydrochlorothiazide
2

Lisinopril
3

Amlodipine
4

Metroprolol
5

Diltiazem
Q/Q(M)-482149 Report a Problem
122



Which of the following medications would likely exacerbate your patient's psoriasis?
4

Metroprolol
Several drugs have been incriminated as inducers of psoriasis, in particular and most notably lithium,
beta-blockers, antimalarials, and interferon. More recent additions include terbinafine, calcium channel
blockers, captopril, glyburide, and lipid-lowering drugs such as gemfibrozil
Q/Q(M)-482149 Report a Problem


Which of the following is a manifestation of psoriasis of the nail matrix?
1

Splinter hemorrhages
2

Oil spots
3

Subungal hyperkeratosis
4

Onycholysis
5

Pits
Q/Q(M)-482150 Report a Problem



Which of the following is a manifestation of psoriasis of the nail matrix?
5

Pits
Psoriatic nail changes may be of nail matrix or bed in origen. Pits are the most common findings, while
splinter hemorrhages are the least. Psoriatic nail changes involving the matrix include pits (representing
focal psoriasis of the proximal nail matrix) and leukonychia.
Q/Q(M)-482150 Report a Problem


Which of the following is most characteristic of seborrheic dermatitis?
1

Greasy, yellow, scaly plaques on the central face and chest
2

Well defined, round, scaly, pink plaques on the knees and elbows
3

Stuck on waxy papules and plaques
4

Pink papules and telangiectasias on the cheeks
5

Heliotrope violaceous periocular patches
123

Q/Q(M)-482810 Report a Problem

Which of the following is most characteristic of seborrheic dermatitis?
1

Greasy, yellow, scaly plaques on the central face and chest
Seborrheic dermatitis is characterized by greasy, yellow, scaly plaques involving the scalp, central face,
and chest. It can be treated with medications such as ketoconazole or selenium sulfide. Psoriasis is
defined by well defined round, scaly, pink plaques on the knees and elbows. Seborrheic keratoses are
defined by stuck on waxy papules and plaques. Rosacea may be characterized by pink papules and
telangiectasias on the cheeks. Dermatomyositis is characterized by heliotrope violaceous periocular
patches.
Q/Q(M)-482810 Report a Problem


A 24 year old female suffers periodic attacks of urticaria and fever. Additionally, she has suffered from
worsening deafness and mild renal failure. Her father and grandfather had similar symptoms. What
abnormally deposited protein is driving her symptoms?
1

AA
2

AL
3

B amyloid protein
4

Keratin
5

Beta 2-microglobulin
Q/Q(M)-482758 Report a Problem

A 24 year old female suffers periodic attacks of urticaria and fever. Additionally, she has suffered from
worsening deafness and mild renal failure. Her father and grandfather had similar symptoms. What
abnormally deposited protein is driving her symptoms?
1

AA
Muckle Wells is an autosomal dominant disorder with periodic attacks of urticaria, fever, deafness, and
renal amyloidosis. The implicated protein is AA, which is derived from SAA, an acute phase reactant.
Q/Q(M)-482758 Report a Problem

Which of the following cytokines is associated with follicular plugging and microcomedo formation?
1

IL-4
2

IL-13
3

TNF-
4

IL-1
5

TGF-ooo
Q/Q(M)-474302 Report a Problem

Which of the following cytokines is associated with follicular plugging and microcomedo formation?
4

IL-1
124

IL-1 is a pro-inflammatory cytokine that has been associated with follicular plugging and microcomedo
formation.
Q/Q(M)-474302 Report a Problem

A 40 year old women presents to the clinic with multiple pustules in annular and serpiginous patterns on
the abdomen, axillae and groin. Histopathology reveals pustules below the stratum corneum with many
neutrophils and without any acantholysis. What is the diagnosis?
1

Sneddon-Wilkinson disease
2

Reiter's syndrome
3

SAPHO syndrome
4

Transient pustular melanosis
5

Pthirus pubis
Q/Q(M)-482290 Report a Problem

A 40 year old women presents to the clinic with multiple pustules in annular and serpiginous patterns on
the abdomen, axillae and groin. Histopathology reveals pustules below the stratum corneum with many
neutrophils and without any acantholysis. What is the diagnosis?
1

Sneddon-Wilkinson disease
Sneddon- Wilkinson disease, also know as subcorneal pustular dermatosis classically presents with what
has been described in this question. SAPHO syndrome is a pneumonic which stands for synovitis, acne
conglobata or fulminans, pustular psoriasis, hyperostosis and osteomyelitis. Transient pustualr melanosis
is seen in neonates. Pthirus pubis is pubic lice.
Q/Q(M)-482290 Report a Problem

A patient presents with recurrent genital and oral ulcerations and a diagnosis of posterior uveitis. What
HLA type is associated with the diagnosis you suspect?
1

HLA-B27
2

HLA-B51
3

HLA-DR3
4

HLA-Cw6
5

HLA-DR4
Q/Q(M)-480527 Report a Problem

A patient presents with recurrent genital and oral ulcerations and a diagnosis of posterior uveitis. What
HLA type is associated with the diagnosis you suspect?
2

HLA-B51
The patient has Behcet's disease. Behcet's disease is diagnosed based on recurrent oral ulceration (at least
3 times in a 12 month period) plus 2 of the following: recurrent genital ulceration, posterior uveitis, skin
lesions (erythema nodosum, pseudofolliculitis, papulopustular lesions, or acneiform nodules), and a
positive pathergy test. Behcet's disease is associated with HLA-B51.
125

Q/Q(M)-480527 Report a Problem

A patient presents to you with purpura after minor trauma. In addition, macroglossia with teeth
indentations. Which amyloid protein is associated with her condition?
1

AB2M
2

AA
3

AL
4

Altered keratinAB
5

AB
Q/Q(M)-482876 Report a Problem

A patient presents to you with purpura after minor trauma. In addition, macroglossia with teeth
indentations. Which amyloid protein is associated with her condition?
3

AL
AL protein is associated with primary systemic amyloid. AA is associated with secondary systemic
amyloid. AB2M is associated with hemodialysis. AB is associated with alzheimers disease. Altered
keratin is associated with lichen and macular amyloid.
Q/Q(M)-482876 Report a Problem


One of your acne patients has been treated with doxycycline for several months and develops culture
positive gram negative folliculitis. What is the next appropriate therapy
1

Tetracycline
2

Bactrim
3

Isotretinoin
4

Ceftriaxone
5

Cefepime
Q/Q(M)-482159 Report a Problem

One of your acne patients has been treated with doxycycline for several months and develops culture
positive gram negative folliculitis. What is the next appropriate therapy
3

Isotretinoin
Gram negative folliculitis may occur after prolonged systemic antibiotic use for acne vulgaris. It should
be suspected in patients who develop a sudden acneiform eruption after having been stable for some time.
The treatment of choice is isotretinoin.
Q/Q(M)-482159 Report a Problem


Which of the following is true regarding neonatal lupus erythematosus?
1

Most cases involve boys
2

Lesions generally resolve spontaneously by 6 months, healing with scarring
126

3

Photosensitivity is generally not a feature
4

75% of mothers have symptomatic systemic lupus erythematous at the time of delivery
5

Congenital heart block may be the only manifestation of the disease
Q/Q(M)-480513 Report a Problem

Which of the following is true regarding neonatal lupus erythematosus?
5

Congenital heart block may be the only manifestation of the disease
Neonatal LE presents with annular scaling erythematous macules and plaques on the head and extremities
within the first few months of life in babies born to mothers with LE, rheumatic diseases, or other
connective tissue disorders. 50% of mothers are asymptomatic at delivery. Lesions resolve spontaneously
by 6 months, healing without scarring. Photosensitivity may be prominent. 75% of cases involve girls.
50% have congenital heart block, which is permanent, and may be the only manifestation of the disease.
Thrombocytopenia and hepatic disease are as frequent as cardiac disease.
Q/Q(M)-480513 Report a Problem


The joint most frequently affected in acne fulminans is the:
1

Elbow
2

Intervertebral
3

Distal interphalangeal joints of the hand
4

Sternoclavicular
5

Sacroiliac
Q/Q(M)-474328 Report a Problem

The joint most frequently affected in acne fulminans is the:
4

Sternoclavicular
Acne fulminans may be associated with lytic bone changes which are indicative of a sterile osteomyelitis.
The sternoclavicular and chest wall joints are most frequently affected.
Q/Q(M)-474328 Report a Problem
Which antibody is 93% specific for Sjogren's syndrome?
1

anti-Ro
2

anti-La
3

anti-fodrin
4

anti-Schirmer
5

RF
Q/Q(M)-480520 Report a Problem


Which antibody is 93% specific for Sjogren's syndrome?
127

3

anti-fodrin
Sjogren's syndrome is a triad of keratoconjunctivitis sicca, xerostomia, and rheumatoid arthritis. More
than 90% of patients are women. Labial salivary gland biopsy is useful for diagnosis, and the Schirmer
test for xerostomia detects diminished glandular secretions. Laboratory findings often include positive
cryoglobulins, anti-Ro, anti-La, and RF positivity. Antibodies to fodrin are 93% specific for this
diagnosis.
Q/Q(M)-480520 Report a Problem


What surgical dressing would you use if you wished to debride a wound, reduce pain, and provide a
cooling effect?
1

Alginates
2

Foams
3

Hydrocolloid
4

Hydrogel
5

Films
Q/Q(M)-482341 Report a Problem


What surgical dressing would you use if you wished to debride a wound, reduce pain, and provide a
cooling effect?
4

Hydrogel
Hydrogels are 70-90% water, decrease pain, provide a cooling effect, and are useful for dry and necrotic
wounds. Foams are suitable for use on light-to-medium exuding wounds. Hydrocolloids ensure the moist
wound environment, promote the formation of granulation tissue, and provide pain relief by covering
nerve endings with both gel and exudate. Large quantities of alginates are used each year to treat
exudating wounds, such as leg ulcers, pressure sores, and infected surgical wounds.
Q/Q(M)-482341 Report a Problem


Patients with psoriasis treated with cyclosporine should be monitored for:
1

Sicca symptoms
2

Hypermagnesemia
3

Alopecia
4

Acute interstitial pneumonitis
5

Hyperkalemia
Q/Q(M)-480390 Report a Problem

Patients with psoriasis treated with cyclosporine should be monitored for:
5

Hyperkalemia
Well-documented adverse effects and toxicities of cyclosporine include renal impairment, hypertension,
elevated triglycerides, hyperkalemia, hypomagnesemia, hepatotoxicity, hypertrichosis, and long-term
128

increased risk of malignancy.
Q/Q(M)-480390 Report a Problem



Which medication used to treat pruritus works by depleting substance P from nerve fibers?
1

Capsaicin
2

Doxepin
3

Diphenhydramine
4

Pramoxine
5

Gabapentin
Q/Q(M)-482820 Report a Problem


Which medication used to treat pruritus works by depleting substance P from nerve fibers?
1

Capsaicin
Capsaicin is an antipruritic and analgesic that works by depleting substance P. Repeated use leads to
decreased heat, pain, and itch sensations. Doxepin is a topical tricyclic antidepressant. Diphenhydramine
is an antihistamine. Pramoxine is a topical anesthetic. Gabapentin, commonly used to treat notalgia
paresthetica, has an unknown mechanism of action.
Q/Q(M)-482820 Report a Problem


This patient also has poliosis and deafness. What is the associated ophthalmologic finding?
1

Normal ophthalmologic exam
2

Retinal detachment
3

Lisch nodules
4

Corneal opacities
5

Granulomatous uveitis
Q/Q(M)-477845 Report a Problem


129


This patient also has poliosis and deafness. What is the associated ophthalmologic finding?
5

Granulomatous uveitis
This patient has Vogt-Koyanagi-Haradi syndrome (VKHS). VKHS is marked by bilateral granulomatous
uveitis, vitiligo, deafness, poliosis, and alopecia. This syndrome begins with a meningoencephalitic phase
with fever, malaise, headaches, nausea, and vomiting. Varying degrees of neurological impairment may
be present. Recovery is usual complete. The second phase is the ophthalmic-auditory phase during which
uveitis appears rapidly and can results in blindness. Treatment with systemic steroids may prevent
blindness. Lisch nodules are iris hamartomas which are seen in neurofibromatosis. Corneal opacities are
seen in X-linked ichthyosis. Retinal detachment can occur as a result of head trauma, as in shaken-baby
syndrome.
Q/Q(M)-477845 Report a Problem

Which of the following disorders is primarily associated with a monoclonal gammopathy of the IgA
type?
1

Scleromyxedema
2

Scleredema
3

Necrobiotic xanthogranuloma
4

Schnitzler's syndrome
5

Pyoderma gangrenosusm
Q/Q(M)-476547 Report a Problem


Which of the following disorders is primarily associated with a monoclonal gammopathy of the IgA
type?
5

Pyoderma gangrenosusm
Pyoderma gangrenosum is an destructive, inflammatory disease of the skin characterized by a painful
nodule or pustule that later forms a progressively enlarging ulcer. On pathology, there is a neutrophilic
infiltrate with leukocytoclasia. Diseases that have been associated with pyoderma gangrenosum include
inflammatory bowel disease, arthritis, an IgA monoclonal gammopathy (primarily), and myelodysplasia.
Q/Q(M)-476547 Report a Problem

Carcinoma of which of the following has been most associated with erythema gyratum repens
1

Lung cancer
2

Breast cancer
3

Colon cancer
4

Prostate cancer
5

Upper GI tract cancer
Q/Q(M)-482154 Report a Problem


Carcinoma of which of the following has been most associated with erythema gyratum repens
130

1

Lung cancer
80% of cases of erythema gyratum repens have been associated with underlying malignancy. Lung
cancer is the most common neoplasm. The skin eruption preceedes the detection of malignancy by an
average of 9 months
Q/Q(M)-482154 Report a Problem

Secondary systemic amyloidosis:
1

Classically involves the tongue and periorbital skin
2

Involves deposition of AL protein
3

Can involve deposition of beta 2-microglobulin in the setting of rheumatoid arthritis
4

Can be noted on biopsy of normal skin
5

Involves deposition of keratin-derived amyloid
Q/Q(M)-480503 Report a Problem

Secondary systemic amyloidosis:
4

Can be noted on biopsy of normal skin
Secondary systemic amyloidosis presents with deposition of amyloid in the adrenals, liver, spleen, and
kidney as a result of some chronic disease, such as TB, leprosy, Hodgkin's, Behcet's, rheumatoid arthritis,
ulcerative colitis, schistosomiasis, or syphilis. The skin is not involved. AA amyloid fibrils, derived from
SAA protein (an acute phase reactant) are deposited. AA is also seen in Muckle-Wells and familial
mediterranean fever. Biopsy of normal skin may be positive for perivascular amyloid. Dialysis-related
amyloidosis occurs via deposition of beta 2-microglobulin component altered by uremia, and resuls in
carpal tunnel syndrome, bone cysts, and spondyloarthropathy.
Q/Q(M)-480503 Report a Problem



Ulcerative colitis and crohn's disease are most commonly associated with which of the following:
1

Pyoderma gangrenosum
2

Erythema elevatum diutinum
3

Sweet's syndrome
4

IgA pemphigus
5

Sneddon-Wilkinson's disease
Q/Q(M)-480559 Report a Problem
131



Ulcerative colitis and crohn's disease are most commonly associated with which of the following:
3

Sweet's syndrome
Sweet's syndrome is also associated with acute myelogenous leukemia, colony-stimulating factor, all-
trans retinoic acid and tetracyclines. Flu-like symptoms, high-grade fever, malaise and peripheral
neutrophils accompany the cutaneous lesions. Steroids and dapsone are the treatments of choice.
Q/Q(M)-480559 Report a Problem


Cigarette smoking has been shown to:
5

Increase cutaneous vasoconstriction
Cigarette smoking accelerated photoaging. Solar elastosis in patients with significant smoking history is present
not only in the papillary dermis, but also the reticular dermis. It has also been shown to increase the incidence of
skin cancer, decrease the survival of surgical grafts, increase the severity of wrinkling.
Q/Q(M)-477174 Report a Problem

Which of the following medications is not associated with exacerbating this condition?
5

Propranolol
Acne or acneiform erruptions can be caused or exacerbated by corticosteroids, oral contraceptives, androgens,
ACTH, lithium, phenytoin, INH, and haloperidol
Q/Q(M)-482158 Report a Problem
132


A patient with a recent diagnosis of tuberculosis on treatment presents with a new photosensitive eruption on
his face, neck and upper chest. In addition, he has progressive diarrhea and depression. What is the etiology of
his symptoms?
2

Drug induced pellagra
This patient has pellagra secondary to isoniazid treatment. He has the photosensitive eruption and "Casal's
necklace" , in addition to diarrhea and depression. Other symptoms include the 3 D 's: dermatitis, diarrhea,
dementia. Other potential medications that may cause this constellations of symptoms include azathioprine and
5-FU.
Q/Q(M)-482488 Report a Problem
Kveim-Siltzback test is useful in the diagnosis of:
2

Sarcoidosis
Kveim-Siltzback test is for sarcoidosis, Montenegro-leishmanin test is for Leishmaniasis, and lepromin test or
histamine/methacholine sweat test can be used for leprosy. Kveim-Siltzback test is done by intradermal injection
of a suspension from granuloma-containing spleen, lymph node, or other tissue from a confirmed case of
sarcoidosis. A positive test is characterized by the formation of a papule at the site of injection within 4-6 weeks
which, on microscopic examination, exhibits non-necrotizing granulomas and the absence of foreign material.
This teast is rarely done nowdays because of the difficulties involved in preparation, standardization and
validation of the test material as well as significant variation in the sensitivity and specificity of test suspensions
obtained from different sources.
Q/Q(M)-481900 Report a Problem
Solid confluent palmoplantar keratosis, salmon-orange follicular papules and diffuse symmetrical involvement
with characteristic small islands of normal skin within affected areas are all clinical characteristics of what skin
disease?
133

5

Pityriasis rubra pilaris
PRP can affect any portion of the body, but commonly affect the extensor surfaces of the extremities as well as
the sides of the neck and trunk. It may progress with an almost erythrodermic type state with characteristic small
island of normal skin within affected areas. Phrynoderma refers to the toadskin like skin seen in some vitamin
deficiencies including vitamin A deficiency.
Q/Q(M)-482300 Report a Problem
A 35 year old man with a history of celiac disease presents with a beefy, red tongue, hyperpigmented palmar
creases, and premature grey hair. Which of the following statements are is correct?
2

This condition mimics folate deficiency
This condition is vitamin B12 deficiency and is characterized by glossitis and hyperpigmentation in sun exposed
areas and creases. Neurologic abnormaolities and megaloblastic anemia can be seen. The symptoms can mimic
folic acid deficiency. Riboflavin (B2) is associated with oral-ocular-genital syndrome. Carcinoid tumors as well as
azathioprine, 5-FU, and isoniazid are associated with niacin deficiency. Eating raw eggs is a risk factor for biotin
deficiency.
Q/Q(M)-482160 Report a Problem
A 3 month old presents with a diffuse vesiculobullous rash with copper colored macules on the palms and soles.
You ascertain from the history that his mother had a nonpainful erosion on her labia during pregnancy which
spontaneously resolved. What other symptoms would you expect this infant to have?
1

Pseudoparalysis of Parrot
Pseudoparalysis of Parrot is a sign of early congenital syphilis. Early congenital syphilis occurs before 2 years of
age, whereas late congenital syphilis generally occurs after 2 years of age. Higoumenakis sign, clutton joints,
mulberry molars, and saddle nose deformity are all signs of late congenital syphilis.
Q/Q(M)-482335 Report a Problem
Natural infection with which of the following infectious agents has been shown to mitigate atopic dermatitis?
1

Measles
Natural measles infection has been shown to ameliorate atopic dermatitis.
Q/Q(M)-474307 Report a Problem



Which immunoglobulins are seen in type III cryoglobulinemia?
134

5

Polyclonal IgM and polyclonal IgG
Type III (mixed) cryoglobulinemia consists of rheumatoid factors that are polyclonal IgM and IgG, complexed with
each other or with protein. Type II (mixed) cryoglobulinemia consists of monoclonal IgM rheumatoid factor
complexed with polyclonal IgG. Type I cryoglobulinemia consists of a single monoclonal immunoglobulin, usually
IgG or IgM, usually due to an underlying B-cell malignancy (myeloma or lymphoma).
Q/Q(M)-480547 Report a Problem
What is the most common extracutaneous manifestation of Sweet's syndrome?
1

Arthralgias
2

Conjunctivitis
3

Renal involvement
4

Sterile osteomyelitis
5

Fever
Q/Q(M)-482566 Report a Problem

What is the most common extracutaneous manifestation of Sweet's syndrome?
5

Fever
Extracutaneous manifestations of Sweet's syndrome occur in more than 75 percent of patients with Sweet s
syndrome. The most common is fever, followed by arthralgias, arthritis, or myalgias. Other less frequent
manifestations include conjunctivitis, episcleritis, oral aphthae- like lesions, cough, dyspnea, pleuritis, and sterile
multifocal osteomyelitis. Cardiac, renal, hepatic, intestinal, or neurologic manifestations are rare.
Q/Q(M)-482566 Report a Problem
What is the most common cause of death in malignant atrophic papulosis Degos disease?
1

Cutaneous infection
2

Cerebrovascular accident
3

Myocardial ischemia
4

Bowel infarction
5

Pulmonary hemorrhage
135

Q/Q(M)-482681 Report a Problem


What is the most common cause of death in malignant atrophic papulosis Degos disease?
4

Bowel infarction
Degos disease is an obliterative arteritis clinically presenting with pink-red papules that later develop atrophic,
porcelain-white centers. Death is usually secondary to Infarction of the bowel and perforation. Less commonly,
death results from cerebral infarction.
Q/Q(M)-482681 Report a Problem


A 16 year old developmentally normal male presents to his pediatrician intermittent vague epidodes of hand and
feet paresthesias and non specific episode of GI distress. He is referred to you to evaluate numerous punctate to
5 slightly verrucous, deep-red to blue-black papules distributed diffusely on his trunk in a bathing suit
distribution. Polarization microscopy of the sediment of his urine demonstrates birefringent lipid globules (ie,
renal tubular epithelial cells or cell fragments with lipid inclusions) with the characteristic Maltese cross
configuration. How is this disorder inherited?
1

Autosomal dominant
2

Autosomal recessive
3

X-linked dominant
4

X-linked recessive
5

Not an inherited disorder
Q/Q(M)-481632 Report a Problem


A 16 year old developmentally normal male presents to his pediatrician intermittent vague epidodes of hand and
feet paresthesias and non specific episode of GI distress. He is referred to you to evaluate numerous punctate to
5 slightly verrucous, deep-red to blue-black papules distributed diffusely on his trunk in a bathing suit
distribution. Polarization microscopy of the sediment of his urine demonstrates birefringent lipid globules (ie,
renal tubular epithelial cells or cell fragments with lipid inclusions) with the characteristic Maltese cross
configuration. How is this disorder inherited?
4

X-linked recessive
136

Fabry's disease is inherited is x-linked recessive. Female carriers often exhibit mild forms of this disorder. It can
be establish that a patient has FD by searching for low activity of alpha-galactosyl A in plasma, leukocytes,
cultured skin fibroblasts, or dried blood spots on filter paper. Because of the Lyon effect, enzymatic detection of
carriers can be misleading; thus, specific genetic analysis can be helpful in making the diagnosis.
Q/Q(M)-481632 Report a Problem

Twenty-nail dystrophy, nail plate splitting and pterygium formation are nail changes seen in:
1

Lichen Planus
2

Darier's disease
3

Psoriasis
4

Scleroderma
5

Dermatomyositis
Q/Q(M)-480467 Report a Problem

Twenty-nail dystrophy, nail plate splitting and pterygium formation are nail changes seen in:
1

Lichen Planus
Twenty-nail dystrophy, nail plate splitting and pterygium formation are nail changes associated with Lichen
Planus. Darier's disease is associated with longitudinal red and white streaks of the nail plate and V-shaped
knicking distally. Psoriasis is associated with many nail abnormalities including pitting, onycholysis and oil-spots.
Scleroderma may cause nail fold capillary dilation and destruction while patients with dermatomyositis may
exhibit nail fold telangiectasias and frayed cuticles.
Q/Q(M)-480467 Report a Problem


137

The protein component of nodular amyloidosis is:
1

SAA protein
2

AL protein
3

Keratin
4

Collagen
5

Bp180
Q/Q(M)-474332 Report a Problem


The protein component of nodular amyloidosis is:
2

AL protein
Nodular amyloidosis presents as single or multiple nodules, often on the extremities. The primary protein
component is of the AL type.
Q/Q(M)-474332 Report a Problem


Which of the following is most associated with pruritus?
1

Purpura annularis telangiectodes
2

Gougerot Blum syndrome (pigmented purpuric lichenoid dermatitis)
3

Lichen aureus
4

Schamberg\'s disease
5

Ducas and Kapetanakis pigmented purpura
Q/Q(M)-482541 Report a Problem

Which of the following is most associated with pruritus?
5

Ducas and Kapetanakis pigmented purpura
138

Ducas and Kapetanakis pigmented purpura presents as eczematous patches with petechiae and hemosiderin
staining. Pruritus is common, and the lesions are often more extensive than the other pigmented purpuras.
Histologically, spongiosis is noted. The other listed pigmented purpuric eruptions tend to be asymptomatic,
although lesions of Schamberg\'s disease can rarely be associated with itch.
Q/Q(M)-482541 Report a Problem


A morbidly obese 52-year-old woman presents with dark brown velvety plaques and acrochordons on the neck
and axillae. Which of the following is most likely to be associated with her cutaneous findings?
1

Diabetes mellitus
2

Hypertension
3

Hypercholesterolemia
4

Hypertriglyceridemia
5

Peripheral vascular disease
Q/Q(M)-482815 Report a Problem

A morbidly obese 52-year-old woman presents with dark brown velvety plaques and acrochordons on the neck
and axillae. Which of the following is most likely to be associated with her cutaneous findings?
1

Diabetes mellitus
This obese patient has acanthosis nigricans and acrochordons, both of which may be associated with insulin
resistance and diabetes mellitus. While she may very well have hypertension, hyperlipidemia,
hypercholesterolemia, and peripheral vascular disease, these are not significantly related to her cutaneous
findings. Acanthosis nigricans may also be seen in patients with gastric adenocarcinoma.
Q/Q(M)-482815 Report a Problem

Patients with this HLA type, which has the most definitive association with psoriasis, have a relative risk of having
psoriasis that is 9-15 times normal.
1

HLA-B13
2

HLA-B17
3

HLA-Bw57
139

4

HLA-Cw6
5

HLA-B27
Q/Q(M)-480386 Report a Problem
Patients with this HLA type, which has the most definitive association with psoriasis, have a relative risk of having
psoriasis that is 9-15 times normal.
4

HLA-Cw6
HLA-Cw6 is the HLA type most definitively associated with psoriasis. It carries a relative risk 9-15 times normal.
Q/Q(M)-480386 Report a Problem

Which organic system is least likely to be involved in the attached image
1

Liver
2

Spleen
3

Genitourinary
4

Gastrointestinal tract
5

Bone marrow
Q/Q(M)-482133 Report a Problem


Which organic system is least likely to be involved in the attached image
3

Genitourinary
Attached image shows mastocytosis. The disease presents classically with tan-like or red brown macules or
papules. There are different forms of cutaneous mastocytosis, most common form is urticaria pigmentosa which
140

can appear in both children and adults and diffuse cutaneous mastocytosis which occur almost exclusively in
infants, solitary mastocytoma and telangiectasia macularis eruptiva perstans. Systemic involvement occurs most
commonly in adults and bone marrow,lumph nodes, liver, spleen and GIT are among the most common involved
systems whearas genitourinary and neuroendocrine systems are least likely to be involved.
Q/Q(M)-482133 Report a Problem
Relapsing polychondritis is characterized by circulating auto-antibodies to which of the following?
1

Collagen type IV
2

Collagen type VII
3

Collagen type II
4

Elastin
5

Chondroitin sulfate
Q/Q(M)-482843 Report a Problem

Relapsing polychondritis is characterized by circulating auto-antibodies to which of the following?
3

Collagen type II
Relapsing polychondritis is a chronic, recurrent rheumatic disease characterized by cartilaginous inflammation
involving the nose, ears, and trachea. A non-erosive inflammatory arthritis may also be present, and affected
patients my suffer from neurosensory hearing loss, tinnitus, and/or vertigo secondary to vestibular or cochlear
damage. Patients may have circulating auto-antibodies to type II collagen (found exclusively in cartilage) with
circulating titers corresponding to disease activity.
Q/Q(M)-482843 Report a Problem

A patient presents with recurrent crops of papules that ulcerate and then spontaneously heal. What
immunohistochemical stain would be helpful in making the diangosis?
1

CD 4
2

CD 20
3

CD 30
4

CD 56
141

5

CD 68
Q/Q(M)-476881 Report a Problem
A patient presents with recurrent crops of papules that ulcerate and then spontaneously heal. What
immunohistochemical stain would be helpful in making the diangosis?
3

CD 30
Lymphomatoid papulosis Lymphomatoid papulosis (LyP; Macaulay disease) is a chronic lymphoproliferative
disease of the skin characterized by recurrent crops of papules that may ulcerate. The papules heal
spontaneously over a period of 1-2 months and may leave a depressed scars. The T-cells in this disorder typically
stain positively for CD 30.
Q/Q(M)-476881 Report a Problem

Human orf, also known as ecthyma contagiosum, was diagnosed in a 43 year old farmer by an astute resident
dermatologist. Patient presented with a dome shaped, firm bulla with an umbilicated crust. Which of the
following virus is responsible?
1

Pox virus
2

HHV-8
3

Parapox virus
4

HPV 5,8
5

Mycobacterium bovis
Q/Q(M)-482788 Report a Problem
Human orf, also known as ecthyma contagiosum, was diagnosed in a 43 year old farmer by an astute resident
dermatologist. Patient presented with a dome shaped, firm bulla with an umbilicated crust. Which of the
following virus is responsible?
3

Parapox virus
Orf, caused by the parapox virus is usually contracted by direct exposure to infected sheep or goats. Milkers
nodules are caused by a closely related virus found in cows. Both of these viruses can be contracted by exposure
to fomites (fence post, soil) containing the virus. One to several lesions may develop usually on the hands or
forearm and generally resolve without therapy in 4-6 weeks.
Q/Q(M)-482788 Report a Problem

ACE inhibitors cause angioedema via stimulation of?
142

1

Bradykinins
2

Histamine
3

Prostaglandins
4

Leukotrienes
5

Complement
Q/Q(M)-477505 Report a Problem

ACE inhibitors cause angioedema via stimulation of?
1

Bradykinins
Bradykinins are responsible for angiotensin converting enzyme inhibitor induced angioedema.
Q/Q(M)-477505 Report a Problem

Periorbital purpura is a characteristic cutaneous manifestation of:
1

Sarcoidosis
2

Amyloidosis
3

Dermatomyositis
4

Lymphoma
5

Thrombocytopenia
Q/Q(M)-477160 Report a Problem

Periorbital purpura is a characteristic cutaneous manifestation of:
2

Amyloidosis
Primary systemic amyloidosis, often myeloma associated, is characterized by the deposition of amyloid fibrils
derived from immunoglobulin light chains. 40% of patients manifest with cutaneous findings including petechiae,
purpura, waxy translucent papules on the eyelids, diffuse alopecia, scleroderma and nail dystrophy.
143

Q/Q(M)-477160 Report a Problem

Sweet's syndrome may be caused by:
1

Bleomycin
2

Cytoxan
3

Granulocyte colony stimulating factor
4

Intravenous immune globulin
5

Methotrexate
Q/Q(M)-476958 Report a Problem

Sweet's syndrome may be caused by:
3

Granulocyte colony stimulating factor
Sweet's syndrome is a dramatic skin disease characterized by eruptions of tender, pseudovesicular coalescing
papules and plaques most commonly appearing on the face, neck and upper trunk. It is seen in association with
fever, leukocytosis and typically responds promptly to systemic steroid therapy. Cutaneous pathergy has been
described. The etiology of Sweet's syndrome is unclear. A hypersensitivity reaction to a bacterial, viral or tumor
antigen has been postulated. Nearly 20% of cases are seen in association with an underlying malignancy,
particularly hematologic malignancies, with acute myelogenous leukemia being the most common. In addition,
drug-related variants of Sweet's syndrome have been described. The following drugs have been implicated:
granulocyte colony-stimulating factor (G-CSF), all-trans retinoic acid, hydralazine, carbamazepine,
levonorgestrel/ethinyl estradiol, trimethoprim/sulfamethoxazole, and minocycline.
Q/Q(M)-476958 Report a Problem

A teenager comes into your office requesting laser hair removal of her significant facial hair. Before you agree to
treat her, you order which of the following laboratory analyses?
1

FSH and LH
2

Glucose and hemoglobin A1C
3

Ferritin and TIBC
4

Testosterone and DHEA-S
144

5

TSH and T3
Q/Q(M)-477384 Report a Problem

A teenager comes into your office requesting laser hair removal of her significant facial hair. Before you agree to
treat her, you order which of the following laboratory analyses?
4

Testosterone and DHEA-S
Hisrutism describes excessive terminal hair growth in areas on women that are normally found only in post-
pubescent males (such as beard, chest, inner thigh). Hypertrichosis refers to excessive hair density or length.
Hirsutism is under the influence of androgen stimulation. Testosterone and DHEA-S can be used to detect
excessive adrenal or ovarian androgen production. Other clinical manifestations of androgen excess in women
are acne and virilization.
Q/Q(M)-477384 Report a Problem

What is the minimum amount of time that you would advise this woman to avoid childbearing after completing a
course of isotretinoin?
1

1 week
2

1 month
3

6 months
4

1 year
5

3 years
Q/Q(M)-476827 Report a Problem

What is the minimum amount of time that you would advise this woman to avoid childbearing after completing a
course of isotretinoin?
2

1 month
145

Isotretinoin is a synthetic retinoid that is used primarily in the treatment of acne. Isotretinoin has a half-life of
approximately 20 hours. Women should be advised to avoid becoming pregnant for at least one month.
Q/Q(M)-476827 Report a Problem
A potentially dangerous side effect of spironolactone is:
1

Hypokalemia
2

Hyperkalemia
3

Hypercalcemia
4

Hyponatremia
5

Hypernatremia
Q/Q(M)-474313 Report a Problem
A potentially dangerous side effect of spironolactone is:
2

Hyperkalemia
Spironolactone is a potassium-sparing aldactone antagonist. Renal failure predisposes patients to potentially
dangerous hyperkalemia.
Q/Q(M)-474313 Report a Problem


Syndrome that is associated with disease shown in image is known as
1

SAPHO syndrome
2

Follicular occlusion tetrad syndrome
3

LEOPARD syndrome
4

NAME syndrome
5

PAPA syndrome
Q/Q(M)-481899 Report a Problem
146



Syndrome that is associated with disease shown in image is known as
5

PAPA syndrome
PAPA syndrome ( Pyogenic Arthritis, Pyoderma gangerosum and Acne) The image shows classic ulcerative
pyoderma gangrenosum with undermined violaceous gray border. Re-epithelialization occurs from the margins
and the ulcer heal usually with atrophic cribriform pigmented scar. Although the classic morphologic clinical
presentation of pyoderma gangrenosum is an ulceration, there are several variants (bullous, pustular, and
superficial granulomatous) which differ by their clinical presentation, location, and associated diseases.
Q/Q(M)-481899 Report a Problem
Efficacy of oral contraceptives is reduced by co-administration with which of the following drugs?
1

Minocycline
2

Doxycycline
3

Rifampin
4

TMP-SMX
5

All of these answers are correct.
Q/Q(M)-474303 Report a Problem

Efficacy of oral contraceptives is reduced by co-administration with which of the following drugs?
3

Rifampin
Rifampin is a potent hepatic microenzyme inducer and has been shown to reduce efficacy of oral contraceptives.
Anecdotal reports of decreased contraceptive efficacy on oral antibiotics have not been confirmed in the
literature. The putative mechanism is decreased absorption of hormones secondary to altered gut microflora.
Q/Q(M)-474303 Report a Problem
147


Which technique used for hemostasis may cause iatrogenic tattooing?
1

Monsels
2

Aluminium Cloride
3

Gel Foam
4

Cautery
5

Epinephrine
Q/Q(M)-482792 Report a Problem
Which technique used for hemostasis may cause iatrogenic tattooing?
1

Monsels
Monsels solution is also known as ferrous subsulfate and can be a cause of iatrogenic tattooing.
Q/Q(M)-482792 Report a Problem


Treatment of Raynaud's phenomenon includes:
1

Nifedipine
2

Metoprolol
3

UVB
4

Tetracyline
5

Simvastatin
Q/Q(M)-479614 Report a Problem

Treatment of Raynaud's phenomenon includes:
1

Nifedipine
Treatment of Raynaud's includes calcium channel blockers such as nifedipine 30-6-mg/d, antiplatelet aggregation
drugs (such as aspirin or dipyridamole), pentoxyphilline 400mg BID-TID and D-penicillamine. Losartan 50mg/d
148

may reduce frequency and severity of Raynaud's.
Q/Q(M)-479614 Report a Problem
Which of the following is NOT a feature associated with this condition?
1

Accelerated blanch response
2

White dermatographism
3

Anterior subcapsular cataracts
4

Keratoconus
5

Pityriasis alba
Q/Q(M)-480480 Report a Problem


The most effective treatment for symptomatic ocular rosacea is:
1

Lubricant eye drops
2

Combination topical antibiotic and corticosteroid eye drops
3

Dilute baby shampoo washes
4

Doxycycline
5

Artificial tears
Q/Q(M)-474326 Report a Problem


The most effective treatment for symptomatic ocular rosacea is:
149

4

Doxycycline
The most definitive treatment for symptomatic ocular rosacea is an oral tetracycline.
Q/Q(M)-474326 Report a Problem

A patient has allergic contact dermatitis to paraphenylenediamine. Which of the following allergens may show a
potential cross- reaction?
1

Adhesive
2

Sulfa drugs
3

Lidocaine
4

Fragrance
5

Nickel
Q/Q(M)-482568 Report a Problem

A patient has allergic contact dermatitis to paraphenylenediamine. Which of the following allergens may show a
potential cross- reaction?
2

Sulfa drugs
Paraphenylenediamine (PPD) may cross react with a number of substances, including azo dyes, benzocaine,
procaine, sulfa drugs, and para-aminobenzoic acid (PABA). PPD is often found in dark hair dyes and in modified
henna tattoos. Fragrance, nickel, lidocaine, and adhesive do not show cross-reaction with PPD.
Q/Q(M)-482568 Report a Problem

Patients of which HLA type are more prone to drug-induced lupus erythematosus when exposed to hydralazine?
1

HLA-Cw6
2

HLA-B7
3

HLA-DR3
4

HLA-DR4
5

HLA-B6
150

Q/Q(M)-480515 Report a Problem
Patients of which HLA type are more prone to drug-induced lupus erythematosus when exposed to hydralazine?
4

HLA-DR4
Drug-induced lupus erythematous has been associated with exposure to hydralazine, procainamide,
sulfonamides, penicillin, anticonvulsants, minocycline, and INH. It generally has a benign course. The presence of
anti-histone antibodies are closely associated with symptomatic disease. Hydralazine and procainamide are
common culprits, with slow acetylators of hydralazine (HLA-DR4) more prone.
Q/Q(M)-480515 Report a Problem
Which HLA is associated with psoriatic arthritis in disequilibrium, especially if spondylitis is present?
1

HLA-B17
2

HLA-B13
3

HLA-Bw57
4

HLA-Cw6
5

HLA-B27
Q/Q(M)-482284 Report a Problem

Which HLA is associated with psoriatic arthritis in disequilibrium, especially if spondylitis is present?
5

HLA-B27
HLA-B17 is associated with an earlier onset and more serious disease of psoriasis. HLA-Cw6 is the most definitive
associated HLA type of psoriasis, with a relative risk 9-15 times normal. HLA-B17 amd HLA-B13 are also associated
with psoriasis but not psoriatic arthiritis
Q/Q(M)-482284 Report a Problem
Androstenedione is produced by:
1

The adrenals
2

The ovaries
3

The adrenals and ovaries
4

Extraglandular conversion
151

5

The adrenals, ovaries, and extraglandular conversion
Q/Q(M)-480495 Report a Problem

Androstenedione is produced by:
3

The adrenals and ovaries
Androstenedione is produced by the adrenals and the ovaries. Testosterone is produced by the adrenals, ovaries,
and by extraglandular conversion of androstenedione and dehydroepiandrosterone.
Q/Q(M)-480495 Report a Problem
Which of the following is a clinical manifestation of neonatal lupus erythematosus:
1

Renal disease
2

Cerebral vascular accident
3

Arthritis
4

Thrombocytopenia
5

Neutrophilia
Q/Q(M)-474336 Report a Problem
Which of the following is a clinical manifestation of neonatal lupus erythematosus:
4

Thrombocytopenia
Neonatal lupus involves annular scaling macules and plaques appearing within the first few months of life in
babies born to mothers with LE or other autoimmune connective tissue diseases. In addition to the rash, babies
display congenital heart block, hepatic disease, and thrombocytopenia.
Q/Q(M)-474336 Report a Problem
Most often, xanthelasma is associated with:
1

An IgM paraprotein
2

An IgG paraprotein
3

No associated disease
4

Type I hyperlipoproteinemia
152

5

Type III hyperlipoproteinemia
Q/Q(M)-480551 Report a Problem
Most often, xanthelasma is associated with:
3

No associated disease
Xanthelasma are the most common type of xanthoma. They are usually present without any other disease, but
may occur concomitantly with other xanthomas, and can occur in types II and III hyperlipoproteinemias (familial
hypercholesterolemia, common hypercholesterolemia, and familial dysbetalipoproteinemia). They are also
common among women with biliary or hepatic disoders, and are also seen in myxedema, diabetes, and
phytosterolemia.
Q/Q(M)-480551 Report a Problem

Which of the following is a function of Propionibacterium acnes (P. acnes) in contributing to the pathogenesis of
acne?
1

Downregulation of TLR-2 expression
2

Inhibition of complement
3

Activation of TLR-2
4

Inhibition of Il-1
5

Reduction in free fatty acid levels
Q/Q(M)-480338 Report a Problem

Which of the following is a function of Propionibacterium acnes (P. acnes) in contributing to the pathogenesis of
acne?
3

Activation of TLR-2
P. acnes flourishes in the presence of sebum and makes an enzymatic lipase which cleaves triglycerides into free
fatty acids. Free fatty acids serve as an indirect measure of P. acnes populations on the skin. P. acnes itself is also
pro-inflammatory; it can activate complement, as well as neutrophil chemotaxis and activity. Il-1, a pro-
inflammatory cytokine, may promote follicular plugging and microcomedo formation. TLR-2 activation by P.
acnes leads to stimulation of pro-inflammatory cytokines.
Q/Q(M)-480338 Report a Problem

Which of the following medications is most likely to induce or exacerbate psoriasis in your patient.
153

1

Hydrochlorothiazide
2

Lisinopril
3

Amlodipine
4

Metoprolol
5

Diltiazem
Q/Q(M)-478626 Report a Problem
Which of the following medications is most likely to induce or exacerbate psoriasis in your patient.
4

Metoprolol
Several drugs have been incriminated as inducers of psoriasis, in particular lithium, B-Blockers, antimalarials, and
interferon. More recent additions include terbinafine, calcium channel blockers (nicardipine, nifedipine,
nisoldipine, verapamil, and diltiazem), captopril, glyburide, and lipi-lowering drugs such as gemfibrozil.
Q/Q(M)-478626 Report a Problem

The majority of patients with positive U1RNP autoantibodies have which diagnosis?
1

Mixed connective tissue disorder (MCTD)
2

Rheumatoid arthritis
3

Systemic lupus erythematosus
4

Sjogren's syndrome
5

Dermatomyositis
Q/Q(M)-480523 Report a Problem
The majority of patients with positive U1RNP autoantibodies have which diagnosis?
3

Systemic lupus erythematosus
Anti-U1RNP is found in 100% of patients with MCTD and in 30% of patients with SLE; patients with SLE and anti-
U1RNP also have other positive serologies. The majority of patients with positive U1RNP have SLE rather than
MCTD. The presence of U1RNP autoantibodies is associated with sclerodactyly, Raynaud's, and esophageal
dysmotility.
154

Q/Q(M)-480523 Report a Problem
What is deposited in the upper dermis in this condition?
1

AK
2

AL
3

Transthyretin
4

Beta2-microglobulin
5

AA
Q/Q(M)-482078 Report a Problem


What is deposited in the upper dermis in this condition?
1

AK
Lichen Amyloidosis is a primary cutaneous amyloidosis. Deposits of amyloid of believed to be derived from
necrotic keratinocytes. AL deposits are seen in systemic amyloidosis which is usually associated with a
monoclonal gammopathy or myeloma and the amyloid is derived from immunoglobulin light chains. Amyloid
deposits composed of AA are seen in secondary systemic amyloidosis such as familial Mediterranean fever and
Muckle Wells syndrome, amyloid deposits composed of transthyretin are seen in familial amyloidotic
polyneuropathy and amyloid deposits composed of Beta2-microglobulin are seen in hemodialysis patients
Q/Q(M)-482078 Report a Problem
A patient with inflammatory bowel disease develops acute tender juicy plaques on the head and neck, fever, and
malaise. The skin lesions respond well to prednisone. Data shows increased ESR and neutrophilic infiltrate with
dermal edema on skin biopsy. Which one of the following is a major criteria for this condition?
1

Fever and malaise
2

History of inflammatory bowel disease
155

3

Abrupt onset of plaques
4

Increased ESR
5

Good response to prednisone
Q/Q(M)-482394 Report a Problem
A patient with inflammatory bowel disease develops acute tender juicy plaques on the head and neck, fever, and
malaise. The skin lesions respond well to prednisone. Data shows increased ESR and neutrophilic infiltrate with
dermal edema on skin biopsy. Which one of the following is a major criteria for this condition?
3

Abrupt onset of plaques
This condition is called Sweet's syndrome, or acute febrile neutrophilic dermatosis. Diagnosis relies on two major
and two minor criteria. Major ones include 1) an abrupt onset of juicy painful plaques and bullae and 2)
neutrophilic infiltration in the dermis on pathology. Minor criteria include 1) presence of associated conditions ie
inflammatory bowel disease, infections, pregnancy, leukemia, etc 2) fever and malaise, 3) laboratory values ie
high ESR and CRP, and 4) excellent response to prednisone.
Q/Q(M)-482394 Report a Problem

The most appropriate treatment for the condition shown is:
1

Topical clindamycin 1% lotion
2

Topical metronidazole 0.75% gel
3

Azelaic acid 20% cream
4

Salicylic acid 2% wash
5

Adapalene 0.1% gel
Q/Q(M)-474885 Report a Problem
156



The most appropriate treatment for the condition shown is:
5

Adapalene 0.1% gel
This slide shows comedonal acne. Topical retinoids are the treatment of choice. Salicylic acid-containing cleansers
may be helpful adjuncts to treatment with mild comedolytic properties.
Q/Q(M)-474885 Report a Problem
In a well-designed study, the impact of psoriasis on health-related quality of life was found to be similar to which
of the following conditions?
1

Diabetes
2

Acne vulgaris
3

Onychomycosis
4

Seasonal allergic rhinitis
5

Hypercholesterolemia
Q/Q(M)-474299 Report a Problem

In a well-designed study, the impact of psoriasis on health-related quality of life was found to be similar to which
of the following conditions?
1

Diabetes
The emotional and physical burden of psoriasis is easily underestimated. Rapp et al found that patients with
psoriasis report reduced physical and mental functioning (heath related quality of life) similar to that seen in
patients with cancer, hypertension, arthritis, heart disease, diabetes, and depression.
157

Q/Q(M)-474299 Report a Problem

A 20 year old woman presents with a sunken appearance to her face and trunk with normal legs. The patient has
attributed her appearance to stress from college, but is seeing you because of her family's concern. What lab
finding would you expect in this patient
1

Decreased C3 nephritic factor
2

Decreased C3
3

Decreased C1
4

Increased C1
5

Increased C4
Q/Q(M)-482166 Report a Problem

A 20 year old woman presents with a sunken appearance to her face and trunk with normal legs. The patient has
attributed her appearance to stress from college, but is seeing you because of her family's concern. What lab
finding would you expect in this patient
2

Decreased C3
Acquired partial lipodystrophy (Barraquer-Simmons Syndrome) is characterized by an insidious progressive loss of
fat that usually begins in the face and scalp and progressed downward. Most patients with this form of
lipodystrophy have reduced levels of C3 resulting from circulation polyclonal IgG called "C3 nephritic factor"
which results in uncontrolled activation of C3 and contributes to renal damage.
Q/Q(M)-482166 Report a Problem
A 35 year old man has a history of intensely pruritic papules and vesicles on the extensor surfaces of his lower
extremities. Antibodies to epidermal transglutaminase are detected. What is this patient at risk for developing?
1

Non-Hodgkin lymphoma
2

Colon cancer
3

Lung cancer
4

CLL
5

Esophageal cancer
158

Q/Q(M)-482466 Report a Problem

A 35 year old man has a history of intensely pruritic papules and vesicles on the extensor surfaces of his lower
extremities. Antibodies to epidermal transglutaminase are detected. What is this patient at risk for developing?
1

Non-Hodgkin lymphoma
A number of studies have indicated an increased risk of non-Hodgkins lymphoma and enteropathy T-cell
lymphoma in patients with dermatitis herpetiformis. There is also increased prevalence of thyroid disease, type I
diabetes, and other autoimmune disorders, such as vitiligo, Addison's, and alopecia areata.
Q/Q(M)-482466 Report a Problem
Which of the following is associated with this scarring condition?
1

Thyroid disease
2

Acne conglobata
3

Sarcoidosis
4

Systemic lupus erythematosus
5

Lichen planus
Q/Q(M)-476824 Report a Problem

Which of the following is associated with this scarring condition?
2

Acne conglobata
Dissecting cellulitis is part of the follicular occlusion tetrad which also includes acne conglobata, hidradenitis
suppurtiva, and pilonidal cysts.
Q/Q(M)-476824 Report a Problem
Which of following medications may increase the likelihood of pseudotumor cerebri in patients taking oral
isotretinoin?
159

1

Spironolactone
2

Dapsone
3

Amoxicillin
4

Rifampin
5

Tetracycline
Q/Q(M)-480420 Report a Problem
Which of following medications may increase the likelihood of pseudotumor cerebri in patients taking oral
isotretinoin?
5

Tetracycline
Pseudotumor cerebri, or benign intracranial hypertension is more common in adolescent and young adult
women, but can occur in children. Medicines reported to be associated with the condition include vitamin A
analogues, tetracyclines, steroids (especially in withdrawal), nalidixic acid, sulphonamides, lithium, thyroxine,
growth hormone, amiodarone and tamoxifen. It most often presents with headache (90% of cases), pulsatile in
quality. Less frequent symptoms are visual disturbances and pulsatile tinnitus. Pseudotumor can also be
completely asymptomatic. The mechanism is not fully understood but current opinion favors impaired
reabsorption of cerebrospinal fluid. Of the medicines associated with the condition, minocycline is most
frequently reported in the literature followed by tetracycline and doxycycline. Isotretinion has been repored to
cause it and it is possible that the incidence of pseudotumor may increase if two or more drugs which might
cause it are used together. For this reason tetracyclines should not be prescribed concomitantly with oral
retinoids.
Q/Q(M)-480420 Report a Problem

Malignancy of the aerodigestive tract is most closely associated with which paraneoplastic dermatosis?
1

Bazex's syndrome
2

Paraneoplastic pemphigus
3

Sweet's syndrome
4

Acanthosis nigricans
5

Dermatomyositis
Q/Q(M)-480474 Report a Problem
160


Malignancy of the aerodigestive tract is most closely associated with which paraneoplastic dermatosis?
1

Bazex's syndrome
Bazex's syndrome (acrokeratosis paraneoplastica) is a paraneoplastic dermatosis associated with malignancy of
the aerodigestive tract. Bazex's syndrome classically presents with violaceous erythema involving the ears, nose,
hands and feet. Lesions progress to become hyperkeratotic and psoriasiform in appearance. The other answer
choices all represent paraneoplastic dermatoses, but none are as closely associated with aerodigestive tract
malignancies as Bazex's syndrome.
Q/Q(M)-480474 Report a Problem
A 69 year old man with a history of multiple myeloma develops a deposition disorder of his heart, tongue, GI
tract, and skin. What form of amyloid is most likely present?
1

AL
2

AA
3

Beta 2-microglobulin
4

Keratin
5

Colloid
Q/Q(M)-482756 Report a Problem
A 69 year old man with a history of multiple myeloma develops a deposition disorder of his heart, tongue, GI
tract, and skin. What form of amyloid is most likely present?
1

AL
Primary systemic amyloidosis involves the deposition of protein AL, which is derived from Ig light chains. AL is
also found in nodular amyloidosis produced by a plasmacytoma. 40% of patients with primary systemic
amyloidosis have skin involvement.
Q/Q(M)-482756 Report a Problem
The most common location of dermatofibrosarcoma protuberans is:
1

Trunk
2

Head and neck
3

Extremities
161

4

None of these answers are correct
5

head and neck and extremities
Q/Q(M)-474985 Report a Problem
The most common location of dermatofibrosarcoma protuberans is:
1

Trunk
Dermatofibrosarcoma protuberans is typically located on the trunk. The extremities are the second most
common location for this type of neoplasm.
Q/Q(M)-474985 Report a Problem
What is the most common ocular finding in this patient with this granulomatous disease?
1

Blepharitis
2

Acute anterior uveitis
3

Posterior uveitis
4

Keratoconus
5

Cataracts
Q/Q(M)-476820 Report a Problem


What is the most common ocular finding in this patient with this granulomatous disease?
2

Acute anterior uveitis
162

Sarcoidosis is a systemic disease characterized by the formation of non-caseating granulomas. Organs involved
include the skin, eyes, lungs, liver and spleen. The most common ocular finding is acute anterior uveitis. Other
findings may include posterior uveitis, blurred vision, and excessive lacrimation.
Q/Q(M)-476820 Report a Problem
A 9-year-old boy presents with calcinosis cutis, periorbital violaceous erythema, and periungual telangiectasias.
Which substance is most likely to be elevated in this patient?
1

Fodrin
2

Fibrillarin
3

Thrombospondin-1
4

Complement
5

Triglycerides
Q/Q(M)-482392 Report a Problem


A 9-year-old boy presents with calcinosis cutis, periorbital violaceous erythema, and periungual telangiectasias.
Which substance is most likely to be elevated in this patient?
3

Thrombospondin-1
This patient has juvenile dermatomyositis which can feature a heliotrope rash, periungual telangiectasias,
psoriasiform dermatitis, calcinosis and lipodystrophy. Thrombospondin-1 is a mediator of angiogeneis that is
increased in patients with juvenile dermatomyositis. Systemic sclerosis can demonstrate antibodies to fibrillarin.
Q/Q(M)-482392 Report a Problem
Which of the following would be an important diagnostic sign of this non-infectious disorder?
1

Lacrimal gland enlargement
2

Periungual fibromas
3

Cafe-au-lait macules
4

Peg shaped teeth
5

Dystopia canthorum
163

Q/Q(M)-476632 Report a Problem


Which of the following would be an important diagnostic sign of this non-infectious disorder?
1

Lacrimal gland enlargement
Sarcoidosis is a non-infectious granulomatous disorder of unclear etiology. Ocular manifestation are relatively
common in sarcoidosis and may include acute anterior uveitis (classic finding), posterior uveitis, lacrimal gland
enlargement, and conjuctival nodules.
Q/Q(M)-476632 Report a Problem


Which of the following statements is true regarding this entity?
1

80% of patients with early onset disease have a positive family history
2

Twin concordance (identical twins) is 55%
3

The inner root sheath and matrix of normal hair express MHC class I
4

The sisapho pattern involves the occipital scalp
5

Atopic dermatitis is a predictor of good prognosis
Q/Q(M)-476641 Report a Problem
164


Which of the following statements is true regarding this entity?
2

Twin concordance (identical twins) is 55%
Twin concordance (identical twins) is 55% in alopecia areata. There is a high frequency of positive family history,
especially in patients with early onset (37%). The inner root sheath and matrix are immune privileged, not
expressing MHC class I, and this privilege may collapse in alopecia areata. The sisapho or ophiasis inversus
pattern of alopecia areata is a bandlike pattern of the fronto parietotemporal scalp. Atopic dermatitis is a
predictor of poor prognosis in patients with alopecia areata.
Q/Q(M)-476641 Report a Problem
What is the most common digital manifestation of patients with systemic lupus erythematosus?
1

Periungual telangiectasia
2

Raynaud phenomenon
3

Splinter hemorrhages
4

Sclerodactyly
5

Digital pitting scars
Q/Q(M)-482870 Report a Problem


What is the most common digital manifestation of patients with systemic lupus erythematosus?
2

Raynaud phenomenon
This is the most common digital manifestation in patients with SLE and it could present in up to 30% of the
patients. It is a paroxysmal vasospam of digits in response to cold exposure or emotional stress.
Q/Q(M)-482870 Report a Problem
The arthritis of Behcet's disease is characteristically:
165

1

Symmetric, erosive polyarthritis
2

Asymmetric, erosive polyarthritis
3

Asymmetric, non-erosive polyarthritis
4

Asymmetric, erosive monoarthritis
5

Symmetric, non-erosive polyarthritis
Q/Q(M)-480528 Report a Problem

The arthritis of Behcet's disease is characteristically:
3

Asymmetric, non-erosive polyarthritis
Behcet's disease is characterized by recurrent oral ulceration plus 2 of the following: recurrent genital ulceration,
eye lesions (posterior uveitis), skin lesions, positive pathergy test. Clinical features include thrombosis of the
superior vena cava, thrombophlebitis, CNS lesions that give a picture of multiple sclerosis, and an asymmetric
non-erosive polyarthritis.
Q/Q(M)-480528 Report a Problem

Topical calcineurin inhibitors may offer several benefits over topical steroids in treating the scaly, lichenified
periorbital plaques in this patient with atopic dermatitis, including:
1

More cost effective
2

Quicker onset of action
3

Decreased irritant potential
4

Absence of reports of cutaneous atrophy
5

Once daily application
Q/Q(M)-474898 Report a Problem
166


Topical calcineurin inhibitors may offer several benefits over topical steroids in treating the scaly, lichenified
periorbital plaques in this patient with atopic dermatitis, including:
4

Absence of reports of cutaneous atrophy
Tacrolimus and pimecrolimus are non-steroid anti-inflammatory drugs and are not associated with side effects
reported with use of topical corticosteroids, including atrophy, striae formation, telangiectasias, cataracts, and
HPA axis suppression.
Q/Q(M)-474898 Report a Problem
Which lab abnormality is most associated with sarcoidosis?
1

Hypercalcemia
2

Hyperkalemia
3

Hypernatremia
4

Hyperchloremia
5

Hyperlipidemia
Q/Q(M)-482828 Report a Problem

Which lab abnormality is most associated with sarcoidosis?
1

Hypercalcemia
Up to 10 % of patients with sarcoidosis may have hypercalcemia. Resultant hypercalciuria and nephrolithiasis
may lead to renal failure. The other listed lab abnormalities are not typically associated with sarcoidosis.
Q/Q(M)-482828 Report a Problem

A 25 year-old previously healthy man presents with the skin findings shown, urethritis, and one month of
peripheral arthritis. Which of the following is true regarding this condition?
167

1

Females and males are equally affected
2

A chronic deforming arthritis occurs in 20%
3

TNF-alpha inhibiting agents have no role in the treatment of this condition
4

Patients must have urethritis, conjunctivitis, and arthritis for diagnosis
5

Chlamydia cervicitis is not associated with this condition
Q/Q(M)-480393 Report a Problem


A 25 year-old previously healthy man presents with the skin findings shown, urethritis, and one month of
peripheral arthritis. Which of the following is true regarding this condition?
2

A chronic deforming arthritis occurs in 20%
The patient has Reiter's syndrome. Reiter's syndrome is a chronic inflammatory disease similar to psoriasis with
psoriatic arthritis, and is thought to be a variant form. The classic triad consists of urethritis, conjunctivitis, and
arthritis. Few patients present with the classic triad, and thus the syndrome can be diagnosed with peripheral
arthritis >1 month duration and associated urethritis (or cervicitis). It occurs in young men with the HLA-B27
genotype and rarely occurs in women. Skin findings include keratoderma blennorrhagicum and circinate balanitis
(in men), as well as oral erosions, severe stomatitis, and nail changes. The course of disease is marked by
exacerbation and remission; a chronic deforming arthritis occurs in 20%. Treatment includes topical steroids,
NSAIDs, methotrexate, acitretin, cyclosporine, and TNF-inhibiting biologics, such as etanercept.
Q/Q(M)-480393 Report a Problem

An elderly gentleman with rheumatoid arthritis has a 2-month history of recurrent painful, red, swollen ears and
hearing loss. Physical exam is notable for sparing of the earlobes. You suspect he has:
1

Chondrodermatitis nodularis helices
2

Recurrent otitis externa
168

3

Relapsing polychondritis
4

Systemic lupus erthematosus
5

Severe seborrheic dermatitis
Q/Q(M)-477200 Report a Problem

An elderly gentleman with rheumatoid arthritis has a 2-month history of recurrent painful, red, swollen ears and
hearing loss. Physical exam is notable for sparing of the earlobes. You suspect he has:
3

Relapsing polychondritis
Relapsing polychondritis is a rare disease manifested by recurring inflammation of cartilaginous tissue. Antibodies
to type II collagen are thought to be pathogenic in this disease. Clinically, patients have auricular chondritis and
arthritis. The chondritis is limited to the cartilaginous portion of the external ears. Involvement of the
cartilaginous portions of the eye, respiratory tract, the inner ear, and the cardiovascular system has been
reported as well. A significant portion of patients with relapsing polychondritis have an associated rheumatic or
autoimmune disease.
Q/Q(M)-477200 Report a Problem

Which special stain is utilized to confirm the diagnosis of Langerhans cell histiocytosis?
1

H&E sufficient for diagnosis
2

S100
3

CD20
4

CD1a
5

Chloroacetate esterase
Q/Q(M)-477179 Report a Problem
Which special stain is utilized to confirm the diagnosis of Langerhans cell histiocytosis?
4

CD1a
Langerhan cell histiocytosis includes the diseases Letterer-Siwe, Hand-Schuller-Christian, Eosinophilic Granuloma,
and Hashimoto-Pritzker. Cells stain positively with CD1a, S100, and peanut agglutinin. Cells may also be
indentified by the presence of Birbeck granules.
169

Q/Q(M)-477179 Report a Problem
What is the most common cause of erythroderma in non-HIV patients?
1

Drugs
2

Underlying malignancy
3

Pre-existing dermatoses
4

Infection
5

Idiopathic
Q/Q(M)-482155 Report a Problem
What is the most common cause of erythroderma in non-HIV patients?
3

Pre-existing dermatoses
Pre-existing dermatoses is most common cause in non-HIV patients, including atopic dermatitis, psoriasis,
seborrheic dermatitis, chronic actinic dermatitis, mycosis fungoides, pityriasis rubra pilaris, and allergic contact
dermatitis. Drugs are the second most common overall, and the most common in HIV patients
Q/Q(M)-482155 Report a Problem
A 32 year old woman, now 12 weeks pregnant, presents to your office with pruritic scaly papules and plaques. A
biopsy reveals focal spongiosis and parakeratosis in mounds, a superficial perivascular dermatitis, and
extravasated red blood cells in the dermis. Which of the following is true?
1

It has been associated with EBV.
2

There is often a flare post-partum and during subsequent pregnancies.
3

There is no increased incidence in immunocompromised patients.
4

There is an increased risk of miscarriage in mothers who developed pityriasis rosea within the first 26
weeks of their pregnancy
5

Acyclovir may be effective in this condition
Q/Q(M)-482492 Report a Problem
A 32 year old woman, now 12 weeks pregnant, presents to your office with pruritic scaly papules and plaques. A
biopsy reveals focal spongiosis and parakeratosis in mounds, a superficial perivascular dermatitis, and
extravasated red blood cells in the dermis. Which of the following is true?
170

5

Acyclovir may be effective in this condition
The question describes a case of pityriasis rosea, which has been associated with HHV6,7. There is an increased
incidence in immunocompromised patients. There is an increased risk of miscarriage in women who develop
pityriasis rosea before 15 weeks of gestation. Acyclovir may be effective in the treatment of pityriasis rosea,
especially if treated in the first week of presentation (JAAD 2006;54(1):82-5).
Q/Q(M)-482492 Report a Problem

Which of the following is true regarding this entity?
1

Extragenital lesions commonly involve the torso and are usually pruritic
2

Childhood onset is reported in ~5% of cases
3

20% of both men and women with this diagnosis have at least one autoimmune disease
4

Girls outnumber boys 2:1
5

Genital disease represents 50% of childhood cases
Q/Q(M)-480491 Report a Problem


Which of the following is true regarding this entity?
3

20% of both men and women with this diagnosis have at least one autoimmune disease
The patient pictured has lichen sclerosus. This likely represents an autoimmune phenomenon, as 20% of both
men and women have at least one autoimmune disease (vitiligo, alopecia areata, or thyroid disease), and a larger
proportion have circulating antibodies. Extragenital lesions most frequently involve the torso and are usually
asymptomatic. Childhood onset occurs in 10-15% of cases, and girls outnumber boys 10:1. Genital disease
171

represents 90% of childhood lichen sclerosus.
Q/Q(M)-480491 Report a Problem
Acute hemorrhagic edema of childhood often presents initially with:
1

Facial edema
2

Laryngospasm
3

Acute abdomen
4

Hematuria
5

Hematochezia
Q/Q(M)-480532 Report a Problem
Acute hemorrhagic edema of childhood often presents initially with:
1

Facial edema
Acute hemorrhagic edema of childhood affects children and infants < 2 years of age. It presents with painful,
edematous petechiae and ecchymoses on the head and distal extremities. Facial edema may be the initial sign.
Triggering factors include infection, drugs, and immunization. It lacks systemic features and resolves in 1-3 weeks
without sequelae.
Q/Q(M)-480532 Report a Problem
Which antibody is specific for CREST syndrome?
1

Anti-mitochondrial
2

Anti-histone
3

Anti-ds DNA
4

Anti-nucleolar
5

Anti-centromere
Q/Q(M)-477319 Report a Problem
Which antibody is specific for CREST syndrome?
5

Anti-centromere
172

The antinuclear antibody (ANA) pattern most specific for CREST is the anti-centromere pattern. The specificity
rate is approximately 50-90% and carries a more favorable prognosis than Scl-70. The target protein for the anti-
centromere pattern is the kinetochore.
Q/Q(M)-477319 Report a Problem
This 35 year-old man presents with the lesions shown. He was treated elsewhere for a different skin condition.
Biopsy of these lesions is likely to show:
1

Increased staining on Fontana Masson but not Perls stain
2

Increased staining on Perls stain but not Fontana Masson
3

Increased staining on both Fontana Masson and Perls stain
4

Increased melanin at the basal layer and within macrophages only
5

Fibrosis and increased mucin deposition
Q/Q(M)-480378 Report a Problem




This 35 year-old man presents with the lesions shown. He was treated elsewhere for a different skin condition.
Biopsy of these lesions is likely to show:
3

Increased staining on both Fontana Masson and Perls stain
The patient has Type 2 minocycline-associated hyperpigmentation. Three types of minocycline-associated
hyperpigmentation are generally described. The first is blue-black discoloration appearing in areas of prior skin
injury, such as acne scars. The second type is a blue-gray discoloration, often on the lower anterior legs and
forearms. The third type is the least common, and is characterized by muddy brown discoloration of sun-exposed
173

areas. The first two types show staining for both iron and melanin (Fontana Masson stains melanin black; Perls
stains iron [hemosiderin] blue). The third type shows increased melanin at the basal layer and within
macrophages.
Q/Q(M)-480378 Report a Problem
First line treatment of chromoblastomycosis includes surgery and a medication with which mechanism of action?
1

Inhibits squalene epoxidase
2

Inhibits 14-alpha-demethylase
3

Disrupts microtubule mitotic spindle formation
4

Inhibits sythesis of beta-1,3-diglucan and disrupts cell walls
5

Inhibits fungal cytochrome P-450 mediated 14 alpha-lanosterol demethylation
Q/Q(M)-482344 Report a Problem

First line treatment of chromoblastomycosis includes surgery and a medication with which mechanism of action?
2

Inhibits 14-alpha-demethylase
The correct answer is surgery and itraconazole, which inhibits 14-alpha-demethylase. Terbinafine inhibits
squalene epoxidase. Griseofulvin disrupts microtubule mitotic spindle formation. Caspofungin inhibits synthesis
of beta-1,3-diglucan. Foscarnet inhibits fungal cytochrome P-450 mediated 14 alpha-lanosterol demethylation.
Q/Q(M)-482344 Report a Problem

A 35 year-old otherwise healthy man presents with moderate-to-severe plaque psoriasis, improved on
cyclosporine. The patient weighs 70 kilograms and is taking a dose of 300 mg/day. His baseline creatinine was
0.8; on follow-up testing it is 1.1. All other exam and laboratory parameters are within normal limits. The patient
is pleased with his treatment and asks to continue it. Which of the following is correct?
1

The patient has exceeded the recommeded dosage of cyclosporine
2

You offer a decrease of the cyclosporine dose to 225 mg/day and close follow-up
3

The change in creatinine is not significant, no change is needed
4

Cyclosporine rarely has renal toxicity in young, healthy individuals; thus you must work-up other causes of
the increased creatinine
5

The cyclosporine should be stopped immediately; the patient must avoid cyclosporine in the future
174

Q/Q(M)-480391 Report a Problem
A 35 year-old otherwise healthy man presents with moderate-to-severe plaque psoriasis, improved on
cyclosporine. The patient weighs 70 kilograms and is taking a dose of 300 mg/day. His baseline creatinine was
0.8; on follow-up testing it is 1.1. All other exam and laboratory parameters are within normal limits. The patient
is pleased with his treatment and asks to continue it. Which of the following is correct?
2

You offer a decrease of the cyclosporine dose to 225 mg/day and close follow-up
Cyclosporine is highly effective in most patients with severe chronic plaque-type psoriasis. Doses start at 2.5 to
4mg/kg/day and can go as high as 5.5mg/kg/day. Renal impairment may occur and is often reversible. If the
creatinine increases 30% or greater from baseline the dose should be reduced by 25% and the patient followed
closely.
Q/Q(M)-480391 Report a Problem
What complication can occur if this disease is left untreated?
1

Development of a squamous cell carcinoma
2

Testicular torsion
3

Erectile dysfunction
4

Impotence
5

Pseudo-ainhum
Q/Q(M)-476813 Report a Problem
175



What complication can occur if this disease is left untreated?
1

Development of a squamous cell carcinoma
Lichen sclerosis is most commonly located in the genitalia. Longstanding lichen sclerosis is thought to be a risk
factor for the development of a squamous cell carcinoma.
Q/Q(M)-476813 Report a Problem
Which antibody is associated with a positive prognosis in dermatomyositis?
1

Anti-Jo
2

Anti-Smith
3

SRP
4

Anti-Mi
5

Aldolase
Q/Q(M)-482855 Report a Problem
Which antibody is associated with a positive prognosis in dermatomyositis?
4

Anti-Mi
Several antibodies have prognostic importance in dermatomyositis. Anti-Mi is associated with an improved
176

prognosis. Anti-Jo is associated with a poor prognosis and lung involvement. SRP is also associated with a poor
prognosis and heart involvement.
Q/Q(M)-482855 Report a Problem
Nekam's Disease:
1

Is generally responsive to topical and intralesional steroids
2

Characteristically lacks scale
3

Rarely involves the buttocks
4

Presents with a reticulate pattern on the dorsal hands and feet
5

Presents with hypopigmented, atrophic lesions on the extremities
Q/Q(M)-480398 Report a Problem
Nekam's Disease:
4

Presents with a reticulate pattern on the dorsal hands and feet
Nekam's Disease (keratosis lichenoides chronica) presents with violaceous papules and nodules, hyperpigmented
and hyperkeratotic, covered with gray scales. There is often a linear and reticulate pattern on the dorsal hands
and feet, extremities and buttocks. This condition is generally very refractory to treatment.
Q/Q(M)-480398 Report a Problem
What bacteria is thought to possibly play a role in the pathogenesis of this disease?
1

B. burgdorferi
2

H. pylori
3

E. coli
4

S. aureus
5

S. enteritidis
Q/Q(M)-476891 Report a Problem
177



What bacteria is thought to possibly play a role in the pathogenesis of this disease?
1

B. burgdorferi
Morphea is a inflammatory disease primarily of the dermis and subcutaneous fat that ultimately leads to a scar
like sclerosis. Clinically, morphea can be divided into plaque, linear, and generalized. There is some thought that
Borrelia infection may play a role in morphea. mRNA specific for Borrelia has been found in some lesions of
morphea. This association however is controversial.
Q/Q(M)-476891 Report a Problem
In cryosurgery, the temperature needed for destruction of keratinocytes is?
1

0-10 degrees C
2

-20- -30 degrees C
3

-5 degrees C
4

-50 degrees C
5

-196 degrees C
Q/Q(M)-482906 Report a Problem


178

In cryosurgery, the temperature needed for destruction of keratinocytes is?
2

-20- -30 degrees C
Cryosurgery is the application of low temperature to produce local tissue destruction. -20- -30 degrees C is
needed to destroy keratinocytes. -4- -7 degrees are needed to destroy melanocytes. Temperatures of -50 are
recommended to treat malignant lesions.
Q/Q(M)-482906 Report a Problem

Which of the following is NOT associated with this disease of symmetric induration caused mucin deposition?
1

Diabetes mellitus
2

Streptococcal infection
3

Monoclonal gammopathy
4

Hepatitis C
5

All of the answers are associated with this disease
Q/Q(M)-476629 Report a Problem


Which of the following is NOT associated with this disease of symmetric induration caused mucin deposition?
4

Hepatitis C
Scleredema is a type of dermal degenerating mucinosis characterized by diffuse symmetric induration of the
upper body. 3 types of scleredema have been described. The first type is seen in children following a stretococcal
infection. The second type is associated with a monoclonal gammopathy. The third type is related to insulin
179

dependent diabetes.
Q/Q(M)-476629 Report a Problem
In lichen planus pemphigoides:
1

Bullae develop characteristically in lesions of longstanding lichen planus
2

Circulating IgG antibodies react to the 230 kDa antigen within the basement membrane zone
3

There is granular deposition of IgG and C3 at the dermoepidermal junction
4

Bullae result from intense lichenoid inflammation and extensive liquefactive degeneration of basal
keratinocytes
5

Vesicles may develop de novo on previously uninvolved skin
Q/Q(M)-480397 Report a Problem

In lichen planus pemphigoides:
5

Vesicles may develop de novo on previously uninvolved skin
Lichen planus pemphigoides presents with tense blisters atop lesions of LP or de novo on uninvolved skin. It can
be differentiated from bullous LP, where blisters develop in lesions of longstanding LP as a result of intense
lichenoid inflammation and extensive liquefactive degeneration of basal keratinocytes. Histologically it resembles
LP, with linear deposition of IgG and C3 at the DE junction. Circulating IgG autoantibodies react to a 180/200 kDa
antigen within the basement membrane zone.
Q/Q(M)-480397 Report a Problem
Patients with xanthoma disseminaturn may demonstrate:
1

Elevated serum beta lipoproteins
2

Sparing of the oral mucosa
3

Involvement of the pituitary gland
4

Sparing of the flexural areas
5

Increased risk of malignant degeneration
Q/Q(M)-477125 Report a Problem

Patients with xanthoma disseminaturn may demonstrate:
180

3

Involvement of the pituitary gland
Xanthoma disseminatum is a rare mucocutaneous disease of discrete yellowish-red to brown papules in the
axillary and inguinal folds. Patient are generally normolipidemic. Involvement of the pituitary gland may occur,
leading to diabetes insipidus.
Q/Q(M)-477125 Report a Problem

A 52 year-old man presents with large comedones as well as inflammatory papules, pustules, and cysts on the
malar cheeks, postauricular scalp, and scrotum. Which of the following could be a cause of this presentation?
1

Erlotinib chemotherapy
2

A pituitary adenoma
3

2,3,7,8 tetrachlorobenzodioxin exposure
4

PTEN mutations
5

Isotretinoin overdose
Q/Q(M)-480374 Report a Problem

A 52 year-old man presents with large comedones as well as inflammatory papules, pustules, and cysts on the
malar cheeks, postauricular scalp, and scrotum. Which of the following could be a cause of this presentation?
3

2,3,7,8 tetrachlorobenzodioxin exposure
This patient has a presentation consistent with industrial acne. Dioxin (2,3,7,8 tetrachlorobenzodioxin) is a well-
known, potent trigger of this acneiform eruption. Overall, insoluble cutting oils are the most frequent cause of
industrial acne.
Q/Q(M)-480374 Report a Problem

The protein component of primary cutaneous amyloidosis is:
1

SAA protein
2

AL protein
3

Keratin
4

Collagen
181

5

Bp180
Q/Q(M)-474331 Report a Problem

The protein component of primary cutaneous amyloidosis is:
3

Keratin
Primary cutaneous amyloidosis presents as either macular or lichen amyloidosis. The protein component is
keratin. Macular amyloidosis often presents over the upper back, while lichen amyloid presents over the shins.
Q/Q(M)-474331 Report a Problem
What type of collagen is overrepresents in this lesion?
1

Collagen I
2

Collagen III
3

Collagen IV
4

Collagen V
5

Collagen VII
Q/Q(M)-476815 Report a Problem


What type of collagen is overrepresents in this lesion?
2

Collagen III
182

Keloids are dense overgrowths of fibrous tissue that usually develop after injury to the skin. Collagen III is
overrepresented in these keloids.
Q/Q(M)-476815 Report a Problem
Which of the following is an innate antimicrobial peptide expressed by keratinocytes in response to injury or
inflammation?
1

A-defensin 1
2

A-defensin 2
3

TNF-A
4

IL-4
5

IL-13
Q/Q(M)-474306 Report a Problem

Which of the following is an innate antimicrobial peptide expressed by keratinocytes in response to injury or
inflammation?
2

A-defensin 2
Human A-defensins (HBD) and cathelicidins are innate antimicrobial peptides in human skin. HBD-1 is
constitutively expressed. HBD-2 (and the cathelicidin LL 37) is expressed in response to skin injury and
inflammation.
Q/Q(M)-474306 Report a Problem

The hyperproliferative epithelium of this mature psoriasis plaque is associated with increased expression of
which keratin(s)?
1

K1, K10
2

K5, K14
3

K6, K16
4

K17
5

K2e
183

Q/Q(M)-474893 Report a Problem


The hyperproliferative epithelium of this mature psoriasis plaque is associated with increased expression of
which keratin(s)?
3

K6, K16
Expression of keratins K6 and K16 is upregulated in hyperprolferative psoriasis plaques.
Q/Q(M)-474893 Report a Problem


Attached image can be associated with all of the followings excpet:
1

Smooth muscle hamartoma
2

Unilateral breast hypoplasia
3

Acneform lesion
4

Skeletal defect
5

Cardiac defect
Q/Q(M)-481910 Report a Problem
184



Attached image can be associated with all of the followings excpet:
5

Cardiac defect
Becker's nevus can be occasionally associated with smooth muscle hamartoma, hypoplasia of underlying
structures, such as unilateral breast hypoplasia, unilateral or ipsilateral pectoralis major aplasia, ipsilateral limb
shortening, ipsilateral foot enlargement, spina bifida, scoliosis, pectus carinatum, localized lipoatrophy,
congenital adrenal hyperplasia, polythelia, and accessory scrotum. In addition to acneform lesions and
eczematous dermatitis. There is no association with cardiac defect.
Q/Q(M)-481910 Report a Problem
The Dunnigan variant of partial lipodystrophy is caused by a mutation in which gene?
1

AGPAT2
2

Seipin
3

Zinc metalloproteinase
4

Neutrophil elastase
5

LMNA
Q/Q(M)-482165 Report a Problem

The Dunnigan variant of partial lipodystrophy is caused by a mutation in which gene?
5

LMNA
The Dunnigan variant of partial lipodystrophy is characterized by normal appearance at birth followed by a
gradual loss of subcutaneous tissue from the arms and legs around the time of puberty. The genetic defect
involves the gene encoding lamins A and C (LMNA) which are intermediate filaments integral to the nuclear
envelope. The AGPAT2 and seipin genes are mutated in type I and type II congenital generalized lipodystrophies,
respectively. The gene for zinc metalloproteinase (ZMPSTE24) is mutated in mandibuloacral dysplasia.
185

Q/Q(M)-482165 Report a Problem
Immunohistochemistry of this disease would likely show:
1

Perivascular IgA
2

IgM staining of colloid bodies
3

Linear band of IgA at the basement membrane
4

IgM intercellularly
5

Granular C3 at the basement membrane
Q/Q(M)-476506 Report a Problem


Immunohistochemistry of this disease would likely show:
2

IgM staining of colloid bodies
Colloid bodies are apoptotic cell remnants in the papillary dermis. In lichen planus, the colloid bodies will often
stain positively with IgM.
Q/Q(M)-476506 Report a Problem


A patient recently underwent parotid surgery and now reports unilateral flushing and sweating around mealtime.
The nerve injured in this syndrome is a branch of the
186

1

Facial nerve
2

Maxillary nerve
3

Mandibular nerve
4

Cervical nerve
5

Frontal nerve
Q/Q(M)-482349 Report a Problem
A patient recently underwent parotid surgery and now reports unilateral flushing and sweating around mealtime.
The nerve injured in this syndrome is a branch of the
3

Mandibular nerve
This patient has Frey syndrome or auriculotemporal nerve syndrome. This is characterized by facial flushing,
sweating, or both localized to the distribution of the auriculotemporal nerve that occurs in response to gustatory
stimuli. In adults, the syndrome usually results from surgical injury or trauma to the parotid gland.
Q/Q(M)-482349 Report a Problem
Patient was complaining of skin irritation of her inner thigh. What is the most likely diagnosis?
1

Tinea corporis
2

Eczema
3

Molluscum contagiosum
4

Herpes infection
5

Warts
Q/Q(M)-482883 Report a Problem


Patient was complaining of skin irritation of her inner thigh. What is the most likely diagnosis?
3

Molluscum contagiosum
It is a poxvirus. Clinically, it presents with umbilicated papules. Generally affects the pediatric age group, but
when it affects groin or genital area is most likely due to sexual transmission and occurs more commonly in
adults.
187

Q/Q(M)-482883 Report a Problem
A 35-year-old man presents with scattered infiltrative cutaneous plaques, chronic fever, parotid gland
enlargement, and a facial nerve palsy. What other finding is most likely on physical exam?
1

Anterior uveitis
2

Periungual telangiectasia
3

Nail pits
4

Non-scarring alopecia
5

Condyloma lata
Q/Q(M)-482219 Report a Problem
A 35-year-old man presents with scattered infiltrative cutaneous plaques, chronic fever, parotid gland
enlargement, and a facial nerve palsy. What other finding is most likely on physical exam?
1

Anterior uveitis
Heerfordt-Waldenstrom Syndrome is a form of sarcoidosis consisting of fever, parotid enlargement, facial nerve
palsy, and anterior uveitis. Periungual telangiectasias are commonly seen in collage vascular diseases. Nail pits
are commonly seen in psoriasis and alopecia areata. Sarcoidosis involvement of the scalp often results in a
scarring alopecia. Condyloma lata is seen in secondary syphilis.
Q/Q(M)-482219 Report a Problem
Behcet's disease is associated with which HLA type?
1

HLA-B51
2

HLA-B17
3

HLA-B13
4

HLA-DR4
5

HLA-Cw6
Q/Q(M)-480465 Report a Problem

Behcet's disease is associated with which HLA type?
188

1

HLA-B51
Behcet's disease is associated with HLA-B51. HLA-Cw6 is the HLA type most closely associated with psoriasis. HLA-
B13 and HLA-B17 are also both associated with psoriasis; HLA-B17 is assiated with earlier onset and more serious
psoriasis.
Q/Q(M)-480465 Report a Problem

You prescribe oral erythromycin to a 35 year-old woman. Co-administration of which of the following
medications could lead to potential adverse outcomes?
1

Oral contraceptives
2

Warfarin
3

Carbamazepine
4

Methylprednisolone
5

Warfarin, carbamazepine, or methylprednisolone
Q/Q(M)-480380 Report a Problem

You prescribe oral erythromycin to a 35 year-old woman. Co-administration of which of the following
medications could lead to potential adverse outcomes?
5

Warfarin, carbamazepine, or methylprednisolone
Erythromycin inhibits the hepatic cytochrome P450 system and can increase serum levels and potential toxicities
of carbamazepine, theophylline, warfarin, digoxin, and methylprednisolone.
Q/Q(M)-480380 Report a Problem

A 32 year-old pregnant woman presents for treatment comedonal and moderately inflammatory acne. You
discuss treatment options with her, and she asks about the evidence for safety in pregnancy of various
treatments. According to the FDA classifications, which of the following treatments either shows no risk to the
fetus in controlled studies (but may show risk to animals), or shows no risk in animal studies (but no human
studies have been conducted)?
1

Benzoyl peroxide
2

Trimethoprim-sulfamethoxazole
189

3

Topical tretinoin
4

Azeleic acid
5

Tetracycline
Q/Q(M)-480376 Report a Problem
A 32 year-old pregnant woman presents for treatment comedonal and moderately inflammatory acne. You
discuss treatment options with her, and she asks about the evidence for safety in pregnancy of various
treatments. According to the FDA classifications, which of the following treatments either shows no risk to the
fetus in controlled studies (but may show risk to animals), or shows no risk in animal studies (but no human
studies have been conducted)?
4

Azeleic acid
The FDA classifies medication safety for use in pregnancy as follows: Category A: Controlled studies in humans
show no risk to fetus. Category B: Controlled human studies show no risk (but may show risk to animals), or no
risk in animal studies (but no human studies have been conducted). Category C: Risk to human fetus cannot be
ruled out, studies are lacking; animal studies are equivocal. Category D: Controlled studies show risk, but in some
instances benefits may outweigh risks. Category X: Contraindicated in pregnancy. The question describes
category B. Azeleic acid is a category B medication. Benzoyl peroxide, topical tretinoin, and TMP/SMX are
category C medications. Tetracycline is a category D medication.
Q/Q(M)-480376 Report a Problem
What is the recommended allowance of daily vitamin D intake for a 35-year-old female?
1

200 IU
2

400 IU
3

600 IU
4

800 IU
5

1000 IU
Q/Q(M)-482678 Report a Problem

What is the recommended allowance of daily vitamin D intake for a 35-year-old female?
3

600 IU
Recommended dietary allowance of vitamin D as established by the Institute of Medicine in 2010 is 400
international units for infants 0 to 12 months, 600 IU for ages 1 to 70 years, and 800 IU for ages 71 and above.
190

Additionally, recommended allowance for pregnant/lactating women is 600 IU.
Q/Q(M)-482678 Report a Problem

An overweight, post-menopausal woman presents with intense pain in the fatty deposits on her knees, thighs
and hips. She also describes swelling of her hands and feet, associated fatigue, and history of depression. On
clinical exam the pain in the adipose tissue appears out of proportion to the clinical findings. The diagnosis is:
1

Angiolipomas
2

Adiposis dolorosa
3

Congenital lipodystrophy
4

Lipodermatosclerosis
5

Erythema nodosum
Q/Q(M)-482527 Report a Problem

An overweight, post-menopausal woman presents with intense pain in the fatty deposits on her knees, thighs
and hips. She also describes swelling of her hands and feet, associated fatigue, and history of depression. On
clinical exam the pain in the adipose tissue appears out of proportion to the clinical findings. The diagnosis is:
2

Adiposis dolorosa
Adiposis dolorosa (Dercum\'s disease) is a disorder that usually occurs in obese women 40 to 60 years of age, and
it is characterized by pain in adipose tissue that appears to be out of proportion to the physical findings. The pain
can be localized to multiple, painful lipomas on lower extremities and knees. This condition is accompanied by
swelling of different areas of the body, such as hand and feet, which may be transient. Patients typically have
subjective fatigue or confusion, and may have history of depression or emotional instability. The cause of adiposis
dolorosa is not known. The cause of the pain is speculated to be due to the pressure on nerves by the adipose
tissue deposits. Therapeutic treatments are not very effective, and can involved procedures such as liposuction,
systemic corticosteroids, pregabalin, lidocaine, and psychiatric care.
Q/Q(M)-482527 Report a Problem

A 57yo male with hx of hypothyroidism and SCC of the head and neck s/p XRT several years ago presented with
this skin eruption present on his back and upper arms for the past several months. What is the most likely
diagnosis?
1

Tinea corporis
2

Granuloma annulare
191

3

Metastatic squamous cell carcinoma
4

Radiation dermatitis
5

Mycosis fungoides
Q/Q(M)-482888 Report a Problem


A 57yo male with hx of hypothyroidism and SCC of the head and neck s/p XRT several years ago presented with
this skin eruption present on his back and upper arms for the past several months. What is the most likely
diagnosis?
2

Granuloma annulare
It is a granulomatous inflammatory disease with infiltrates of macrophages arranged around focus of
degenerated collagen and mucin. The etiology is unknown: metabolic disturbances, autoimmune, allergy, and
infection.
Q/Q(M)-482888 Report a Problem
Cutaneous manifestations of vitamin D deficiency include:
1

Alopecia
2

Follicular hyperkeratosis
3

Edema
4

Angular cheilitis
5

Atrophic glossitis
Q/Q(M)-480553 Report a Problem


Cutaneous manifestations of vitamin D deficiency include:
1

Alopecia
Alopecia is the only cutaneous manifestation of vitamin D deficiency. Edema is the only cutaneous manifestation
of vitamine B1 (thiamine) deficiency (Beriberi). Follicular hyperkeratosis is seen in hypervitaminosis A. Angular
cheilitis is seen in vitamin B2 (riboflavin), B6 (pyridoxine), folic acid, niacin (vitamin B3 or nicotinic acid), biotin,
zinc, and iron deficiency. Atrophic glossitis is observed in vitamin B2 (riboflavin), vitamin B6 (pyridoxine), vitamin
192

B12, folic acid, and iron deficiency (among others).
Q/Q(M)-480553 Report a Problem

Acquired C1 esterase inhibitor deficiency results in:
1

A normal level of serum C1q
2

A decreased level of serum C1q
3

None of these answers are correct
4

A presentation of angioedema very early in life
5

A normal level of C4
Q/Q(M)-474334 Report a Problem

Acquired C1 esterase inhibitor deficiency results in:
2

A decreased level of serum C1q
Acquired C1 esterase inhibitor deficiency occurs in the setting of lymphoproliferative disease, monoclonal
gammopathy of undetermined significance, or rheumatologic disease and results in a decreased serum C1q level.
Inherited C1 esterase inhibitor deficiency shows an autosomal dominant inheritance, and earlier presentation,
and a normal level of serum C1q. In the inherited disease, the C1 esterase inhibitor may display normal levels but
be functionally impaired.
Q/Q(M)-474334 Report a Problem

A patient is on hemodialysis for end stage renal disease. After a few years of therapy, he develops carpal tunnel
syndrome, bone cysts, and spondyloarthropathy. What deposited substance is likely causing his symptoms?
1

Beta 2-microglobulin
2

Uric acid
3

AL
4

Triglyceride
5

AA
193

Q/Q(M)-482757 Report a Problem

A patient is on hemodialysis for end stage renal disease. After a few years of therapy, he develops carpal tunnel
syndrome, bone cysts, and spondyloarthropathy. What deposited substance is likely causing his symptoms?
1

Beta 2-microglobulin
The patient described likely has dialysis associated amyloidosis. Beta 2-microglobulin is the protein component
which is involved, and can become deposited in various tissues causing, among other thing, carpal tunnel
syndrome, bone cysts, and spondyloarthropathy.
Q/Q(M)-482757 Report a Problem

MAGIC syndrome involves:
1

Relapsing polychondritis
MAGIC syndrome is a combination of Behets disease and relapsing polychondritis. Patients show mouth and
genital ulcers with inflamed cartilage.
Q/Q(M)-474340 Report a Problem

P. acnes activation of which of the following may stimulate a pro-inflammatory cytokine cascade?
1

TLR-2
Toll-like receptors are a large group of receptors that recognize a variety of bacterial motifs. P. acnes has been
shown to activate TLR-2, leading to signal transduction and production of pro-inflammatory cytokines.
Q/Q(M)-474315 Report a Problem

Phrynoderma can be seen in all of the following nutritional deficiency except
1

Vitamin A
2

Vitamin B
3

Vitamin C
4

Vitamin D
5

Vitamin E
194

Q/Q(M)-482112 Report a Problem
Phrynoderma can be seen in all of the following nutritional deficiency except
4

Vitamin D
Phrynoderma or toad skin is typically associated with vitamin A deficiency. These keratotic follicular
papules often first develop on anterolateral thighs and posterolateral upper arms then spread to extremities,
shoulders, abdomen and back. Although phrynoderma is originally reported in association with vitamin A
deficiency, it can also be observed with defeciencies in B-complex vitamins and vitamins C and E, in addition to
essential fatty acid deficiency.
Q/Q(M)-482112 Report a Problem

This woman is being treated for corticosteroid-induced rosacea with topical metronidazole and an oral
tetracycline. She is very concerned about the redness of her face. Judicious use of what color concealer can
reduce the appearance of redness on the skin?
1

Yellow
2

Lavender
3

Pink
4

Bronze
5

Green
Q/Q(M)-474897 Report a Problem


This woman is being treated for corticosteroid-induced rosacea with topical metronidazole and an oral
tetracycline. She is very concerned about the redness of her face. Judicious use of what color concealer can
reduce the appearance of redness on the skin?
5

Green
195

This slide shows a woman with rosacea. Green and red are on opposite sides of the color wheel and thus can
cancel each other out. Green concealer can neutralize redness on the skin.
Q/Q(M)-474897 Report a Problem
The most notable histologic difference between the oral and cutaneous lesions of lichen planus is the presence in
oral LP of:
1

Acanthosis
2

Parakeratosis
3

Compact orthokeratosis
4

Wedge-shaped hypergranulosis
5

Vacuolar changes at the basal layer
Q/Q(M)-474899 Report a Problem

The most notable histologic difference between the oral and cutaneous lesions of lichen planus is the presence in
oral LP of:
2

Parakeratosis
This slide shows lichen planus of the buccal mucosa. Mucosal surfaces lack a granular layer. Parakeratosis, rather
than compact orthokeratosis, is prominent.
Q/Q(M)-474899 Report a Problem
Melanocytes can be found in all of the following except:
1

Nevus depigmentosa
2

Tyrosinase positive albinism
3

Nevus anemicus
4

Vitiligo
5

Postinflammatory hypopigmentation
Q/Q(M)-477371 Report a Problem

Melanocytes can be found in all of the following except:
196

4

Vitiligo
Vitiligo is an acquired disease in which there is total loss of pigment. The central process in vitiligo is the
destruction of melanocytes. With silver stains or dopa reaction, well established lesions of vitiligo are completely
devoid of melanocytes.
Q/Q(M)-477371 Report a Problem
Sarcoidosis presenting with fever, cough, joint pains, hilar adenopathy and erythema nodosum is known as:
1

Erythema contusiforme
2

Loeffler's syndrome
3

Lofgren's syndrome
4

Darier-Roussy sarcoid
5

Heerfordt's syndrome
Q/Q(M)-478705 Report a Problem
Sarcoidosis presenting with fever, cough, joint pains, hilar adenopathy and erythema nodosum is known as:
3

Lofgren's syndrome
Sarcoidosis presenting with fever, cough, joint pains, hilar adenopathy and erythema nodosum is known as
Lofgren's syndrome. Erythema nodosum is the most common nonspecific cutaneous finding in sarcoidosis.
Lofgren's syndrome occurs frequently in Scandinavian whites and is uncommon in American blacks.
Q/Q(M)-478705 Report a Problem

The pigmentation shown here is most likely due to which of the following drugs?
1

Chloroquine
2

Amiodarone
3

Minocycline
4

Doxycycline
5

TMP-SMX
Q/Q(M)-474889 Report a Problem
197



The pigmentation shown here is most likely due to which of the following drugs?
3

Minocycline
This slide shows blue-gray discoloration at the anterior shins and is characteristic of minocycline
hyperpigmentation.
Q/Q(M)-474889 Report a Problem

Das könnte Ihnen auch gefallen